Chapter 20: Assessment of Respiratory Function, prep U Nurs225, Assessment of Respiratory Function, Chapter 20: Assessment of Respiratory Function, Chapter 19, RESPIRATORY TEST, Respiratory Ch20-24 PrepU, PrepU: Chapter 19 - Respiratory, Chapter 19:…

¡Supera tus tareas y exámenes ahora con Quizwiz!

A nurse is teaching a client how to perform flow type incentive spirometry prior to his scheduled thoracic surgery. What instruction should the nurse provide to the client?

"Breathe in deeply through the spirometer, hold your breath briefly, and then exhale."

You are caring for a client admitted with chronic bronchitis. The client is having difficulty breathing, and the family asks you what causes this difficulty. What would be your best response?

"Conditions such as chronic bronchitis cause thickening of the bronchial mucosa so it makes it harder to breathe." Explanation: Conditions that may alter bronchial diameter and affect airway resistance include contraction of bronchial smooth muscle (e.g., asthma); thickening of bronchial mucosa (e.g., chronic bronchitis); airway obstruction by mucus, a tumor, or a foreign body; and loss of lung elasticity (e.g., emphysema). Option A is incorrect, not all chronic diseases make it hard to breathe. Option B is incorrect; not all chronic respiratory diseases caused scarring in the lung. Option C is incorrect; this response negates the families question and belittles their concern.

The nurse is caring for a client with an exacerbation of COPD and scheduled for pulmonary function studies using a spirometer. Which client statement would the nurse clarify?

"I will breathe in through my mouth and out through my nose." Explanation: The nurse would clarify the client's statement of improper breathing technique. During a pulmonary function test using a spirometer, a nose clip prevents air from escaping through the client's nose when blowing into the spirometer. All other statements are correct.

A nurse is instructing the client on the normal sensations that can occur when contrast medium is infused during pulmonary angiography. Which client statement demonstrates an understanding of the teaching? "I will feel light-headed when the contrast medium is introduced." "I will feel warm and may have chest pain" "I will feel waves of nausea throughout the procedure." "I will feel a dull pain when the catheter is introduced."

"I will feel warm and may have chest pain" During a pulmonary angiography, a contrast medium is injected into the femoral circulation. When the medium is infused, the client will feel warm and flushed, with a possibility of chest pain. The client will feel pressure when the catheter is inserted. The client does not typically feel light-headed or nauseated during this procedure.

A nurse is instructing the client on the normal sensations that can occur when contrast medium is infused during pulmonary angiography. Which client statement demonstrates an understanding of the teaching?

"I will feel warm and may have chest pain" Explanation: During a pulmonary angiography, a contrast medium is injected into the femoral circulation. When the medium is infused, the client will feel warm and flushed, with a possibility of chest pain. The client will feel pressure when the catheter is inserted. The client does not typically feel light-headed or nauseated during this procedure.

Following a chest X-ray, a patient has been diagnosed with a pleural effusion. The care team has concluded that the quantity of fluid in the patient's intrapleural space necessitates thoracentesis. What patient education should the nurse provide in anticipation of this procedure?

"It's very important that you remain still while the doctor is performing the procedure." To prevent pneumothorax, it is imperative that a patient remain immobile during thoracentesis. The patient should remain on bed rest after the procedure and should not cough during thoracentesis. The preferred positioning is with the patient upright; if this is not possible, side-lying may be used as an alternative.

A client on the medical unit has told the nurse that he is experiencing significant dyspnea, despite that he has not recently performed any physical activity. What assessment question should the nurse ask the client while preparing to perform a physical assessment?

"On a scale from 1 to 10, how bad would you rate your shortness of breath?" Gauging the severity of the client's dyspnea is an important part of the nursing process. Oral intake and nausea are much less important considerations. The nurse must perform assessment prior to interventions such as providing medication.

Computed tomography of a patient's chest is suggestive of a malignancy, but these results are not conclusive. As a result, the patient has been scheduled for a bronchoscopy. What patient education should the nurse provide for this patient regarding this diagnostic procedure?

"We'll monitor you closely after the procedure, especially until your gag reflex returns." After a bronchoscopy, it is important that the patient takes nothing by mouth until the cough reflex returns, because the preoperative sedation and local anesthesia impair the protective laryngeal reflex and swallowing for several hours. However, this normally takes a few hours, not one or two days. Bronchoscopy is normally performed under conscious sedation. Contrast solution is not used.

The divisions of the lung proceed in the following order, beginning at the mainstream bronchi?

-lobar bronchi -segmented bronchi -subsegmented bronchi -bronchioles

When performing endotracheal suctioning, the nurse applies suctioning while withdrawing and gently rotating the catheter 360 degrees for how long?

10 to 15 seconds Explanation: In general, the nurse should apply suction no longer than 10 to 15 seconds. Applying suction for 20-25 or 30-35 seconds is hazardous and may result in the development of hypoxia, which can lead to dysrhythmias and, ultimately, cardiac arrest. Applying suction for 0-5 seconds would provide too little time for effective suctioning of secretions.

A client has recently been diagnosed with malignant lung cancer. The nurse is calculating the client's smoking history in pack-years. The client reports smoking two packs of cigarettes a day for the past 11 years. The nurse correctly documents the client's pack-years as

22. Explanation: Smoking history is usually expressed in pack-years, which is the number of packs of cigarettes smoked per day times the number of years the patient smoked. In this situation, the client's pack-years is 22 (2 × 11). It is important to find out whether the client is still smoking or when the client quit smoking.

Inspection of a patient's skin color is part of the assessment of the integumentary system. Cyanosis, which is a late indicator of hypoxia, is present when the unoxygenated hemoglobin level is:

5 g/dL Explanation: Normal hemoglobin is approximately 15 g/dL. Cyanosis appears when a full one-third of the hemoglobin is deoxygenated.

The alveoli begin to lose elasticity at about age____years, resulting in decreased gas diffusion.

50

Tidal volume, which may significantly change with disease, has a normal value of approximately?

500mL

A client with a respiratory condition is receiving oxygen therapy. While assessing the client's PaO2, the nurse knows that the therapy has been effective based on which of the following readings?

84 mm Hg In general, clients with respiratory conditions are given oxygen therapy only to increase the arterial oxygen pressure (PaO2) back to the client's normal baseline, which may vary from 60 to 95 mm Hg.

A nurse understands that a safe but low level of oxygen saturation provides for adequate tissue saturation while allowing no reserve for situations that threaten ventilation. What is a safe but ow oxygen saturation level for a patient?

95 mm Hg

A nurse understands that a safe but low level of oxygen saturation provides for adequate tissue saturation while allowing no reserve for situations that threaten ventilation. What is a safe but low oxygen saturation level for a patient? 95% 80% 75% 40%

95% Normal SpO2 values are more than 95%. Values less than 90% indicate that the tissues are not receiving enough oxygen, in which case further evaluation is needed.

A nurse understands that a safe but low level of oxygen saturation provides for adequate tissue saturation while allowing no reserve for situations that threaten ventilation. What is a safe but low oxygen saturation level for a patient?

95% Explanation: Normal SpO2 values are more than 95%. Values less than 90% indicate that the tissues are not receiving enough oxygen, in which case further evaluation is needed.

A client is described as having pectus carinatum. What would be the physical manifestation of this condition? A. the sternum protrudes and the ribs are sloped backward B. the chest is rounded, ribs are horizontal, and the sternum is pulled forward C. the thoracic and lumbar spine have a lateral S-shaped curvature D. the sternum is depressed from the second intercostal space

A

A client who has aspirated a piece of candy is brought to the ED. Which stimulated structure is causing the client's unrelenting cough? A. carina B. trachea C. bronchoile D. bronchi

A

A nurse would question the accuracy of a pulse oximetry evaluation in which of the following conditions?

A client experiencing hypothermia Explanation: Pulse oximetry is a noninvasive method of continuously monitoring the oxygen saturation of hemoglobin. The reading is referred to as SpO2. A probe or sensor is attached to the fingertip, forehead, earlobe, or bridge of the nose. Values less than 85% indicate that the tissues are not receiving enough oxygen. SpO2 values obtained by pulse oximetry are unreliable in states of low perfusion such as hypothermia.

What would the critical care nurse recognize as a condition that may indicate a client's need to have a tracheostomy?

A client requires permanent ventilation. A tracheostomy permits long-term use of mechanical ventilation to prevent aspiration of oral and gastric secretions in the unconscious or paralyzed client. Indications for a tracheostomy do not include a respiratory rate of 10 breaths per minute, symptoms of dyspnea, or respiratory acidosis.

A client is on a ventilator. Alarms are sounding, indicating an increase in peak airway pressure. The nurse assesses first for

A kink in the ventilator tubing Explanation: One event that could cause the ventilator's peak-airway-pressure alarm to sound is a kink in the ventilator tubing. After making this and other assessments without correction, then it could be a malfunction of the alarm button. Higher than normal endotracheal cuff pressure could cause client tissue damage but would not make the ventilator alarms sound. A cut or slice in the tubing from the ventilator would result in decreased pressure.

The nurse is educating a PT who is scheduled for a perfusion lung scan. What should be included in the information about the procedure? Select all that apply

A mask will be placed over the nose and mouth during the test The PT will be expected to lie under the camera The imaging time will amount to 20-40 minutes

The ED nurse is assessing a client complaining of dyspnea. The nurse auscultates the client's chest and hears wheezing throughout the lung fields. What might this indicate about the client?

A narrowed airway. Wheezing is a high-pitched, musical sound that is often the major finding in a client with bronchoconstriction or airway narrowing. Wheezing is not normally indicative of pneumonia or hemothorax. Wheezing does not indicate the need for physiotherapy.

The nurse is caring for a client who is in respiratory distress. The physician orders arterial blood gases (ABGs) to determine various factors related to blood oxygenation. What site can ABGs be obtained from?

A puncture at the radial artery Explanation: ABGs determine the blood's pH, oxygen-carrying capacity, levels of oxygen, CO2, and bicarbonate ion. Blood gas samples are obtained through an arterial puncture at the radial, brachial, or femoral artery. A client also may have an indwelling arterial catheter from which arterial samples are obtained. Blood gas samples are not obtained from the pleural surfaces or trachea and bronchi.

A client with a diagnosis of thrombocytopenia has been admitted to your unit for IV corticosteroid treatment. As the nurse caring for this client, you would expect the physician's orders to include which of the following?

A tapering course of solumedrol at discharge Explanation: If instituting corticosteroid therapy in thrombocytopenia, the nurse observes the client for adverse drug effects. The dose and frequency of steroid medication is tapered before discontinuing it to avoid adrenal insufficiency or crisis. When a client is on high dose steroid treatment, a sleeping medication might be ordered; also a medication for steroid headaches, but not oxycodone. Clients on steroids are often hungry throughout the day. However, what you would expect to find in the orders is a tapering course of the steroids.

Not every structure in the upper airway has a purpose in respiration. There are some structures whose role is immunological. Which structures protect against infection? Select all that apply. A. palatine tonsils B. pharyngeal tonsils C. epiglottis D. nasopharynx

A, B

A client requires increased respiratory effort to breathe. Which factors may affect lung compliance? A. edema B. fibrosis C. chronic bronchitits D. decreased surfanctant E. mucus obstruction

A, B, D

After reviewing the pharmacological treatment for pulmonary diseases, the nursing student knows that bronchodilators relieve bronchospasm in three ways. Choose the correct three of the following options. a) Reduce airway obstruction b) Alter smooth muscle tone c) Decrease alveolar ventilation d) Increase oxygen distribution

A, B, D. Bronchodilators relieve bronchospasm by altering smooth muscle tone and reduce airway obstruction by allowing increased oxygen distribution throughout the lungs and improving alveolar ventilation

A nursing student knows that there are three most common symptoms of asthma. Select all that apply. a) Cough b) Crackles c) Dyspnea d) Wheezing

A, B, D. The three most common symptoms of asthma are cough, dyspnea, and wheezing. In some instances, cough may be the only symptom.

The nurse is assessing a patient for obstructive sleep apnea (OSA). Which of the following are signs and symptoms of OSA? Select all that apply. a) Insomnia b) Polycythemia c) Evening headaches d) Loud snoring e) Pulmonary hypotension

A, B, D. Signs and symptoms include excessive daytime sleepiness, frequent nocturnal awakening, insomnia, loud snoring, morning headaches, intellectual deterioration, personality changes, irritability, impotence, systemic hypertension, dysrhythmias, pulmonary hypertension, corpulmonale, polycythemia, and enuresis.

Following are statements regarding medications taken by a patient diagnosed with COPD. Choose which statements correctly match the drug name to the drug category. Select all that apply. a) Prednisone is a corticosteroid. b) Ciprofloxacin is an antibiotic. c) Bactrim is a bronchodilator. d) Decadron is an antibiotic. e) Albuterol is a bronchodilator.

A, B, E. Theophylline, albuterol, and atropine are bronchodilators. Dexamethasone (Decadron) and prednisone are corticosteroids. Amoxicillin, ciprofloxacin, and cotrimoxazole (Bactrim) are antibiotics. These are all drugs that could be prescribed to a patient with COPD

Upon assessment, the nurse suspects that a patient with COPD may have bronchospasm. What manifestations validate the nurse's concern? (Select all that apply.) a) Wheezes b) Ascites c) Decreased airflow d) Compromised gas exchange e) Jugular vein distention

A, C, D. Bronchospasm, which occurs in many pulmonary diseases, reduces the caliber of the small bronchi and may cause dyspnea, static secretions, and infection. Bronchospasm can sometimes be detected on auscultation with a stethoscope when wheezing or diminished breath sounds are heard. Increased mucus production, along with decreased mucociliary action, contributes to further reduction in the caliber of the bronchi and results in decreased airflow and decreased gas exchange. This is further aggravated by the loss of lung elasticity that occurs with COPD (GOLD, 2010).

A client with chronic bronchitis is admitted with an exacerbation of symptoms. During the nursing assessment, the nurse will expect which of the following findings? Select all that apply. a) Purulent sputum with frequent coughing b) Respiratory rate of 10 breaths per minute c) Use of accessory muscles to breathe d) Tympany percussed bilaterally over the lung bases e) Hypoventilatory breathing pattern

A, C. Chronic bronchitis increases airway resistance and can thicken bronchial mucosa during an exacerbation. The client will have dyspnea requiring the use of accessory muscles to breathe, along with tachypnea and sputum production. Bronchial irritation and the need to expectorate mucus will lead to coughing. Percussion in this client would lead to resonant or hyperresonant sounds

A patient comes to the clinic with a cold and wants something to help relieve the symptoms. What should the nurse include in educating the patient about the uncomplicated common cold? (Select all that apply.) a) Inform the patient about the symptoms of secondary infection. b) Inform the patient that taking an antihistamine will help to decrease the duration of the cold. c) Tell the patient to take prescribed antibiotics to decrease the severity of symptoms. d) Suggest that the patient take adequate fluids and get plenty of rest. e) Inform the patient that the virus is contagious for 2 days before symptoms appear and during the first part of the symptomatic phase.

A, D, E. Colds are highly contagious because virus is shed for about 2 days before the symptoms appear and during the first part of the symptomatic phase. Management consists of symptomatic therapy that includes adequate fluid intake, rest, prevention of chilling, and the use of expectorants as needed. The nurse instructs the patient about methods to treat symptoms of the common cold and provides both verbal and written information to assist in the prevention and management of URIs

A nurse caring for a patient with a pulmonary embolism understands that a high ventilation-perfusion ratio may exist. What does this mean for the patient? a) Ventilation exceeds perfusion. b) Perfusion exceeds ventilation. c) Ventilation matches perfusion. d) There is an absence of perfusion and ventilation.

A. A high ventilation-perfusion rate means that ventilation exceeds perfusion, causing dead space. The alveoli do not have an adequate blood supply for gas exchange to occur. This is characteristic of a variety of disorders, including pulmonary emboli, pulmonary infarction, and cardiogenic shock

The nurse auscultates the lung sounds of a patient during a routine assessment. The sounds produced are harsh and cracking, sounding like two pieces of leather being rubbed together. The nurse would be correct in documenting this finding as which of the following? a) Pleural friction rub b) Crackles c) Sonorous wheezes d) Sibilant wheezes

A. A pleural friction rub is heard secondary to inflammation and loss of lubricating pleural fluid. Crackles are soft, high-pitched, discontinuous popping sounds that occur during inspiration. Sonorous wheezes are deep, low-pitched rumbling sounds heard primarily during expiration. Sibilant wheezes are continuous, musical, high-pitched, whistlelike sounds heard during inspiration and expiration.

The nurse identifies which finding to be most consistent prior to the onset of acute respiratory distress? a) Normal lung function b) Chronic lung disease c) Loss of lung function d) Slow onset of symptoms

A. Acute respiratory failure occurs suddenly in clients who previously had normal lung function.

The nurse is caring for a patient with recurrent hemoptysis who has undergone a bronchoscopy. Immediately following the procedure, the nurse should complete which of the following? a) Assess the patient for a cough reflex. b) Ensure the patient remains moderately sedated to decrease anxiety. c) Offer the patient ice chips. d) Instruct the patient that bed rest must be maintained for 2 hours.

A. After the procedure, the patient must take nothing by mouth until the cough reflex returns, because the preoperative sedation and local anesthesia impair the protective laryngeal reflex and swallowing. Once the patient demonstrates a cough reflex, the nurse may offer ice chips and eventually fluids. The patient is sedated during the procedure, not afterward. The patient is not required to maintain bed rest following the procedure

The nurse is performing a nutritional assessment on a client who has been diagnosed with cancer of the larynx. Which laboratory values would be assessed when determining the nutritional status of the client? Select all that apply. A. Albumin level B. Glucose level C. Protein level

A. Albumin level B. Glucose level C. Protein level The nurse also assesses the client's general state of nutrition, including height and weight and body mass index, and reviews laboratory values that assist in determining the client's nutritional status (albumin, protein, glucose, and electrolyte levels). The white blood cell count and the platelet count would not normally assist in determining the client's nutritional status.

Asthma is cause by which type of response? a) IgE-mediated b) IgA-mediated c) IgD-mediated d) IgM-mediated

A. Atopy, the genetic predisposition for the development of an IgE-mediated response to allergens, is the most common identifiable predisposing factor for asthma. Chronic exposure to airway allergens may sensitize IgE antibodies and the cells of the airway

A nurse is preparing a client for bronchoscopy. Which instruction should the nurse give to the client? a) Don't eat. b) Don't walk. c) Don't cough. d) Don't talk.

A. Bronchoscopy involves visualization of the trachea and bronchial tree. To prevent aspiration of stomach contents into the lungs, the nurse should instruct the client not to eat or drink anything for approximately 6 hours before the procedure. It isn't necessary for the client to avoid walking, talking, or coughing

A nurse is aware that crackles, non-contiguous breath sounds, are assessed for a patient with: a) Pulmonary fibrosis b) Asthma c) A collapsed alveoli d) Chronic bronchitis

A. Crackles (formerly referred to as rales) are discrete, noncontinuous sounds that result from delayed reopening of deflated airways. Crackles may or may not be cleared by coughing. They reflect underlying inflammation or congestion and are often present in such conditions as pneumonia, bronchitis, heart failure, bronchiectasis, and pulmonary fibrosis. Crackles are usually heard on inspiration, but they may also be heard on expiration

A nurse is caring for a client who has a tracheostomy and temperature of 103° F (39.4° C). Which intervention will most likely lower the client's arterial blood oxygen saturation? a) Endotracheal suctioning b) Incentive spirometry c) Encouragement of coughing d) Use of a cooling blanke

A. Endotracheal suctioning removes secretions as well as gases from the airway and lowers the arterial oxygen saturation (SaO2) level. Coughing and using an incentive spirometer improve oxygenation and should raise or maintain oxygen saturation. Because of superficial vasoconstriction, using a cooling blanket can lower peripheral oxygen saturation readings, but SaO2 levels wouldn't be affected.

A client has just undergone bronchoscopy. Which nursing assessment is most important at this time? a) Level of consciousness (LOC) b) Memory c) Personality changes d) Intellectual ability

A. Following bronchoscopy, LOC is the most important assessment because changes in the client's LOC may alert the nurse to serious neurologic problems. Memory, personality changes, and intellectual ability are important but don't take precedence at this time.

Which type of oxygen therapy includes the administration of oxygen at pressure greater than 1 atmosphere? a) Hyperbaric b) High-flow systems c) Low-flow systems d) Transtracheal

A. Hyperbaric oxygen therapy is the administration of oxygen at pressures greater than 1 atmosphere. As a result, the amount of oxygen dissolved in plasma is increased, which increases oxygen levels in the tissues. Low-flow systems contribute partially to the inspired gas the patient breathes, which means that the patient breathes some room air along with the oxygen. High-flow systems are indicated for patients who require a constant and precise amount of oxygen. During transtracheal oxygenation, patients achieve adequate oxygenation at lower rates, making this method less expensive and more efficient

A home health nurse is visiting a home care client with advanced lung cancer. Upon assessing the client, the nurse discovers wheezing, bradycardia, and a respiratory rate of 10 breaths/minute. These signs are associated with which condition? A. Hypoxia B. Delirium C. Hyperventilation D. Semiconsciousness

A. Hypoxia. As the respiratory center in the brain becomes depressed, hypoxia occurs, producing wheezing, bradycardia, and a decreased respiratory rate. Delirium is a state of mental confusion characterized by disorientation to time and place. Hyperventilation (respiratory rate greater than that metabolically necessary for gas exchange) is marked by an increased respiratory rate or tidal volume, or both. Semiconsciousness is a state of impaired consciousness characterized by limited motor and verbal responses and decreased orientation.

A nurse is assessing a client who comes to the clinic for care. Which findings in this client suggest bacterial pneumonia? a) Dyspnea and wheezing b) Hemoptysis and dysuria c) Sore throat and abdominal pain d) Nonproductive cough and normal temperature

A. In a client with bacterial pneumonia, retained secretions cause dyspnea, and respiratory tract inflammation causes wheezing. Bacterial pneumonia also produces a productive cough and fever, rather than a nonproductive cough and normal temperature. Sore throat occurs in pharyngitis, not bacterial pneumonia. Abdominal pain is characteristic of a GI disorder, unlike chest pain, which can reflect a respiratory infection such as pneumonia. Hemoptysis and dysuria aren't associated with pneumonia

A patient has had a laryngectomy and was able to retain his airway, with no difficulty swallowing. There is no split of thyroid cartilage. The nurse would record this type of laryngectomy as which of the following? a) Partial laryngectomy b) Supraglottic laryngectomy c) Hemilaryngectomy d) Total laryngectomy

A. In a partial laryngectomy, a portion of the larynx is removed, along with one vocal cord and the tumor; all other structures remain. The airway remains intact, and the patient is expected to have no difficulty swallowing. During a supraglottic laryngectomy, a tracheostomy is left in place until the glottic airway is established. Hemilaryngectomy is done by splitting the thyroid cartilage of the larynx in the midline of the neck, and the portion of the vocal cord is removed with the tumor. During a total laryngectomy, a complete removal of the larynx is performed, including the hyoid bone, epiglottis, cricoids cartilage, and two or three rings of the trachea.

You are caring for a client with chronic respiratory failure. What are the signs and symptoms of chronic respiratory failure? a) Progressive loss of lung function associated with chronic disease b) Sudden loss of lung function associated with chronic disease c) Progressive loss of lung function with history of normal lung function d) Sudden loss of lung function with history of normal lung function

A. In chronic respiratory failure, the loss of lung function is progressive, usually irreversible, and associated with chronic lung disease or other disease. This makes options B, C, and D incorrect

When performing endotracheal suctioning, the nurse applies suctioning while withdrawing and gently rotating the catheter 360 degrees for which of the following time periods? a) 10 to 15 seconds b) 20 to 25 seconds c) 0 to 5 seconds d) 30 to 35 seconds

A. In general, the nurse should apply suction no longer than 10 to 15 seconds because hypoxia and dysrhythmias may develop, leading to cardiac arrest. Applying suction for 30 to 35 seconds is hazardous and may result in the patient's developing hypoxia, which can lead to dysrhythmias and, ultimately, cardiac arrest. Applying suction for 20 to 25 seconds is hazardous and may result in the patient's developing hypoxia, which can lead to dysrhythmias and, ultimately, cardiac arrest. Applying suction for 0 to 5 seconds would provide too little time for effective suctioning of secretions.

The nurse is planning the care for a client at risk of developing pulmonary embolism. What nursing interventions should be included in the care plan? Select all that apply. A. Instructing the client to move the legs in a "pumping" exercise B. Applying a sequential compression device C. Using elastic stockings, especially when decreased mobility would promote venous stasis D. Encouraging a liberal fluid intake

A. Instructing the client to move the legs in a "pumping" exercise B. Applying a sequential compression device C. Using elastic stockings, especially when decreased mobility would promote venous stasis D. Encouraging a liberal fluid intake The use of anti-embolism stockings or intermittent pneumatic leg compression devices reduces venous stasis. These measures compress the superficial veins and increase the velocity of blood in the deep veins by redirecting the blood through the deep veins. Having the client move the legs in a "pumping" exercise helps increase venous flow. Legs should not be dangled or feet placed in a dependent position while the client sits on the edge of the bed; instead, feet should rest on the floor or on a chair.

A client appears to be breathing faster than during the last assessment. Which of the following interventions should the nurse perform? a) Count the rate of respirations. b) Inquire if there have been any stressful visitors. c) Assess the radial pulse. d) Assist the client to lie down.

A. Observing the rate and depth of respiration is an important aspect of a nursing assessment. The normal adult resting respiratory rate is 12 to 18 breaths per minute. Tachypnea is rapid breathing with a rate greater than 24 breaths per minute. An increase in the rate of respirations needs further investigation and must be reported.

When assessing a client's potential for pulmonary emboli, what finding by the nurse indicates possible deep vein thrombosis? A. Pain in the calf B. Negative Homan's sign

A. Pain in the calf When assessing the client's potential for pulmonary emboli, the client lies on his or her back and lifts his or her leg and his or her foot. If the client reports calf pain during this maneuver, he or she may have a deep vein thrombosis.

A client seeks care for hoarseness that has lasted for 1 month. To elicit the most appropriate information about this problem, the nurse should ask which question? a) "Do you smoke cigarettes, cigars, or a pipe?" b) "Do you eat a lot of red meat?" c) "Do you eat spicy foods?" d) "Have you strained your voice recently?"

A. Persistent hoarseness may signal throat cancer, which commonly is associated with tobacco use. To assess the client's risk for throat cancer, the nurse should ask about smoking habits. Although straining the voice may cause hoarseness, it wouldn't cause hoarseness lasting for 1 month. Consuming red meat or spicy foods isn't associated with persistent hoarseness

Pink frothy sputum may be an indication of a) pulmonary edema. b) bronchiectasis. c) an infection. d) a lung abscess.

A. Profuse, frothy pink material, often welling up into the throat, may indicate pulmonary edema. Foul-smelling sputum and bad breath may indicate a lung abscess, bronchiectasis, or an infection caused by fusospirochetal or other anaerobic organisms

After diagnosing a client with pulmonary tuberculosis, the physician tells family members that they must receive isoniazid (INH [Laniazid]) as prophylaxis against tuberculosis. The client's daughter asks the nurse how long the drug must be taken. What is the usual duration of prophylactic isoniazid therapy? a) 6 to 12 months b) 2 to 4 months c) 3 to 5 days d) 1 to 3 weeks

A. Prophylactic isoniazid therapy must continue for 6 to 12 months at a daily dosage of 300 mg. Taking the drug for less than 6 months may not provide adequate protection against tuberculosis

A nurse would question the accuracy of a pulse oximetry evaluation in which of the following conditions? a) A client experiencing hypothermia b) A client on a ventilator with PEEP c) A client sitting in a chair after prolonged bed rest d) A client receiving oxygen therapy via Venturi mask

A. Pulse oximetry is a noninvasive method of continuously monitoring the oxygen saturation of hemoglobin. The reading is referred to as SpO2. A probe or sensor is attached to the fingertip, forehead, earlobe, or bridge of the nose. Values less than 85% indicate that the tissues are not receiving enough oxygen. SpO2 values obtained by pulse oximetry are unreliable in states of low perfusion such as hypothermia.

The nurse is caring for a client whose respiratory status has declined since shift report. The client has tachypnea, is restless, and displays cyanosis. Which diagnostic test should be assessed first? a) Pulse oximetry b) Pulmonary function test c) Arterial blood gases d) Chest x-ray

A. Pulse oximetry is a noninvasive method to determine arterial oxygen saturation. Normal values are 95% and above. Using this diagnostic test first provides rapid information of the client's respiratory system. All other options vary in amount of time and patient participation in determining further information regarding the respiratory system.

A client with thrombocytopenia, secondary to leukemia, develops epistaxis. The nurse should instruct the client to: a) sit upright, leaning slightly forward. b) blow his nose and then put lateral pressure on his nose. c) hold his nose while bending forward at the waist. d) lie supine with his neck extended.

A. Sitting upright and leaning slightly forward avoids increasing vascular pressure in the nose and helps the client avoid aspirating blood. Lying supine won't prevent aspiration of the blood. Nose blowing can dislodge any clotting that has occurred. Bending at the waist increases vascular pressure in the nose and promotes bleeding rather than halting it.

A client is on a positive-pressure ventilator with a synchronized intermittent mandatory ventilation (SIMV) setting. The ventilator is set for 8 breaths per minute. The client is taking 6 breaths per minute independently. The nurse a) Continues assessing the client's respiratory status frequently b) Contacts the respiratory therapy department to report the ventilator is malfunctioning c) Consults with the physician about removing the client from the ventilator d) Changes the setting on the ventilator to increase breaths to 14 per minute

A. The SIMV setting on a ventilator allows the client to breathe spontaneously with no assistance from the ventilator for those extra breaths. Data in the stem suggest that the ventilator is working correctly. The nurse would continue making frequent respiratory assessments of the client. There are not sufficient data to suggest the client could be removed from the ventilator. There is no reason to increase the ventilator's setting to 14 breaths per minute or to contact respiratory therapy to report the machine is not working properly

A client has a nursing diagnosis of acute pain related to upper airway irritation. The best short-term goal for this client is for the client to a) Report relief of pain to level 3 using a pain intensity scale of 1 to 10. b) Use a pain intensity rating scale of 0 to 10. c) Gargle with a warm saline solution frequently. d) Take acetaminophen with codeine when pain is 5 or above

A. The client statement of relief of pain to level 3 indicates improvement of the problem. The other options are actually interventions or actions that can help achieve a long-term goal of relief of pain

A victim has sustained a blunt force trauma to the chest. A pulmonary contusion is suspected. Which of the following clinical manifestations correlate with a moderate pulmonary contusion? a) Blood-tinged sputum b) Productive cough c) Bradypnea d) Respiratory alkalosis

A. The clinical manifestations of pulmonary contusions are based on the severity of bruising and parenchymal involvement. The most common signs and symptoms are crackles, decreased or absent bronchial breath sounds, dyspnea, tachypnea, tachycardia, chest pain, blood-tinged secretions, hypoxemia, and respiratory acidosis. Patients with moderate pulmonary contusions often have a constant, but ineffective cough and cannot clear their secretions

In relation to the structure of the larynx, the cricoid cartilage is which of the following? a) The only complete cartilaginous ring in the larynx b) The largest of the cartilage structures c) Used in vocal cord movement with the thyroid cartilage d) The valve flap of cartilage that covers the opening to the larynx during swallowing

A. The cricoid cartilage is the only complete cartilaginous ring in the larynx (located below the thyroid cartilage). The arytenoid cartilages are used in vocal cord movement with the thyroid cartilage. The thyroid cartilage is the largest of the cartilage structures; part of it forms the Adam's apple. The epiglottis is the valve flap of cartilage that covers the opening to the larynx during swallowing.

A patient comes to the clinic complaining of a possible upper respiratory infection. What should the nurse inspect that would indicate that an upper respiratory infection may be present? a) The nasal mucosa b) The buccal mucosa c) The tracheal mucosa d) The frontal sinuses

A. The nurse inspects the nasal mucosa for abnormal findings such as increased redness, swelling, exudate, and nasal polyps, which may develop in chronic rhinitis. The mucosa of the nasal turbinates may also be swollen (boggy) and pale bluish-gray. The nurse palpates the frontal and maxillary sinuses for tenderness, which suggests inflammation, and then inspects the throat by having the patient open the mouth wide and take a deep breath.

You are caring for a client who has been diagnosed with viral pneumonia. You are making a plan of care for this client. What nursing interventions would you put into the plan of care for a client with pneumonia? a) Encourage increased fluid intake. b) Offer nutritious snacks 2 times a day. c) Place client on bed rest. d) Give antibiotics as ordered

A. The nurse places the client in semi-Fowler's position to aid breathing and increase the amount of air taken with each breath. Increased fluid intake is important to encourage because it helps to loosen secretions and replace fluids lost through fever and increased respiratory rate. The nurse monitors fluid intake and output, skin turgor, vital signs, and serum electrolytes. He or she administers antipyretics as indicated and ordered. Antibiotics are not given for viral pneumonia. The client's activity level is ordered by the physician, not decided by the nurse

A nurse is providing discharge teaching for a client who had a laryngectomy. Which instruction should the nurse include in her teaching? a) "Cover the stoma whenever you shower or bathe." b) "Wear a tight cloth at the stoma to prevent anything from entering it." c) "Swimming is good exercise after this surgery." d) "Keep the humidity in your house low."

A. The nurse should instruct the client to gently cover the stoma with a loose plastic bib, or even a hand, when showering or bathing to prevent water from entering the stoma. The client should cover the stoma with a loose-fitting, not tight, cloth to protect it. The client should keep his house humidified to prevent irritation of the stoma that can occur in low humidity. The client should avoid swimming, because it's possible for water to enter the stoma and then enter the client's lung, causing him to drown without submerging his face.

You are an occupational health nurse in a large ceramic manufacturing company. How would you intervene to prevent occupational lung disease in the employees of the company? a) Fit all employees with protective masks. b) Give workshops on disease prevention. c) Insist on adequate breaks for each employee. d) Provide employees with smoking cessation materials.

A. The primary focus is prevention, with frequent examination of those who work in areas of highly concentrated dust or gases. Laws require work areas to be safe in terms of dust control, ventilation, protective masks, hoods, industrial respirators, and other protection. Workers are encouraged to practice healthy behaviors, such as quitting smoking. Adequate breaks, giving workshops, and providing smoking cessation materials do not prevent occupational lung diseases.

A client comes into the Emergency Department with epistaxis. What intervention should you perform when caring for a client with epistaxis? a) Apply direct continuous pressure. b) Apply a moustache dressing. c) Provide a nasal splint. d) Place the client in a semi-Fowler's position.

A. The severity and location of bleeding determine the treatment of a client with epistaxis. To manage this condition, the nurse should apply direct continuous pressure to the nares for 5 to 10 minutes with the client's head tilted slightly forward. Application of a moustache dressing or a drip pad to absorb drainage, application of a nasal splint, and placement of the client in a semi-Fowler's position are interventions related to the management of a client with a nasal obstruction.

A young adult visited a clinic because he was injured during a softball game. He told the nurse that the ball struck him in his "Adam's apple." To assess the initial impact of injury, the nurse: a) Palpates the thyroid cartilage. b) Inspects the epiglottis. c) Inspects the vocal cords. d) Palpates the cricoid cartilage.

A. The term "Adam's Apple" is used to refer to a lump or protrusion, a laryngeal prominence. It is formed by the angle of the thyroid cartilage surrounding the larynx

Knowing respiratory physiology is important to understand how the disease process can work within that system. Which hollow tube transports air from the laryngeal pharynx to the bronchi? a) Trachea b) Larynx c) Bronchioles d) Pharynx

A. The trachea is a hollow tube composed of smooth muscle and supported by C-shaped cartilage. The trachea transports air from the laryngeal pharynx to the bronchi and lungs. This is a cartilaginous framework between the pharynx and trachea that produces sound. The bronchioles are smaller subdivisions of bronchi within the lungs. The pharynx, or throat, carries air from the nose to the larynx and food from the mouth to the esophagus.

A nurse practitioner diagnosed a patient with an infection in the maxillary sinuses. Select the area that the nurse palpated to make that diagnosis. a) On the cheeks below the eyes b) Above the eyebrows c) Between the eyes and behind the nose d) Behind the ethmoid sinuses

A. To palpate the maxillary sinuses, the nurse should apply gentle pressure in the cheek area below the eyes, adjacent to the nose.

You are caring for a 65-year-old client who has been newly diagnosed with emphysema. The client is confused by the new terms and wants to know what ventilation means. Which of the following can instruct this client? a) Ventilation is breathing air in and out of the lungs. b) Ventilation helps clients who cannot breathe on their own. c) Ventilation provides a blood supply to the lungs. d) Ventilation is when the body changes oxygen into CO2.

A. Ventilation is the actual movement of air in and out of the respiratory tract. Diffusion is the exchange of oxygen and CO2 through the alveolar-capillary membrane. Pulmonary perfusion refers to blood supply to the lungs. A mechanical ventilator assists patients who are unable to breathe on their own.

A new ICU nurse is observed by her preceptor entering a patient's room to suction the tracheostomy after performing the task 15 minutes before. What should the preceptor educate the new nurse to do to ensure that the patient needs to be suctioned? a) Auscultate the lung for adventitious sounds. b) Have the patient inform the nurse of the need to be suctioned. c) Have the patient cough. d) Assess the CO2 level to determine if the patient requires suctioning.

A. When a tracheostomy or endotracheal tube is in place, it is usually necessary to suction the patient's secretions because of the decreased effectiveness of the cough mechanism. Tracheal suctioning is performed when adventitious breath sounds are detected or whenever secretions are obviously present. Unnecessary suctioning can initiate bronchospasm and cause mechanical trauma to the tracheal mucosa.

The client is returning from the operating room following a broncho scopy. Which action, performed by the nursing assistant, would the nurse stop if began prior to nursing assessment? a) The nursing assistant is pouring a glass of water to wet the client's mouth. b) The nursing assistant is assisting the client to a semi-Fowler's position. c) The nursing assistant is assisting the client to the side of the bed to use a urinal. d) The nursing assistant is asking a question requiring a verbal response.

A. When completing a procedure which sends a scope down the throat, the gag reflex is anesthetized to reduce discomfort. Upon returning to the nursing unit, the gag reflex must be assessed before providing any food or fluids to the client. The client may need assistance following the procedure for activity and ambulation but this is not restricted in the post procedure period

The nurse is caring for a patient diagnosed with pneumonia. The nurse will assess the patient for tactile fremitus by completing which of the following? a) Asking the patient to repeat "ninety-nine" as the nurse's hands move down the patient's thorax b) Asking the patient to say "one, two, three" while auscultating the lungs c) Instructing the patient to take a deep breath and hold it while the diaphragm is percussed d) Placing the thumbs along the costal margin of the chest wall and instructing the patient to inhale deeply

A. While the nurse is assessing for tactile fremitus, the patient is asked to repeat "ninety-nine" or "one, two, three," or "eee, eee, eee" as the nurse's hands move down the patient's thorax. The vibrations are detected with the palmar surfaces of the fingers and hands, or the ulnar aspect of the extended hands, on the thorax. The hand or hands are moved in sequence down the thorax. Corresponding areas of the thorax are compared. Asking the patient to say "one, two, three" while auscultating the lungs is not the proper technique for assessing for tactile fremitus. The nurse assesses for anterior respiratory excursion by placing the thumbs along the costal margin of the chest wall and instructing the patient to inhale deeply. The nurse assesses for diaphragmatic excursion by instructing the patient to take a deep breath and hold it while the diaphragm is percussed

The nurse is instructing the patient with asthma in the use of a newly prescribed leukotriene receptor antagonist. What should the nurse be sure to include in the education? a) The patient should take the medication an hour before meals or 2 hours after a meal. b) The patient should take the medication with meals since it may cause nausea. c) The patient should take the medication with a small amount of liquid. d) The patient should take the medication separately without other medications.

A. he nurse should instruct the patient to take the leukotriene receptor antagonist at least 1 hour before meals or 2 hours after meals.

The nurse is an occupational health nurse who is presenting a workshop on laryngeal cancer. What risk factors would the nurse be sure to include in the workshop? Select all that apply. A. industrial pollutants B. alcohol C. tobacco

A. industrial pollutants B. alcohol C. tobacco Carcinogens, such as tobacco, alcohol, and industrial pollutants, are associated with laryngeal cancer.

A client involved in a motor vehicle crash suffered a blunt injury to the chest wall and was brought to the emergency department. The nurse assesses the client for which clinical manifestation that would indicate the presence of a pneumothorax? A. sucking sound at the site of injury B. Diminished breath sounds

A. sucking sound at the site of injury Open pneumothorax is one form of traumatic pneumothorax. It occurs when a wound in the chest wall is large enough to allow air to pass freely in and out of the thoracic cavity with each attempted respiration. Because the rush of air through the wound in the chest wall produces a sucking sound, such injuries are termed "sucking chest wounds."

A nurse working in the radiology clinic is assisting with a client after an unusual arterial procedure. What assessment should the nurse notify the health care provider about? Absent distal pulses Raised temperature in the affected limb Excessive capillary refill Flushed feeling in the client

Absent distal pulses When monitoring clients after a pulmonary angiography, nurses must notify the health care provider about diminished or absent distal pulses, cool skin temperature in the affected limb, and poor capillary refill. When the contrast medium is infused, the client will sense a warm, flushed feeling.

A nurse working in the radiology clinic is assisting with a client after an unusual arterial procedure. What assessment should the nurse notify the health care provider about?

Absent distal pulses Explanation: When monitoring clients after a pulmonary angiography, nurses must notify the health care provider about diminished or absent distal pulses, cool skin temperature in the affected limb, and poor capillary refill. When the contrast medium is infused, the client will sense a warm, flushed feeling.

A client is being treated for a pulmonary embolism and the medical nurse is aware that the client suffered an acute disturbance in pulmonary perfusion. This involved an alteration in what aspect of normal physiology?

Adequate flow of blood through the pulmonary circulation. Explanation: Pulmonary perfusion is the actual blood flow through the pulmonary circulation. Perfusion is not defined in terms of pH balance, muscle tone, or osmotic pressure.

Which is a true statement regarding air pressure variances?

Air is drawn through the trachea and bronchi into the alveoli during inspiration. Explanation: Air flows from a region of higher pressure to a region of lower pressure. During inspiration, movement of the diaphragm and other muscles of respiration enlarge the thoracic cavity, thereby lowering the pressure inside the thorax to a level below that of atmospheric pressure.

Constant bubbling in the water seal of a chest drainage system indicates which problem?

Air leak The nurse needs to observe for air leaks in the drainage system; they are indicated by constant bubbling in the water seal chamber, or by the air leak indicator in dry systems with a one-way valve. Tidaling is fluctuation of the water level in the water seal that shows effective connection between the pleural cavity and the drainage chamber and indicates that the drainage system remains patent.

A patient visited a health care clinic for treatment of upper respiratory tract congestion, fatigue, and sputum production that was rust-colored. Which of the following diagnoses is likely based on this history and inspection of the sputum?

An infection with pneumococcal pneumonia Explanation: Sputum that is rust colored suggests infection with pneumococcal pneumonia. Bronchiectasis and a lung abscess usually are associated with purulent thick and yellow-green sputum. Bronchitis usually yields a small amount of purulent sputum.

The nurse is interviewing a PT who reports a dry, irritating cough that is not "bringing anything up". What medication should the nurse question the patient about talking?

Angiotensin converting enzyme (ACE)

The nurse is interviewing a patient who says he has a dry, irritating cough that is not "bringing anything up." What medication should the nurse question the patient about taking?

Angiotensin converting enzyme (ACE) inhibitors Explanation: Common causes of cough include asthma, gastrointestinal reflux disease, infection, aspiration, and side effects of medications, such as angiotensin converting enzyme (ACE) inhibitors. The other medications listed are not associated with causing a cough

The nurse auscultated a patient's middle lobe of the lungs for abnormal breath sounds. To do this, the nurse placed the stethoscope on the:

Anterior surface of the right side of the chest, between the fourth and fifth rib. Explanation: The middle lobe of the lung is only found on the right side of the thorax and can only be assessed anteriorly. It is located at the fourth rib, at the right sternal border and extends to the fifth rib, in the midaxillary line.

The nurse is caring for clients on the neurological unit. Which triad of neurological mechanisms does the nurse identify as most responsible when there is abnormality in ventilation control?

Aortic arch, pons, and CO2 receptor sites Explanation: Several mechanisms control ventilation. The respiratory center in the medulla oblongata and pons control rate and depth of respirations. The central chemo receptors in the medulla and peripheral chemo receptors in the aortic arch also provide a mechanism for detecting abnormalities and signal changes to alter the pH and levels of oxygen in the blood. The other options have an incorrect piece of the triad.

The two centers in the brain that are responsible for the neurologic control of ventilation are?

Apneustic center & Pneumotaxic Center

A client on long-term mechanical ventilation becomes very frustrated when he tries to communicate. Which intervention should the nurse perform to assist the client?

Ask the client to write, use a picture board, or spell words with an alphabet board. If the client uses an alternative method of communication, such as writing, using a picture board, or spelling words on an alphabet board, he'll feel more in control and be less frustrated. Assuring the client that everything will be all right offers false reassurance and telling him not to be upset minimizes his feelings. Neither of these methods helps the client to communicate. Family members are also likely to encounter difficulty interpreting the wishes of a client with an endotracheal tube or tracheostomy tube. Making them responsible for interpreting the client's gestures may frustrate them. The client may be weaned off a mechanical ventilator only when the physiologic parameters for weaning have been met.

The nurse is caring for a client diagnosed with pneumonia. The nurse assesses the client for tactile fremitus by completing which action?

Asking the client to repeat "ninety-nine" as the nurse's hands move down the client's thorax While the nurse is assessing for tactile fremitus, the client is asked to repeat "ninety-nine" or "one, two, three," or "eee, eee, eee" as the nurse's hands move down the client's thorax. Vibrations are detected with the palmar surfaces of the fingers and hands, or the ulnar aspect of the extended hands, on the thorax. The hand(s) are moved in sequence down the thorax, and corresponding areas of the thorax are compared. Asking the client to say "one, two, three" while auscultating the lungs is not the proper technique to assess for tactile fremitus. The nurse assesses for anterior respiratory excursion by placing the thumbs along the costal margin of the chest wall and instructing the client to inhale deeply. The nurse assesses for diaphragmatic excursion by instructing the client to take a deep breath and hold it while the diaphragm is percussed.

The nurse is caring for a client diagnosed with pneumonia. The nurse assesses the client for tactile fremitus by completing which action? Placing the thumbs along the costal margin of the chest wall and instructing the client to inhale deeply Asking the client to say "one, two, three" while the nurse auscultates the lungs Instructing the client to take a deep breath and hold it while the diaphragm is percussed Asking the client to repeat "ninety-nine" as the nurse's hands move down the client's thorax

Asking the client to repeat "ninety-nine" as the nurse's hands move down the client's thorax While the nurse is assessing for tactile fremitus, the client is asked to repeat "ninety-nine" or "one, two, three," or "eee, eee, eee" as the nurse's hands move down the client's thorax. Vibrations are detected with the palmar surfaces of the fingers and hands, or the ulnar aspect of the extended hands, on the thorax. The hand(s) are moved in sequence down the thorax, and corresponding areas of the thorax are compared. Asking the client to say "one, two, three" while auscultating the lungs is not the proper technique to assess for tactile fremitus. The nurse assesses for anterior respiratory excursion by placing the thumbs along the costal margin of the chest wall and instructing the client to inhale deeply. The nurse assesses for diaphragmatic excursion by instructing the client to take a deep breath and hold it while the diaphragm is percussed.

The nurse is caring for a client diagnosed with pneumonia. The nurse assesses the client for tactile fremitus by completing which action?

Asking the client to repeat "ninety-nine" as the nurse's hands move down the client's thorax Explanation: While the nurse is assessing for tactile fremitus, the client is asked to repeat "ninety-nine" or "one, two, three," or "eee, eee, eee" as the nurse's hands move down the client's thorax. Vibrations are detected with the palmar surfaces of the fingers and hands, or the ulnar aspect of the extended hands, on the thorax. The hand(s) are moved in sequence down the thorax, and corresponding areas of the thorax are compared. Asking the client to say "one, two, three" while auscultating the lungs is not the proper technique to assess for tactile fremitus. The nurse assesses for anterior respiratory excursion by placing the thumbs along the costal margin of the chest wall and instructing the client to inhale deeply. The nurse assesses for diaphragmatic excursion by instructing the client to take a deep breath and hold it while the diaphragm is percussed.

The nurse received a client from the post-anesthesia care unit (PACU) who has a chest tube to a closed drainage system. Report from the PACU nurse included drainage in the chest tube at 80 mL of bloody fluid. Fifteen minutes after transfer from the PACU, the chest tube indicates drainage as pictured. The client is reporting pain at "8" on a scale of 0 to 10. The first action of the nurse is to:

Assess pulse and blood pressure. Explanation: The client has bled 120 mL of bloody drainage in the chest drainage system within 15 minutes. It is most important for the nurse to assess for signs and symptoms of hemorrhage, which may be indicated by a rapid pulse and decreasing blood pressure. The nurse may then lay the client in a flat position and notify the physician.

The nurse is caring for a client with recurrent hemoptysis who has undergone a bronchoscopy. Immediately following the procedure, the nurse should complete which action?

Assess the client for a cough reflex. After the procedure, the client must take nothing by mouth until the cough reflex returns, because the preoperative sedation and local anesthesia impair the protective laryngeal reflex and swallowing. Once the client demonstrates a cough reflex, the nurse may offer ice chips and eventually fluids. The client is sedated during the procedure, not afterward. The client is not required to maintain bed rest following the procedure.

The nurse is caring for a client with recurrent hemoptysis who has undergone a bronchoscopy. Immediately following the procedure, the nurse should complete which action? Ensure the client remains moderately sedated to decrease anxiety. Assess the client for a cough reflex. Offer the client ice chips. Instruct the client that bed rest must be maintained for 2 hours.

Assess the client for a cough reflex. After the procedure, the client must take nothing by mouth until the cough reflex returns, because the preoperative sedation and local anesthesia impair the protective laryngeal reflex and swallowing. Once the client demonstrates a cough reflex, the nurse may offer ice chips and eventually fluids. The client is sedated during the procedure, not afterward. The client is not required to maintain bed rest following the procedure.

The nurse is preparing to suction a client with an endotracheal tube. What should be the nurse's first step in the suctioning process?

Assess the client's lung sounds and SaO2 via pulse oximeter. Explanation: Assessment data indicate the need for suctioning and allow the nurse to monitor the effect of suction on the client's level of oxygenation. Explaining the procedure, performing hand hygiene, and turning on the suction source are interventions that should follow assessment. As with all interventions, assessment should be performed first.

A client's plan of care specifies postural drainage. What action should the nurse perform when providing this noninvasive therapy?

Assist the client into a position that will allow gravity to move secretions. Postural drainage is usually performed two to four times per day. The client uses gravity to facilitate postural draining. The skin should be covered with a cloth or a towel during percussion to protect the skin. Postural drainage is not given in an upright position or directly following a meal.

The ED nurse is assessing the respiratory function of a teenage girl who presented with acute shortness of breath. Auscultation reveals continuous wheezes during inspiration and expiration. This finding is most suggestive of what?

Asthma Sibilant wheezes are commonly associated with asthma. They do not normally accompany pleurisy, emphysema, or pneumonia.

The nurse assessed a 28-year-old woman who was experiencing dyspnea severe enough to make her seek medical attention. The history revealed no prior cardiac problems and the presence of symptoms for 6 months' duration. On assessment, the nurse noted the presence of both inspiratory and expiratory wheezing. Based on this data, which of the following diagnoses is likely?

Asthma Explanation: The presence of both inspiratory and expiratory wheezing usually signifies asthma if the individual does not have heart failure. Sudden dyspnea is an indicator of the other choices.

A new ICU nurse is observed by her preceptor entering a patient's room to suction the tracheostomy after performing the task 15 minutes before. What should the preceptor educate the new nurse to do to ensure that the patient needs to be suctioned?

Auscultate the lung for adventitious sounds. When a tracheostomy or endotracheal tube is in place, it is usually necessary to suction the patient's secretions because of the decreased effectiveness of the cough mechanism. Tracheal suctioning is performed when adventitious breath sounds are detected or whenever secretions are obviously present. Unnecessary suctioning can initiate bronchospasm and cause mechanical trauma to the tracheal mucosa.

A client is being seen in the pediatric clinic for a middle ear infection. The client's mother reports that when the client develops an upper respiratory infection, an ear infection seems quick to follow. What contributes to this event? A. oropharynx B. eustachian tubes C. genetics D. epiglottis

B

A client will be undergoing a bronchoscopy. Which statement shows that the client understands the procedure? A. "If I cough up bloody mucus the next day, I need to call 911." B. "It sounds like the different drugs will make this much easier to stand." C. "At least I can eat normally again as soon as this is over." D. If the tube goes through my nose, will my throat hurt?"

B

What is the primary function of the larynx? A. preventing infection B. producing sound C. faciliating coughing D. protecing the lower airway from foreign objects

B

Which hollow tubes transports air from the laryngeal pharynx to the bronchi? A. larynx B. trachea C. pharynx D. bronchioles

B

The nurse is to make a room assignment for a client diagnosed with an upper respiratory infection. The other clients with empty beds in the room are listed in the accompanying chart. The best room assignment for the new client would be with Client

B The nurse needs to make the appropriate room assignment based on the client's problems, safety, and risk for infection to others. The client with an upper respiratory infection may transmit infection to susceptible people. Clients A, C, and D have increased susceptibility for infection because of immunosuppression or surgery.

What are the primary functions of the lungs? A. destroying CO B. ventilation C. gas exchange D. oxygen production

B, C

The nurse should convey which information to a client prior to a pulmonary function test? A. the client will be reclining during the test B. the client should not eat 2 hours prior to the test C. the client may be tired after the test D. the client should wear loose-fitting clothing E. the client will receive contrast medium via catheter

B, C, D

The nurse is working on a busy respiratory unit. In caring for a variety of clients, the nurse must be knowledgeable of diagnostic studies. With which diagnostic studies would the nurse screen the client for an allergy to iodine? Select all that apply. a) Chest x-ray b) Lung scan c) Pulmonary angiography d) Fluoroscopy e) Pulmonary functions test f) Bronchoscopy

B, C, D. The nurse must be well educated in screening clients before diagnostic procedures which include contrast medium for an allergy to iodine. A lung scan, fluoroscopy and pulmonary angiography all require contrast medium

Which of the following are indicators that a client is ready to be weaned from a ventilator? Select all that apply. a) FiO2 45% b) Tidal volume of 8.5 mL/kg c) PaO2 of 64 mm Hg d) Vital capacity of 13 mL/kg e) Rapid/shallow breathing index of 112 breaths/min

B, C, D. Weaning criteria for clients are as follows: Vital capacity 10 to 15 mL/kg; Maximum inspiratory pressure at least -20 cm H2; Tidal volume: 7 to 9 mL/kg; Minute ventilation: 6 L/min; Rapid/shallow breathing index below 100 breaths/min; PaO2 > 60 mm Hg; FiO2 < 40%

The nurse is educating the patient in the use of a mini-nebulizer. What should the nurse encourage the patient to do? (Select all that apply.) a) Take rapid, deep breaths. b) Cough frequently. c) Frequently evaluate progress. d) Hold the breath at the end of inspiration for a few seconds. e) Prolong the expiratory phase after using the nebulizer.

B, C, D. The nurse instructs the patient to breathe through the mouth, taking slow, deep breaths, and then to hold the breath for a few seconds at the end of inspiration to increase intrapleural pressure and reopen collapsed alveoli, thereby increasing functional residual capacity. The nurse encourages the patient to cough and to monitor the effectiveness of the therapy. The nurse instructs the patient and family about the purpose of the treatment, equipment setup, medication additive, and proper cleaning and storage of the equipment

The nurse is caring for a geriatric client brought to the emergency department after being found by her children feeling poorly with an elevated temperature. Laboratory tests confirm influenza type A, a respiratory virus. Which medical treatment would the nurse anticipate in the discharge instructions? Select all that apply. a) Antibiotics b) Saline gargles c) Increased fluids d) Antiemetics e) Antitussives f) Rest

B, C, E, F. Influenza type A is the most common cause of the flu initiated by a respiratory virus. Common discharge instructions include rest, increased fluids to thin respiratory secretions, saline gargles to help prevent a throat infection such a strep throat, and antitussives if the client is coughing. Antibiotics are not used with a virus unless a bacterial infection subsequently develops. Antiemetics are used for nausea and vomiting not commonly associated with a common respiratory virus

A young client who aspirated a piece of candy has been brought to the ED. Where would be the most likely places to find an aspirated piece of candy? Select all that apply. A. left upper lung B. right upper lung C. left mainstem bronchus D. right mainstem bronchus

B, D

The nurse is caring for a client with a new tracheostomy. Which of the following nursing diagnoses are priorities? Select all that apply. a) Risk for Infection related to operative incision and tracheostomy tube placement b) Ineffective Airway Clearance related to increased secretions c) Knowledge Deficit related to care of the tracheostomy tube and surrounding site d) Impaired Gas Exchange related to shallow breathing and anxiousness

B, D. The client with a new tracheostomy tube has increased secretions, which may become dried and occlude the airway or plug the airway requiring frequent suctioning. Impaired Gas Exchange is an equally important diagnosis. These are related to airway and breathing and are priorities.

A 72-year-old male client finished a course of antibiotics for laryngitis but continues to experience persistent hoarseness. If laryngeal cancer is suspected, the nurse would be most likely to hear which of the following complaints from the client? a) Weight loss b) A feeling of swelling at the back of the throat c) Discomfort when drinking cold liquids d) Headaches in the morning

B. After an initial hoarseness lasting longer than a month, clients with laryngeal cancer will feel a sensation of swelling or a lump in the throat or in the neck. Weight loss often occurs later in the progression of laryngeal cancer due to reduced calorie intake as a result of impaired swallowing and pain. Clients with laryngeal cancer may complain of burning in the throat when swallowing hot or citrus liquids. Clients with obstructive sleep apnea may experience a morning headache

A nurse is concerned that a client may develop postoperative atelectasis. Which nursing diagnosis would be most appropriate if this complication occurs? a) Impaired spontaneous ventilation b) Impaired gas exchange c) Ineffective airway clearance d) Decreased cardiac output

B. Airflow is decreased with atelectasis, which is a bronchial obstruction from collapsed lung tissue. If there is an obstruction, there is limited or no gas exchange in this area. Impaired gas exchange is thus the most likely nursing diagnosis with atelectasis.

A client is being seen in the emergency department for exacerbation of chronic obstructive pulmonary disease (COPD). The first action of the nurse is to administer which of the following prescribed treatments? a) Vancomycin 1 gram intravenously over 1 hour b) Oxygen through nasal cannula at 2 L/minute c) Intravenous methylprednisolone (Solu-Medrol) 120 mg d) Ipratropium bromide (Alupent) by metered-dose inhaler

B. All options listed are treatments that may be used for a client with an exacerbation of COPD. The first line of treatment is oxygen therapy.

A client with chronic obstructive pulmonary disease (COPD) expresses a desire to quit smoking. The first appropriate response from the nurse is: a) "I can refer you to the American Lung Association." b) "Have you tried to quit smoking before?" c) "Many options are available for you." d) "Nicotine patches would be appropriate for you."

B. All the options are appropriate statements; however, the nurse needs to assess the client's statement further. Assessment data include information about previous attempts to quit smoking.

The nurse suctions a patient through the endotracheal tube for 20 seconds and observes dysrhythmias on the monitor. What does the nurse determine is occurring with the patient? a) The patient is in a hypermetabolic state. b) The patient is hypoxic from suctioning. c) The patient is having a myocardial infarction. d) The patient is having a stress reaction.

B. Apply suction while withdrawing and gently rotating the catheter 360 degrees (no longer than 10-15 seconds). Prolonged suctioning may result in hypoxia and dysrhythmias, leading to cardiac arrest.

As status asthmaticus worsens, the nurse would expect which acid-base imbalance? a) Metabolic alkalosis b) Respiratory acidosis c) Respiratory alkalosis d) Metabolic acidosis

B. As status asthmaticus worsens, the PaCO increases and the pH decreases, reflecting respiratory acidosis.

Which of the following would not be considered a primary symptom of COPD? a) Dyspnea on exertion b) Weight gain c) Cough d) Sputum production

B. COPD is characterized by three primary symptoms: cough, sputum production, and dyspnea on exertion. Weight loss is common with COPD.

A patient is diagnosed with mild obstructive sleep apnea after having a sleep study performed. What treatment modality will be the most effective for this patient? a) Surgery to remove the tonsils and adenoids b) Continuous positive airway pressure c) Medications to assist the patient with sleep at night d) Bi-level positive airway pressure

B. Continuous positive airway pressure (CPAP) provides positive pressure to the airways throughout the respiratory cycle. Although it can be used as an adjunct to mechanical ventilation with a cuffed endotracheal tube or tracheostomy tube to open the alveoli, it is also used with a leak-proof mask to keep alveoli open, thereby preventing respiratory failure. CPAP is the most effective treatment for obstructive sleep apnea because the positive pressure acts as a splint, keeping the upper airway and trachea open during sleep. To use CPAP, the patient must be breathing independently

The nurse is providing discharge instructions to a patient with pulmonary sarcoidosis. The nurse concludes that the patient understands the information if the patient correctly states which of the following early signs of exacerbation? a) Fever b) Shortness of breath c) Headache d) Weight loss

B. Early signs and symptoms of pulmonary sarcoidosis may include dyspnea, cough, hemoptysis, and congestion. Generalized symptoms include anorexia, fatigue, and weight loss

For a client with an endotracheal (ET) tube, which nursing action is the most important? a) Turning the client from side to side every 2 hours b) Auscultating the lungs for bilateral breath sounds c) Providing frequent oral hygiene d) Monitoring serial blood gas values every 4 hours

B. For the client with an ET tube, the most important nursing action is auscultating the lungs regularly for bilateral breath sounds to ensure proper tube placement and effective oxygen delivery. Although turning the client from side to side every 2 hours, monitoring serial blood gas values every 4 hours, and providing frequent oral hygiene are appropriate actions for this client, they're secondary to ensuring adequate oxygenation.

The nurse is caring for a patient who is to undergo a thoracentesis. In preparation for the procedure, the nurse will position the patient in which of the following positions? a) Lateral recumbent b) Sitting on the edge of the bed c) Prone d) Supine

B. If possible, it is best to place the patient upright or sitting on the edge of the bed with the feet supported and arms and head on a padded over-the-bed table. Other positions in which the patient could be placed include straddling a chair with arms and head resting on the back of the chair, or lying on the unaffected side with the head of the bed elevated 30 to 45 degrees if unable to assume a sitting position.

The nurse should be alert for a complication of bronchiectasis that results from a combination of retained secretions and obstruction and that leads to the collapse of alveoli. What complication should the nurse monitor for? a) Pleurisy b) Atelectasis c) Pneumonia d) Emphysema

B. In bronchiectasis, the retention of secretions and subsequent obstruction ultimately cause the alveoli distal to the obstruction to collapse (atelectasis).

Another term for clergyman's sore throat is a) aphonia. b) chronic granular pharyngitis. c) atrophic pharyngitis. d) hypertrophic pharyngitis.

B. In clergyman's sore throat, the pharynx is characterized by numerous swollen lymph follicles. Aphonia refers to the inability to use one's voice. Atrophic pharyngitis is characterized by a membrane that is thin, white, glistening, and at times wrinkled. Hypertrophic pharyngitis is characterized by general thickening and congestion of the pharyngeal mucous membrane

A client with acute viral rhinosinusitis is being seen in a clinic. The nurse is providing discharge instructions and includes the following information: a) Dehumidify the air in the home. b) Avoid air travel. c) Severe pain when palpating the sinuses is normal. d) Make sure to follow antibiotic therapy.

B. Information that the nurse should include for a client with acute viral rhinosinusitis is to avoid air travel. Other nursing interventions include referring the client to a physician if severe pain occurs when palpating the sinuses and humidifying the air in the home to promote drainage. Antibiotic therapy is not indicated for a viral infection

A physician has ordered that a client with suspected lung cancer undergo magnetic resonance imaging (MRI). The nurse explains the benefits of this study to the client. Included in teaching would be which of the following regarding the MRI? a) Lung blood flow can be viewed after a radiopaque agent is injected. b) MRI can view soft tissues and can help stage cancers. c) Narrow-beam x-ray can scan successive lung layers. d) Tumor densities can be seen with radiolucent images.

B. MRI uses magnetic fields and radiofrequency signals to produce a detailed diagnostic image. MRI can visualize soft tissues, characterize nodules, and help stage carcinomas. The other options describe different studies

Which of the following terms will the nurse use to document the inability of a patient to breathe easily unless positioned upright? a) Hemoptysis b) Orthopnea c) Hypoxemia d) Dyspnea

B. Orthopnea is the term used to describe a patient's inability to breathe easily except in an upright position. Orthopnea may be found in patients with heart disease and, occasionally, in patients with COPD. Patients with orthopnea are placed in a high Fowler's position to facilitate breathing. Dyspnea refers to labored breathing or shortness of breath. Hemoptysis refers to expectoration of blood from the respiratory tract. Hypoxemia refers to low oxygen levels in the blood.

Nursing instruction during postural drainage should include telling the patient to: a) Remain in each position for 30 to 45 minutes for best results. b) Change positions frequently and cough up secretions. c) Lie supine to rest the lungs. d) Sit upright to promote ventilation

B. Patients who lie supine will have secretions accumulate in the posterior lung sections, whereas upright patients will pool secretions in their lower lobes. By changing positions (see Fig. 10-2 in the text), secretions can drain from the affected bronchioles into the bronchi and trachea and then be removed by coughing or suctioning

A client suspected of developing acute respiratory distress syndrome (ARDS) is experiencing anxiety and agitation due to increasing hypoxemia and dyspnea. A nurse would implement which intervention to improve oxygenation and provide comfort for the client? A. Assist the client into a chair B. Position the client in the prone position

B. Position the client in the prone position The patient is extremely anxious and agitated because of the increasing hypoxemia and dyspnea. It is important to reduce the patient's anxiety because anxiety increases oxygen expenditure. Oxygenation in patients with ARDS is sometimes improved in the prone position. Rest is essential to limit oxygen consumption and reduce oxygen needs.

The nurse is assisting a client with postural drainage. Which of the following demonstrates correct implementation of this technique? a) Administer bronchodilators and mucolytic agents following the sequence. b) Instruct the client to remain in each position of the postural drainage sequence for 10 to 15 minutes. c) Perform this measure with the client once a day. d) Use aerosol sprays to deodorize the client's environment after postural drainage.

B. Postural drainage is usually performed two to four times daily, before meals (to prevent nausea, vomiting, and aspiration) and at bedtime. Prescribed bronchodilators, water, or saline may be nebulized and inhaled before postural drainage to dilate the bronchioles, reduce bronchospasm, decrease the thickness of mucus and sputum, and combat edema of the bronchial walls. The nurse instructs the client to remain in each position for 10 to 15 minutes and to breathe in slowly through the nose and out slowly through pursed lips to help keep the airways open so that secretions can drain while in each position. If the sputum is foul-smelling, it is important to perform postural drainage in a room away from other patients or family members. (Deodorizers may be used to counteract the odor. Because aerosol sprays can cause bronchospasm and irritation, they should be used sparingly and with caution.)

A client has acute bacterial rhinosinusitis for several weeks despite treatment. The nurse observes for a possible complication of the infection by assessing for a) Nausea b) Nuchal rigidity c) Hypertension d) Mild headache

B. Potential complications of acute bacterial rhinosinusitis are nuchal rigidity and severe headache. Hypertension may be a result of over-the-counter decongestant medications. Nausea may be a result of nasal corticosteroids

Positive end-expiratory pressure (PEEP) therapy has which effect on the heart? A. Bradycardia B. Reduced cardiac output

B. Reduced cardiac output Explanation: PEEP reduces cardiac output by increasing intrathoracic pressure and reducing the amount of blood delivered to the left side of the heart. It doesn't affect heart rate, but a decrease in cardiac output may reduce blood pressure, commonly causing compensatory tachycardia, not bradycardia. However, the resulting tachycardia isn't a direct ef

The nurse is caring for a client who is ready to be weaned from the ventilator. In preparing to assist in the collaborative process of weaning, the nurse should anticipate that the weaning of the client will progress in what order? A. Removal from oxygen, ventilator, and then tube B. Removal from the ventilator, tube, and then oxygen

B. Removal from the ventilator, tube, and then oxygen Explanation: The process of withdrawing the client from dependence on the ventilator takes place in three stages: the client is gradually removed from the ventilator, then from the tube, and, finally, oxygen.

During assessment of a patient with OSA, the nurse documents which of the following characteristic signs that occurs because of repetitive apneic events? a) Increased smooth muscle contractility b) Hypercapnia c) Systemic hypotension d) Pulmonary hypotension

B. Repetitive apneic events result in hypoxia and hypercapnia, which triggers a sympathetic response (increased heart rate and decreased tone and contractility of smooth muscle)

A nurse is reviewing arterial blood gas results on an assigned client. The pH is 7.32 with PCO2 of 49 mm Hg and a HCO3−of 28 mEq/L. The nurse reports to the physician which finding? a) Respiratory alkalosis b) Respiratory acidosis c) Metabolic alkalosis d) Metabolic acidosis

B. Respiratory acidosis would be reported to the physician citing the lab values. Analysis of the blood gases reveals that the client is acidotic with a pH under 7.35. Also noted is the PCO2above the normal range of 30 to 40 mm Hg. The HCO3− is slightly elevated because the normal level is 22 to 26 mEq/L

A client with chronic bronchitis is admitted to the health facility. Auscultation of the lungs reveals low-pitched, rumbling sounds. Which of the following describes these sounds? a) Rales b) Rhonchi c) Venous hum d) Bronchovesicular

B. Rhonchi are deep, low-pitched, rumbling sounds heard usually on expiration. The etiology of rhonchi is associated with chronic bronchitis.

The nurse is caring for a patient with an endotracheal tube (ET). Which of the following nursing interventions is contraindicated? a) Checking the cuff pressure every 6 to 8 hours b) Deflating the cuff routinely c) Deflating the cuff prior to tube removal d) Ensuring that humidified oxygen is always introduced through the tube

B. Routine cuff deflation is not recommended because of the increased risk for aspiration and hypoxia. The cuff is deflated before the ET is removed. Cuff pressures should be checked every 6 to 8 hours. Humidified oxygen should always be introduced through the tube

A client with suspected severe acute respiratory syndrome (SARS) comes to the emergency department. Which physician order should the nurse implement first? a) Obtain a nasopharyngeal specimen for reverse-transcription polymerase chain reaction testing. b) Institute isolation precautions. c) Obtain a sputum specimen for enzyme immunoassay testing. d) Begin an I.V. infusion of dextrose 5% in half-normal saline solution at 100 ml/hour.

B. SARS, a highly contagious viral respiratory illness, is spread by close person-to-person contact. Contained in airborne respiratory droplets, the virus is easily transmitted by touching surfaces and objects contaminated with infectious droplets. The nurse should give top priority to instituting infection-control measures to prevent the spread of infection to emergency department staff and clients. After isolation measures are carried out, the nurse can begin an I.V. infusion of dextrose 5% in half-normal saline and obtain nasopharyngeal and sputum specimens

The nurse receives an order to obtain a sputum sample from a client with hemoptysis. When advising the client of the physician's order, the client states not being able to produce sputum. Which suggestion, offered by the nurse, is helpful in producing the sputum sample? a) Drink 8 oz of water to thin the secretions for expectoration. b) Take deep breaths and cough forcefully. c) Use the secretions present in the oral cavity. d) Tickle the back of the throat to produce the gag reflex

B. Taking deep breaths moves air around the sputum and coughing forcefully moves the sputum up the respiratory tract. Once in the pharynx, the sputum can be expectorated into a specimen container. Producing a gag reflex elicits stomach contents and not respiratory sputum. Dilute and thinned secretions are not helpful in aiding expectoration. A sputum culture is not a component of oral secretions.

The pathophysiology of emphysema is directly related to airway obstruction. The end result of deterioration is respiratory acidosis from airway obstruction. Knowing this, the nursing student would choose which of the following as the highest priority action? a) Educate the importance of pursed lip breathing. b) Assess vital signs every 2 hours including O2 saturations and ABG results. c) Refer to respiratory therapy if breathing becomes labored. d) Apply oxygen as ordered via nasal cannula.

B. The acuity of the onset and severity of respiratory failure depend on baseline pulmonary function, pulse oximetry or arterial blood gas values, comorbid conditions, and the severity of other complications of COPD

A client with chronic obstructive pulmonary disease (COPD) is intubated and placed on continuous mechanical ventilation. Which equipment is most important for the nurse to keep at this client's bedside? A. Oxygen analyzer B. Manual resuscitation bag C. Water-seal chest drainage set up D. Tracheostomy cleaning kit

B. The client with COPD depends on mechanical ventilation for adequate tissue oxygenation. The nurse must keep a manual resuscitation bag at the bedside to ventilate and oxygenate the client in case the mechanical ventilator malfunctions. Because the client doesn't have chest tubes or a tracheostomy, keeping a water-seal chest drainage set-up or a tracheostomy cleaning kit at the bedside isn't necessary. Although the nurse may keep an oxygen analyzer (pulse oximeter) on hand to evaluate the effectiveness of ventilation, this equipment is less important than the manual resuscitation bag

The most diagnostic clinical symptom of pleurisy is: a) Dyspnea and coughing. b) Stabbing pain during respiratory movements. c) Dullness or flatness on percussion over areas of collected fluid. d) Fever and chills.

B. The key characteristic of pleuritic pain is its relationship to respiratory movement: taking a deep breath, coughing, or sneezing worsens the pain. Pleuritic pain is limited in distribution rather than diffuse; it usually occurs only on one side. The pain may become minimal or absent when the breath is held; leading to rapid shallow breathing. It may be localized or radiate to the shoulder or abdomen. Later, as pleural fluid accumulates, the pain decreases

A 6-month-old male client and his elder brother, a 3-year-old male, are being seen in the pediatric clinic for their third middle ear infection of the winter. The mother reports they develop an upper respiratory infection and an ear infection seems quick to follow. What contributes to this event? a) Oropharynx b) Eustachian tubes c) Genetics d) Epiglottis

B. The nasopharynx contains the adenoids and openings of the eustachian tubes. The eustachian tubes connect the pharynx to the middle ear and are the means by which upper respiratory infections spread to the middle ear. The client's infection is not caused by genetics. The oropharynx contains the tongue. The epiglottis closes during swallowing and relaxes during respiration

Of the following oxygen administration devices, which has the advantage of providing high oxygen concentration? a) Face tent b) Nonrebreather mask c) Venturi mask d) Catheter

B. The nonrebreather mask provides high oxygen concentration, but it is usually poorly fitting. However, if the nonrebreathing mask fits the patient snugly and both side exhalation ports have one-way valves, it is possible for the patient to receive 100% oxygen, making the nonrebreathing mask a high-flow oxygen system. The Venturi mask provides low levels of supplemental oxygen. The catheter is an inexpensive device that provides a variable fraction of inspired oxygen and may cause gastric distention. A face tent provides a fairly accurate fraction of inspired oxygen but is bulky and uncomfortable. It would not be the device of choice to provide high oxygen concentration

A 62-year-old male client with a history of chronic laryngitis arrives at the clinic complaining of a hoarseness "he can't shake." The nurse is aware that this client may be at risk for which of the following conditions? a) Coryza b) Laryngeal cancer c) Adenoiditis d) Peritonsillar abscess

B. The nurse knows that laryngeal cancer is most common in people 60 to 70 years of age, with men affected more frequently than are women. The client's history of chronic laryngitis may also predispose him to the development of laryngeal cancer. Sore throat, difficulty or pain on swallowing, fever, and malaise are the most common symptoms of adenoiditis. Enlarged adenoids may produce nasal obstruction, noisy breathing, snoring, and a nasal quality to the voice. This is another term for the common cold. Symptoms include sneezing, sore throat, and nasal congestions. Clients with a peritonsillar abscess experience difficulty and pain with swallowing, fever, malaise, ear pain, and difficulty talking

A client experiences a head injury in a motor vehicle accident. The client's level of consciousness is declining, and respirations have become slow and shallow. When monitoring a client's respiratory status, which area of the brain would the nurse realize is responsible for the rate and depth? a) Central sulcus b) The pons c) Wernicke's area d) The frontal lobe

B. The pons in the brainstem controls rate and depth of respirations. When injury occurs or increased intracranial pressure results, respirations are slowed. The frontal lobe completes executive functions and cognition. The central sulcus is a fold in the cerebral cortex called the central fissure. The Wernicke's area is the area linked to speech

The term for the volume of air inhaled and exhaled with each breath is: A. Expiratory reserve volume B. Tidal volume C. Residual volume D. Vital capacity

B. Tidal volume is the volume of air inhaled and exhaled with each breath. Residual volume is the volume of air remaining in the lungs after a maximum expiration. Vital capacity is the maximum volume of air exhaled from the point of maximum inspiration. Expiratory reserve volume is the maximum volume of air that can be exhaled after a normal inhalation.

A 53-year-old male is a regular client in the respiratory group where you practice nursing. As with all adults, millions of alveoli form most of the pulmonary mass. The squamous epithelial cells lining each alveolus consist of different types of cells. Which type of the alveoli cells produce surfactant? a) Type IV cells. b) Type II cells c) Type III cells. d) Type I cells

B. Type II cells—produce surfactant, a phospholipid that alters the surface tension of alveoli, preventing their collapse during expiration and limiting their expansion during inspiration. Type I cells line most alveolar surfaces. The epithelium of the alveoli does not contain Type IV cells. Type III cells destroy foreign material, such as bacteria

A nurse is weaning a client from mechanical ventilation. Which assessment finding indicates the weaning process should be stopped? a) Oxygen saturation of 93% b) Runs of ventricular tachycardia c) Respiratory rate of 16 breaths/minute d) Blood pressure increase from 120/74 mm Hg to 134/80 mm Hg

B. Ventricular tachycardia indicates that the client isn't tolerating the weaning process. The weaning process should be stopped before lethal ventricular arrhythmias occur. A respiratory rate of 16 breaths/minute and an oxygen saturation of 93% are normal findings. Although the client's blood pressure has increased, it hasn't increased more than 20% over baseline, which would indicate that the client isn't tolerating the weaning process

A nurse understands that a safe but low level of oxygen saturation provides for adequate tissue saturation while allowing no reserve for situations that threaten ventilation. What is a safe but low oxygen saturation level for a patient? a) 40% b) 95% c) 75% d) 80%

B. With a normal value for the partial pressure of oxygen (PaO2) (80 to 100 mm Hg) and oxygen saturation (SaO2) (95% to 98%), there is a 15% margin of excess oxygen available to the tissues. With a normal hemoglobin level of 15 mg/dL and a PaO2 level of 40 mm Hg (SaO2 75%), there is adequate oxygen available for the tissues but no reserve for physiologic stresses that increase tissue oxygen demand.

The nurse insects the thorax of a PT with advanced emphysema. What does the nurse except the chest configuration to be for this patient?

Barrel chest

Which is a deformity of the chest that occurs as a result of overinflation of the lungs?

Barrel chest Explanation: A barrel chest occurs as a result of overinflation of the lungs. The anteroposterior diameter of the thorax increases. Funnel chest occurs when a depression occurs in the lower portion of the sternum, which may result in murmurs. Pigeon chest occurs as a result of displacement of the sternum, resulting in an increase in the anteroposterior diameter. Kyphoscoliosis is characterized by elevation of the scapula and a corresponding S-shaped spine. This deformity limits lung expansion within the thorax.

A nurse is wroking with a client being extubated from the ventilator. Before weaning a client from a ventilator, which assessment parameter is the most important for the nurse to obtain?

Baseline arterial blood gas (ABG) levels Before weaning the client from mechanical ventilation, it's most important to have baseline ABG levels. During the weaning process, ABG levels will be checked to assess how the client is tolerating the procedure. Other assessment parameters are less critical. Measuring fluid volume intake and output is always important when a client is being mechanically ventilated. Anemic hypoxia is an issue but would not be most important before weaning ECG results are documented on the client's record, and the nurse can refer to them before the weaning process

The acute medical nurse is preparing to wean a client from the ventilator. Which assessment parameter is most important for the nurse to assess?

Baseline arterial blood gas (ABG) levels Explanation: Before weaning a client from mechanical ventilation, it is most important to have baseline ABG levels. During the weaning process, ABG levels will be checked to assess how the client is tolerating the procedure. Other assessment parameters are relevant, but less critical. Measuring fluid volume intake and output is always important when a client is being mechanically ventilated. Prior attempts at weaning and ECG results are documented on the client's record, and the nurse can refer to them before the weaning process begins.

If concern exists about fluid accumulation in a client's lungs, what area of the lungs will the nurse focus on during assessment? Anterior bronchioles Bilateral lower lobes Left lower lobe Posterior bronchioles

Bilateral lower lobes Crackles are secondary to fluid in the alveoli and create a soft, discontinuous popping sound. Because fluid creates these adventitious sounds, the principle of gravity will remind the nurse to focus the assessment on the lower portion of the thorax or the lower lobes of the lungs.

The nurse is performing an assessment of a PT who arrived in the emergency department with a barbiturate overdose. The respiration's are normal for 3-4 breaths followed by a 60-second period of apnea. How does the nurse document the respirations?

Biot respirations

While assessing an acutely ill client's respiratory rate, the nurse assesses four normal breaths followed by an episode of apnea lasting 20 seconds. How should the nurse document this finding?

Biot's respiration The nurse will document that the client is demonstrating a Biot's respiration pattern. Biot's respiration is characterized by periods of normal breathing (three to four breaths) followed by varying periods of apnea (usually 10 seconds to 1 minute). Cheyne-Stokes is a similar respiratory pattern, but it involves a regular cycle where the rate and depth of breathing increase and then decrease until apnea occurs. Biot's respiration is not characterized by the increase and decrease in the rate and depth, as characterized by Cheyne-Stokes. Eupnea is a normal breathing pattern of 12 to 18 breaths per minute. Bradypnea is a slower-than-normal rate (<10 breaths per minute), with normal depth and regular rhythm, and no apnea.

The nurse is caring for an adolescent client injured in a snowboarding accident. The client has a head injury, a fractured right rib, and various abrasions and contusions. The client has a blood pressure of 142/88 mm Hg, pulse of 102 beats/minute, and respirations of 26 breaths/minute. Which laboratory test best provides data on a potential impairment in ventilation?

Blood gases Explanation: Blood gases report the partial pressure of oxygen, which is dissolved in the blood. Normal readings are 80 to 100 mm Hg. By documenting oxygen levels in the blood, the nurse recognizes the current ventilation. The complete blood count provides information regarding number of blood cells, which can relate to the disease processes such as anemia and infection. The blood chemistry provides information on liver/renal function and electrolytes within the system. Serum alkaline phosphate is a laboratory test used to help detect liver disease and bone disorders.

In a patient diagnosed with increased intracranial pressure (IICP), the nurse would expect to observe which of the following respiratory rate or depth?

Bradypnea Explanation: Bradypnea is a slower than normal rate (<10 breaths/minute), with normal depth and regular rhythm. It is associated with IICP, brain injury, central nervous system depressants, and drug overdose. Tachypnea is associated with metabolic acidosis, septicemia, severe pain, and rib fracture. Hypoventilation is shallow, irregular breathing. Hyperventilation is an increased rate and depth of breathing.

"Swallowing down the wrong pipe" typically causes a significant coughing spasm and gasping for air, followed by recovery. Which upper airway structure malfunctions to cause the event? A. tonsils B. nasopharynx C. epiglottis D. oropharynx

C

A client has been newly diagnosed with emphysema. The nurse should explain to the client that by definition, ventilation: A. is when the body changes the oxygen into CO2 B. provides a blood suply to the lungs C. is breathing air in and out of the lungs D. helps people who cannot breathe on their own

C

A client is at risk for respiratory alkalosis. In attempt to restore the acid-base balance, the client's: A. lungs "blow off" CO2 to raise pH B. lungs retain CO2 to lower pH C. kidneys will excrete more HCO3 to lower the pH D. kidneys retain more HCO3 to raise pH

C

A client originally admitted for surgery has suffered multiple postoperative complications. The pulmonologist has scheduled a thoracentesis diagnostic purposes. What does serous fluid indicate? A. trauma B. infection C. inflammation D. emphysema

C

A client's arterial blood gas results reveal a pH of 7.35. The nurse would expect to find that the patient's PaCO2 is: A. 50 B. 25 C. 35 D. 80

C

Understanding pulmonary physiology, what characteristic would the nurse expect to result in decreased gas exchange in older adults? A. increased pulmonary elasticity B. thickening alveolar walls C. alveolar walls containing fewer capillaries D. reduced numbers of alveoli

C

While palpating the chest wall of a muscular athletic client, the nurse would expect to hear: A. highly palpable fremitus B. no fremitus C. decreased fremitus D. increased fremitus

C

A client experiencing an asthmatic attack is prescribed methylprednisolone (Solu-Medrol) intravenously. The nurse: a) Aspirates for blood return before injecting the medication b) Informs the client to limit fluid intake due to fluid retention c) Assesses fasting blood glucose levels d) Encourages the client to decrease caloric intake due to increased appetite

C. Adverse effects of methylprednisolone include abnormalities in glucose metabolism. The nurse monitors blood glucose levels. Methylprednisolone also increases the client's appetite and fluid retention, but the client will not decrease caloric or fluid intake as a result of these adverse effects. It is not necessary to aspirate for blood return prior to injecting the medication, because doing so would not support the intravenous line in the vein

You are caring for a client who is 42-years-old and status post adenoidectomy. You find the client in respiratory distress when you enter their room. You ask another nurse to call the physician and bring an endotracheal tube into the room. What do you suspect? a) Post operative bleeding b) Plugged tracheostomy tube c) Edema of the upper airway d) Infection

C. An endotracheal tube is inserted through the mouth or nose into the trachea to provide a patent airway for clients who cannot maintain an adequate airway on their own. The scenario does not indicate infection, post operative bleeding, or a plugged tracheostomy tube

A nursing student understands the importance of the psychosocial aspects of disease processes. When working with a patient with COPD, the student would rank which of the following nursing diagnoses as the MOST important when analyzing the psychosocial effects? a) High risk for ineffective therapeutic regimen management related to lack of knowledge b) Disturbed sleep pattern related to cough c) Ineffective coping related to anxiety d) Activity intolerance related to fatigue

C. Any factor that interferes with normal breathing quite naturally induces anxiety, depression, and changes in behavior. Constant shortness of breath and fatigue may make the patient irritable and apprehensive to the point of panic. Although the other choices are correct, the most important psychosocial nursing diagnosis for a patient with COPD is ineffective coping related to a high level of anxiety.

A 65-year-old client who works construction, and has been demolishing an older building,is diagnosed with pneumoconiosis. The nurse is aware that his lung inflammation is most likely caused by exposure to which of the following? a) Silica b) Pollen c) Asbestos d) Coal dust

C. Asbestosis is caused by inhalation of asbestos dust, which is frequently encountered during construction work, particularly when working with older buildings. Laws restrict asbestos use, but old materials still contain asbestos. Inhalation of silica may cause silicosis, which results from inhalation of silica dust and is seen in workers involved with mining, quarrying, stone-cutting, and tunnel building. Inhalation of coal dust and other dusts may cause black lung disease. Pollen may cause an allergic reaction, but is unlikely to cause pneumoconiosis

Which ventilator mode provides full ventilatory support by delivering a present tidal volume and respiratory rate? a) IMV b) Pressure support c) Assist-control d) SIMV

C. Assist-control ventilation provides full ventilator support by delivering a preset tidal volume and respiratory rate. Intermittent mandatory ventilation (IMV) provides a combination of mechanically assisted breaths and spontaneous breaths. SIMV delivers a preset tidal volume and number of breaths per minute. Between ventilator-delivered breaths, the patient can breathe spontaneously with no assistance from the ventilator for those extra breaths.

A client is admitted to the health care facility with active tuberculosis (TB). The nurse should include which intervention in the care plan? a) Wearing a gown and gloves when providing direct care b) Instructing the client to wear a mask at all times c) Putting on an individually fitted mask when entering the client's room d) Keeping the door to the client's room open to observe the client

C. Because TB is transmitted by droplet nuclei from the respiratory tract, the nurse should put on a mask when entering the client's room. Occupation Safety and Health Administration standards require an individually fitted mask. Having the client wear a mask at all times would hinder sputum expectoration and respirations would make the mask moist. A nurse who doesn't anticipate contact with the client's blood or body fluids need not wear a gown or gloves when providing direct care. A client with TB should be in a room with laminar airflow, and the room's door should be shut at all times.

A junior-level nursing class has just finished learning about the management of clients with chronic pulmonary diseases. They learned that a new definition of COPD leaves only one disorder within its classification. Which of the following is that disorder? a) Asthma b) Bronchiectasis c) Emphysema d) Cystic fibrosis

C. COPD may include diseases that cause airflow obstruction (eg, emphysema, chronic bronchitis) or any combination of these disorders. Other diseases such as cystic fibrosis, bronchiectasis, and asthma that were previously classified as types of COPD are now classified as chronic pulmonary disorders. Asthma is now considered a distinct, separate disorder and is classified as an abnormal airway condition characterized primarily by reversible inflammation

A thoracentesis is performed to obtain a sample of pleural fluid or a biopsy specimen from the pleural wall for diagnostic purposes. What does serous fluid indicate? A. Trauma B. Infection C. Cancer D.

C. Cancer. A thoracentesis may be performed to obtain a sample of pleural fluid or to biopsy a specimen from the pleural wall for diagnostic purposes. Serous fluid may be associated with cancer, inflammatory conditions, or heart failure. Blood fluid typically suggests trauma. Purulent fluid is diagnostic for infection. Complications that may follow a thoracentesis include pneumothorax and subcutaneous emphysema.

The nurse is assessing a patient who has been admitted with possible ARDS. What findings would distinguish ARDS from cardiogenic pulmonary edema? a) Elevated troponin levels b) Elevated white blood count c) Elevated B-type natriuretic peptide (BNP) levels d) Elevated myoglobin levels

C. Common diagnostic tests performed in patients with potential ARDS include plasma brain natriuretic peptide (BNP) levels, echocardiography, and pulmonary artery catheterization. The BNP level is helpful in distinguishing ARDS from cardiogenic pulmonary edema

A client suffers acute respiratory distress syndrome as a consequence of shock. The client's condition deteriorates rapidly, and endotracheal (ET) intubation and mechanical ventilation are initiated. When the high-pressure alarm on the mechanical ventilator sounds, the nurse starts to check for the cause. Which condition triggers the high-pressure alarm? a) An ET cuff leak b) A change in the oxygen concentration without resetting the oxygen level alarm c) Kinking of the ventilator tubing d) A disconnected ventilator tube

C. Conditions that trigger the high-pressure alarm include kinking of the ventilator tubing, bronchospasm, pulmonary embolus, mucus plugging, water in the tube, and coughing or biting on the ET tube. The alarm may also be triggered when the client's breathing is out of rhythm with the ventilator. A disconnected ventilator tube or an ET cuff leak would trigger the low-pressure alarm. Changing the oxygen concentration without resetting the oxygen level alarm would trigger the oxygen alarm, not the high-pressure alarm.

Lung compliance (the ability of the lungs to stretch) is a physical factor that affects ventilation. A nurse is aware that a patient who has lost elasticity in the lung tissue has a condition known as: a) Pulmonary edema b) Atelectasis c) Emphysema d) Pleural effusion

C. Emphysema, most commonly caused by smoking cigarettes, results in a loss of lung elasticity, which destroys the capillaries that supply the alveoli. This causes airway collapse during expiration, dyspnea, and eventually cyanosis

Which of the following interventions regarding nutrition is implemented for patients who have undergone laryngectomy? a) Season food to suit an increased sense of taste and smell b) Recommend the long-term use of zinc lozenges c) Use enteral feedings after the procedure d) Offer plenty of thin liquids when intake resumes

C. Enteral feedings are used 10 to 14 days after a laryngectomy to avoid irritation to the sutures and reduce the risk of aspiration. When oral intake resumes, the nurse offers small amounts of thick liquids. Following a laryngectomy, the patient may experience anorexia related to a diminished sense of taste and smell. Excess zinc can impair the immune system and lower the levels of high-density lipoproteins ("good" cholesterol). Therefore, long-term or ongoing use of zinc lozenges to prevent a cold is not recommended

What client would be most in need of an endotracheal tube? a) Ambulatory clients b) Older adult clients c) Comatose clients d) A client status post tonsillectomy

C. Examples include those with respiratory difficulty, comatose clients, those undergoing general anesthesia, and clients with extensive edema of upper airway passages

You are an occupational nurse completing routine assessments on the employees where you work. What might be revealed by a chest radiograph for a client with occupational lung diseases? a) Lung contusion b) Hemorrhage c) Fibrotic changes in lungs d) Damage to surrounding tissues

C. For a client with occupational lung diseases, a chest radiograph may reveal fibrotic changes in the lungs. Hemorrhage, lung contusion, and damage to surrounding tissues are possibly caused by trauma due to chest injuries

What dietary recommendations should a nurse provide a patient with a lung abscess? a) A diet with limited fat b) A carbohydrate-dense diet c) A diet rich in protein d) A diet low in calories

C. For a patient with pleural effusion, a diet rich in protein and calories is pivotal. A carbohydrate-dense diet or diets with limited fat are not advisable for a patient with lung abscess

A nurse takes the initial history of a patient who is being examined for cancer of the larynx. Select the sign that is considered an early clinical indicator. a) Cervical lymph adenopathy b) Dysphasia c) Hoarseness of more than 2 week's duration d) Persistent ulceration

C. Hoarseness of more than 2 weeks' duration occurs in the patient with cancer in the glottic area, because the tumor impedes the action of the vocal cords during speech. The voice may sound harsh, raspy, and lower in pitch. Later symptoms include dysphasia, dyspnea (difficulty breathing), unilateral nasal obstruction or discharge, persistent hoarseness, persistent ulceration, and foul breath. Cervical lymph adenopathy, unintentional weight loss, a general debilitated state, and pain radiating to the ear may occur with metastasis

The nurse is assisting a physician with an endotracheal intubation for a client in respiratory failure. It is most important for the nurse to assess for: a) Cool air humidified through the tube b) Tracheal cuff pressure set at 30 mm Hg c) Symmetry of the client's chest expansion d) A scheduled time for deflation of the tracheal cuff

C. Immediately after intubation, the nurse should check for symmetry of chest expansion. This is one finding that indicates successful endotracheal placement. The tracheal cuff pressure is set between 15 and 20 mm Hg. Routine deflation of the tracheal cuff is not recommended, because the client could then aspirate secretions during the period of deflation. Warm, high, humidified air is administered through the endotracheal tube

The nurse knows that a sputum culture is necessary to identify the causative organism for acute tracheobronchitis. What causative fungal organism would the nurse suspect? a) Haemophilus b) Mycoplasma pneumoniae c) Aspergillus d) Streptococcus pneumoniae

C. In acute tracheobronchitis, the inflamed mucosa of the bronchi produces mucopurulent sputum, often in response to infection by Streptococcus pneumoniae, Haemophilus influenzae, or Mycoplasma pneumoniae. In addition, a fungal infection (e.g., Aspergillus) may also cause tracheobronchitis. A sputum culture is essential to identify the specific causative organism.

As part of a primary cancer prevention program, an oncology nurse answers questions from the public at a health fair. When someone asks about laryngeal cancer, the nurse should explain that: a) laryngeal cancer occurs primarily in women. b) adenocarcinoma accounts for most cases of laryngeal cancer. c) laryngeal cancer is one of the most preventable types of cancer. d) inhaling polluted air isn't a risk factor for laryngeal cancer.

C. Laryngeal cancer is one of the most preventable types of cancer; it can be prevented by abstaining from excessive drinking and smoking. Inhaling noxious fumes, such as in polluted air, is a risk factor for laryngeal cancer. Roughly 80% of laryngeal cancer cases occur in men. Squamous cell carcinoma accounts for most cases of laryngeal cancer.

A student nurse is working with a client who is diagnosed with head trauma. The nurse has documented Cheyne-Stokes respirations. The student would expect to see which of the following? a) Irregular breathing at 14 to 18 breaths per minute b) Period of cessation of breathing c) Regular breathing where the rate and depth increase, then decrease d) Periods of normal breathing followed by periods of apnea

C. Observing the rate and depth of respiration is an important aspect of the nursing assessment. Certain patterns of breathing are characteristic of specific disease states or conditions. Head trauma can cause damage to the respiratory center in the brain, thereby altering the rate and depth of respirations. Cheyne-Stokes breathing is characterized by a regular cycle in which the rate and depth of breathing increase, then decrease until apnea occurs

A patient describes his chest pain as knife-like on inspiration. Which of the following is the most likely diagnosis? a) A lung infection b) Bronchogenic carcinoma c) Pleurisy d) Bacterial pneumonia

C. Pleuritic pain from irritation of the parietal pleura is sharp and seems to "catch" on inspiration. Some patients describe the pain as being "stabbed by a knife." Chest pain associated with the other conditions may be dull, aching, and persistent.

The nurse is caring for a client diagnosed with asthma. While performing the shift assessment, the nurse auscultates breath sounds including sibilant wheezes, which are continuous musical sounds. What characteristics describe sibilant wheezes? a) They occur when the pleural surfaces are inflamed. b) They are heard in clients with decreased secretions. c) They can be heard during inspiration and expiration. d) They result from air passing through widened air passages.

C. Sibilant or hissing or whistling wheezes are continuous musical sounds that can be heard during inspiration and expiration. They result from air passing through narrowed or partially obstructed air passages and are heard in clients with increased secretions. The crackling or grating sounds heard during inspiration or expiration are friction rubs. They occur when the pleural surfaces are inflamed

Upon palpation of the sinus area, what would the nurse identify as a normal finding? a) Tenderness during palpation b) Pain sensation behind the eyes c) No sensation during palpation d) Light not going through the sinus cavity

C. Sinus assessment involves using the thumbs to apply gentle pressure in an upward fashion at the sinuses. Tenderness suggests inflammation. The sinuses can be inspected by transillumination, where a light is passed through the sinuses. If the light fails to penetrate, the cavity contains fluid

The classification of Stage I of COPD is defined as a) severe COPD. b) at risk for COPD. c) mild COPD. d) very severe COPD.

C. Stage I is mild COPD. Stage 0 is at risk for COPD. Stage III is severe COPD. Stage IV is very severe COPD

In which stage of COPD is the forced expiratory volume (FEV) less than 30%? a) 0 b) II c) III d) I

C. Stage III patients demonstrate an FEV less than 30% with respiratory failure or clinical signs of right heart failure. Stage II patients demonstrate an FEV of 30% to 80%. Stage I is mild COPD with an FEV less than 70%. Stage 0 is characterized by normal spirometry

The goal for oxygen therapy in COPD is to support tissue oxygenation, decrease the work of the cardiopulmonary system, and maintain the resting partial arterial pressure of oxygen (PaO2) of at least ______ mm Hg and an arterial oxygen saturation (SaO2) of at least ___%. a) 58 mm Hg; 88% b) 56 mm Hg; 86% c) 60 mm Hg; 90% d) 54 mm Hg; 84%

C. The goal is a PaO2 of at least 60 mm Hg and an SaO2 of 90%.

The nurse is educating a patient with COPD about the technique for performing pursed-lip breathing. What does the nurse inform the patient is the importance of using this technique? a) It will prevent the alveoli from overexpanding. b) It increases the respiratory rate to improve oxygenation. c) It prolongs exhalation. d) It will assist with widening the airway.

C. The goal of pursed-lip breathing is to prolong exhalation and increase airway pressure during expiration, thus reducing the amount of trapped air and the amount of airway resistance

The nurse is caring for a client experiencing laryngeal trauma. Upon assessment, swelling and bruising is noted to the neck. Which breath sound is anticipated? a) Rhonchi in the bronchial region b) Diminished breath sounds throughout c) Audible stridor without using a stethoscope d) Crackles in the bases of the lungs

C. The nurse anticipates hearing audible stridor without needing a stethoscope due to the neck swelling narrowing the airway. Rhonchi in the bronchial region is heard lower in the airways and crackles are heard in the bases of the lungs. Diminished breath sounds that occur throughout are indicative of airway obstruction and not indicative of laryngeal swelling.

The nurse is providing discharge instructions to a patient following nasal surgery who has nasal packing. Which of the following discharge instructions would be most appropriate for the patient? a) Decrease the amount of daily fluids. b) Administer normal saline nasal drops as ordered. c) Avoid sports activities for 6 weeks. d) Take aspirin for nasal discomfort.

C. The nurse instructs the patient to avoid sports activities for 6 weeks. There is no indication for the patient to refrain from taking oral fluids. Mouth rinses help to moisten the mucous membranes and to reduce the odor and taste of dried blood in the oropharynx and nasopharynx. The patient should take analgesic agents, such as acetaminophen or NSAIDs, (i.e., ibuprofen or naproxen) to decrease nasal discomfort, not aspirin. The patient does not need to use nasal drops when nasal packing is in place.

Which of the following clinical manifestations of hemorrhage is related to carotid artery rupture? a) Increased blood pressure b) Shallow respirations c) Increased pulse rate d) Dry skin

C. The nurse monitors vital signs for changes, particularly increased pulse rate, decreased blood pressure, and rapid, deep respirations. Cold, clammy, pale skin may indicate active bleeding.

A nurse prepares to perform postural drainage. How should the nurse ascertain the best position to facilitate clearing the lungs? a) Inspection b) Arterial blood gas (ABG) levels c) Auscultation d) Chest X-ray

C. The nurse should assess breath sounds before doing postural drainage to determine the areas that need draining. Inspection, chest X-rays, and ABG levels are all assessment parameters that give good information about respiratory function but aren't necessary to determine lung areas requiring postural drainage

You are doing preoperative teaching with a client scheduled for laryngeal surgery. What should you teach this client to help prevent atelectasis? a) Caution against frequent coughing. b) Monitor for signs of dysphagia. c) Encourage deep breathing every 2 hours. d) Provide meticulous mouth care every 4 hours.

C. The nurse should encourage a client undergoing laryngeal surgery to practice deep breathing and coughing every 2 hours while the client is awake. These measures prevent atelectasis and promote effective gas exchange. Monitoring for signs of dysphagia and providing meticulous mouth care every 4 hours are the interventions related to the client's caloric intake.

Which of the following interventions does a nurse implement for patients with empyema? a) Institute droplet precautions b) Place suspected patients together c) Encourage breathing exercises d) Do not allow visitors with respiratory infection

C. The nurse teaches the patient with empyema to do breathing exercises as prescribed. The nurse should institute droplet precautions and isolate suspected and confirmed influenza patients in private rooms or place suspected and confirmed patients together. The nurse does not allow visitors with symptoms of respiratory infection to visit the hospital to prevent outbreaks of influenza from occurring in health care settings

The nursing instructor is teaching a pre-nursing pathophysiology class. The class is covering the respiratory system. The instructor explains that the respiratory system is comprised of both the upper and lower respiratory system. The nose is part of the upper respiratory system. The instructor continues to explain that the nasal cavities have a vascular and ciliated mucous lining. What is the purpose of the vascular and ciliated mucous lining of the nasal cavities? a) Move mucus to the back of the throat b) Cool and dry expired air c) Warm and humidify inspired air d) Moisten and filter expired air

C. The vascular and ciliated mucous lining of the nasal cavities warms and humidifies inspired air. It is the function of the cilia alone to move mucus in the nasal cavities and filter the inspired air

The body of a critically ill client may use which of the following homeostatic mechanisms to maintain normal pH? a) The lungs increase respiratory volume. b) The lungs retain more CO2 to lower the pH. c) The lungs eliminate carbonic acid by blowing off more CO2. d) The kidneys retain more HCO3 to raise the pH.

C. To maintain normal pH in critically ill clients, the lungs eliminate carbonic acid by blowing off more CO2. To maintain normal pH in critically ill clients, the lungs conserve CO2 by slowing respiratory volume. This is the way the body would compensate during an acid-base imbalance in cases of metabolic alkalosis. This is the way the body would compensate during an acid-base imbalance in cases of metabolic acidosis

A patient comes to the clinic with complaints of a sore throat and is diagnosed with acute pharyngitis. What does the nurse understand is the cause of acute pharyngitis? a) Gram-negative Klebsiella b) Pseudomonas aeruginosa c) Group A, beta-hemolytic streptococci d) Staphylococcus aureus

C. Viral infection causes most cases of acute pharyngitis. Responsible viruses include the adenovirus, influenza virus, Epstein-Barr virus, and herpes simplex virus. Bacterial infection accounts for the remainder of cases. Ten percent of adults with pharyngitis have group A beta-hemolytic streptococcus (GABHS), which is commonly referred to as group A streptococcus (GAS) or streptococcal pharyngitis

The nurse is auscultating the patient's lung sounds to determine the presence of pulmonary edema. What adventitious lung sounds are significant for pulmonary edema? a) Low-pitched rhonchi during expiration b) Sibilant wheezes c) Crackles in the lung bases d) Pleural friction rub

C. When clinically significant atelectasis develops, it is generally characterized by increased work of breathing and hypoxemia. Decreased breath sounds and crackles are heard over the affected area.

The nurse working in the radiology clinic is assisting with a pulmonary angiography. The nurse knows that when monitoring clients after a pulmonary angiography, what should the physician be notified about? a) Excessive capillary refill b) Flushed feeling in the client c) Absent distal pulses d) Raised temperature in the affected limb

C. When monitoring clients after a pulmonary angiography, nurses must notify the physician about diminished or absent distal pulses, cool skin temperature in the affected limb, and poor capillary refill. When the contrast medium is infused, the client will sense a warm, flushed feeling.

The nurse is preparing to perform chest physiotherapy (CPT) on a patient. Which of the following patient statements would indicate the procedure is contraindicated. a) "I received my pain medication 10 minutes ago, let's do my CPT now." b) "I just changed into my running suit; we can do my CPT now." c) "I just finished eating my lunch, I'm ready for my CPT now." d) "I have been coughing all morning and am barely bringing anything up."

C. When performing CPT, the nurse ensures that the patient is comfortable, is not wearing restrictive clothing, and has not just eaten. The nurse gives medication for pain, as prescribed, before percussion and vibration and splints any incision and provides pillows for support, as needed. A goal of CPT is for the patient to be able to mobilize secretions; the patient who is having an unproductive cough is a candidate for CPT.

The nurse is reviewing the blood gas results for a patient with pneumonia. What arterial blood gas measurement best reflects the adequacy of alveolar ventilation? a) SaO2 b) pH c) PaCO2 d) PaO2

C. When the minute ventilation falls, alveolar ventilation in the lungs also decreases, and the PaCO2 increases.

Which type of ventilator has a pre-sent volume of air to be delivered with each inspiration? a) Pressure cycled b) Negative pressure c) Volume cycled d) Time cycled

C. With volume-cycled ventilation, the volume of air to be delivered with each inspiration is present. Negative pressure ventilators exert a negative pressure on the external chest. Time-cycled ventilators terminate or control inspiration after a preset time. When the pressure-cycled ventilator cycles on, it delivers a flow of air (inspiration) until it reaches a present pressure, and then cycles off, and expiration occurs passively

The health care provider has prescribed continuous positive airway pressure (CPAP) with the delivery of a client's high-flow oxygen therapy. The client asks the nurse what the benefit of CPAP is. What would be the nurse's best response?

CPAP allows a lower percentage of oxygen to be used with a similar effect. Explanation: Prevention of oxygen toxicity is achieved by using oxygen only as prescribed. Often, positive end-expiratory pressure (PEEP) or CPAP is used with oxygen therapy to reverse or prevent microatelectasis, thus allowing a lower percentage of oxygen to be used. Oxygen is moistened by passing through a humidification system. Changing the tubing on the oxygen therapy equipment is the best technique for controlling bacterial growth.

A client is prescribed postural drainage because secretions are building in the superior segment of the lower lobes. Which is the best position to teach the client to use for postural drainage?

Certain positions with the head either up or down or lying on one side or the other will promote drainage of secretions from the smaller bronchial airways to the main bronchi and trachea. Think of the anatomy of the lungs when answering this question.

Postural drainage has been ordered for a client who is having difficulty mobilizing bronchial secretions. Before repositioning the client and beginning treatment, the nurse should perform what health assessment?

Chest auscultation Chest auscultation should be performed before and after postural drainage in order to evaluate the effectiveness of the therapy. Percussion and palpation are less likely to provide clinically meaningful data for the nurse. PFTs are normally beyond the scope of the nurse and are not necessary immediately before postural drainage.

A hospitalized client with terminal heart failure is nearing the end of life. The nurse observes which of the following breathing patterns?

Cheyne-Stokes breathing is characterized by a regular cycle where the rate and depth of breathing increase, then decrease until apnea occurs. The duration of apnea varies but progresses in length. This breathing pattern is associated with heart failure, damage to the respiratory center in the brain, or both.

The nurse assesses a patient with a heart rate of 42 and a blood pressure of 70/46. What type of hypoxia does the nurse determine this patient is displaying?

Circulatory hypoxia Given the vital signs, this client appears to be in shock. Circulatory hypoxia results from inadequate capillary circulation and may be caused by decreased cardiac output, local vascular obstruction, low-flow states such as shock, or cardiac arrest. Although tissue partial pressure of oxygen (PO2) is reduced, arterial oxygen (PaO2) remains normal. Circulatory hypoxia is corrected by identifying and treating the underlying cause. The low blood pressure is consistent with circulatory hypoxia but not consistent with the other options. Anemic hypoxia is a result of decreased effective hemoglobin concentration. Histotoxic hypoxia occurs when a toxic substance interferes with the ability of tissues to use available oxygen. Hypoxemic hypoxia results from a low level of oxygen in the blood.

You are caring for three clients who have the following blood count values: Client A has 24,500 white blood cells (WBCs), client B has 13.4 g/dL hemoglobin, and client C has a 250,000/mm3 platelet count. Which statement correctly describes the condition of each client? a) Client A has a normal WBC count than normal, client B has a normal hemoglobin count, and client C has a normal platelet count. b) Client A has a higher WBC count than normal, client B has a normal hemoglobin count, and client C has a higher platelet count than normal. c) Client A has a normal WBC count, client B has a higher hemoglobin count than normal, and client C has a normal platelet count. d) Client A has a higher WBC count than normal, client B has a normal hemoglobin count, and client C has a normal platelet count.

Client A has a higher WBC count than normal, client B has a normal hemoglobin count, and client C has a normal platelet count. Explanation: The normal leukocyte count is between 5,000 and 10,000/mm3. Client A has an increased number of leukocytes greater than 10,000/mm3 and hence has leukocytosis. In ... (more) The normal leukocyte count is between 5,000 and 10,000/mm3. Client A has an increased number of leukocytes greater than 10,000/mm3 and hence has leukocytosis. In adults, the normal amount of hemoglobin is 12 to 17.4 g/dL; therefore, client B has a normal hemoglobin count. A normal circulating platelet count is 150,000 to 350,000/mm3 platelets; therefore, client C has a normal platelet count

What finding by the nurse may indicate that the patient has chronic hypoxia?

Clubbing of the fingers

What finding by the nurse may indicate that the client has chronic hypoxia?

Clubbing of the fingers Clubbing of the fingers is a change in the normal nail bed. It appears as sponginess of the nail bed and loss of the nail bed angle. It is a sign of lung disease that is found in patients with chronic hypoxic conditions, chronic lung infections, or malignancies of the lung. Cyanosis can be a very late indicator of hypoxia, but it is not a reliable sign of hypoxia. The other signs listed may represent only a temporary hypoxia.

The nurse auscultates crackles in a PT with a respiratory disorder. What condition does the nurse recognize that these symptoms are indicative of?

Collapsed alveoli

A nurse is caring for a client with multiple myeloma. Which laboratory value is the nurse most likely to see? a) Hypernatremia b) Hypermagnesemia c) Hypercalcemia d) Hyperkalemia

Correct response: Hypercalcemia Explanation: Calcium is released when bone is destroyed, causing hypercalcemia. Multiple myeloma doesn't affect potassium, sodium, or magnesium levels.

A client has been diagnosed with multiple myeloma. Which of the following laboratory values should the nurse expect to find in a client with multiple myeloma? a) Decreased calcium level b) Increased urinary protein c) Polycythaemia vera d) Decreased serum protein

Correct response: Increased urinary protein Explanation: A characteristic finding in multiple myeloma is protein in the urine. Other laboratory findings include increased serum protein, hypercalcaemia, anaemia, and hyperuricemia. Polycythaemia vera is not found in multiple myeloma

A client has a nursing diagnosis of "ineffective airway clearance" as a result of excessive secretions. An appropriate outcome for this client would be which of the following? a) Client reports no chest pain. b) Lungs are clear on auscultation. c) Respiratory rate is 12 to 18 breaths per minute. d) Client can perform incentive spirometry

Correct response: Lungs are clear on auscultation. Explanation: Assessment of lung sounds includes auscultation for airflow through the bronchial tree. The nurse evaluates for fluid or solid obstruction in the lung. When airflow is decreased, as with fluid or secretions, adventitious sounds may be auscultated. Often crackles are heard with fluid in the airways

A nurse is caring for a client admitted with pernicious anemia. Which set of findings should the nurse expect when assessing the client? a) Pallor, tachycardia, and a sore tongue b) Angina pectoris, double vision, and anorexia c) Sore tongue, dyspnea, and weight gain d) Pallor, bradycardia, and reduced pulse pressure

Correct response: Pallor, tachycardia, and a sore tongue Explanation: Pallor, tachycardia, and a sore tongue are all characteristic findings in pernicious anemia. Other clinical manifestations include anorexia; weight loss; a smooth, beefy red tongue; a wide pulse pressure; palpitations; angina pectoris; weakness; fatigue; and paresthesia of the hands and feet. Bradycardia, reduced pulse pressure, weight gain, and double vision aren't characteristic findings in pernicious anemia

A client is being admitted to the preoperative holding area for a thoracotomy. Preoperative teaching includes what?

Correct use of incentive spirometry Instruction in the use of incentive spirometry begins before surgery to familiarize the client with its correct use. You do not teach a client the use of a ventilator; you explain that he may be on a ventilator to help him breathe. Rhythmic breathing and mini-nebulizers are unnecessary.

A client appears to be breathing faster than during the last assessment. Which of the following interventions should the nurse perform? Assess the radial pulse. Assist the client to lie down. Inquire if there have been any stressful visitors. Count the rate of respirations.

Count the rate of respirations. Observing the rate and depth of respiration is an important aspect of a nursing assessment. The normal adult resting respiratory rate is 12 to 18 breaths per minute. Tachypnea is rapid breathing with a rate greater than 24 breaths per minute. An increase in the rate of respirations needs further investigation and must be reported.

A client appears to be breathing faster than during the last assessment. Which of the following actions should the nurse perform?

Count the rate of respirations. Explanation: Observing the rate and depth of respiration is an important aspect of a nursing assessment. The normal adult resting respiratory rate is 12 to 18 breaths per minute. Tachypnea is rapid breathing with a rate greater than 24 breaths per minute. An increase in the rate of respirations needs further investigation and must be reported.

A client arrives in the emergency department reporting shortness of breath. She has 3+ pitting edema below the knees, a respiratory rate of 36 breaths per minute, and heaving respirations. The nurse auscultates the client's lungs to reveal coarse, moist, high-pitched, and non-continuous sounds that do not clear with coughing. The nurse will document these sounds as which type?

Crackles

The nurse is assessing the lungs of a patient diagnosed with pulmonary edema. Which of the following would be expected upon auscultation?

Crackles at lung bases Explanation: A patient with pulmonary edema would be expected to have crackles in the lung bases, and possible wheezes. Egophony may occur in patients diagnosed with pleural effusion. Absent breath sounds occurs in pneumothorax. Bronchial breath sounds occur in consolidation, such as pneumonia.

The nurse is caring for the PT with a pulmonary disorder. What observation by the nurse is indicative of a very late symptom of hypoxia?

Cyanosis

When observing the chest wall of a PT the nurse detects a depression in the lower portion of the sternum. What assessment finding will the nurse anticipate detecting due to this condition?

Cyanosis

The nurse is assessing a patient in respiratory failure. What finding is a late indicator of hypoxia?

Cyanosis Cyanosis, a bluish coloring of the skin, is a very late indicator of hypoxia. The presence or absence of cyanosis is determined by the amount of unoxygenated hemoglobin in the blood. Cyanosis appears when there is at least 5 g/dL of unoxygenated hemoglobin.

The nurse is caring for a patient with a pulmonary disorder. What observation by the nurse is indicative of a very late symptom of hypoxia?

Cyanosis Cyanosis, a bluish coloring of the skin, is a very late indicator of hypoxia. The presence or absence of cyanosis is determined by the amount of unoxygenated hemoglobin in the blood. Cyanosis appears when there is at least 5 g/dL of unoxygenated hemoglobin.

A son brings his father into the clinic, stating that his father's color has changed to bluish around the mouth. The father is confused, with a respiratory rate of 28 breaths per minute and scattered crackles throughout. The son states this condition just occurred within the last hour. Which of the following factors indicates that the client's condition has lasted for more than 1 hour? Respiratory rate Cyanosis Son's statement Crackles

Cyanosis The client's appearance may give clues to respiratory status. Cyanosis, a bluish coloring of the skin, is a very late indicator of hypoxia. The presence of cyanosis is from decreased unoxygenated hemoglobin. In the presence of a pulmonary condition, cyanosis is assessed by observing the color of the tongue and lips.

The nurse is caring for a patient with a pulmonary disorder. What observation by the nurse is indicative of a very late symptom of hypoxia?

Cyanosis Explanation: Cyanosis, a bluish coloring of the skin, is a very late indicator of hypoxia. The presence or absence of cyanosis is determined by the amount of unoxygenated hemoglobin in the blood. Cyanosis appears when there is at least 5 g/dL of unoxygenated hemoglobin.

A son brings his father into the clinic, stating that his father's color has changed to bluish around the mouth. The father is confused, with a respiratory rate of 28 breaths per minute and scattered crackles throughout. The son states this condition just occurred within the last hour. Which of the following factors indicates that the client's condition has lasted for more than 1 hour?

Cyanosis Explanation: The client's appearance may give clues to respiratory status. Cyanosis, a bluish coloring of the skin, is a very late indicator of hypoxia. The presence of cyanosis is from decreased unoxygenated hemoglobin. In the presence of a pulmonary condition, cyanosis is assessed by observing the color of the tongue and lips.

A son brings his father into the clinic, stating that his father's color has changed to bluish around the mouth. The father is confused, with a respiratory rate of 28 breaths per minute and scattered crackles throughout. The son states this condition just occurred within the last hour. Which of the following factors indicates that the client's condition has lasted for more than 1 hour? a) Son's statement b) Cyanosis c) Respiratory rate d) Crackles

Cyanosis Explanation: The client's appearance may give clues to respiratory status. Cyanosis, a bluish coloring of the skin, is a very late indicator of hypoxia. The presence of cyanosis is from decreased unoxygenated hemoglobin. In the presence of a pulmonary condition, cyanosis is assessed by observing the color of the tongue and lips

A son brings his father into the clinic, stating that his father's color has changed to bluish around the mouth. The father is confused, with a respiratory rate of 28 breaths per minute and scattered crackles throughout. The son states this condition just occurred within the last hour. Which of the following factors indicates that the client's condition has lasted for more than 1 hour? a) Crackles b) Cyanosis c) Son's statement d) Respiratory rate

Cyanosis Explanation: The client's appearance may give clues to respiratory status. Cyanosis, a bluish coloring of the skin, is a very late indicator of hypoxia. The presence of cyanosis is from decreased unoxygenated hemoglobin. In the presence of a pulmonary condition, cyanosis is assessed by observing the color of the tongue and lips.

A 4 year old client has had recurrent episodes of bronchitis during the winter months. The client spends a significant amount of time outdoors enjoying the snow and breathes primarily through the mouth. What suggestion would the nurse most likely make to the client's mother? A. none of the options is correct B. increase the client's vitamin dosage C. keep the client indoors D. help the client learn to breathe through her nose

D

A client is having a pulmonary angiography. Which sign would indicate an allergic reaction to the contrast medium? A. hematoma B. absent distal pulses C. urge to cough D. difficulty breathing

D

A client is undergoing a thoracentesis. Which nursing action will the nurse perform following the procedure? A. place the client is a supine position B. ambulate the client following the procedure C. instruct the client to cough forcefully every ten minutes D. maintain the client on bed rest for several hours

D

During assessment of the respiratory system, the nurse inspects and palpates the trachea in order to assess: A. evidence of exudate B. color of the mucous membranes C. evidence of muscle weakness D. deviation from the midline

D

Millions of alveoli form of the pulmonarty mass. The squamous epithelial cell lining each alveolus of different types of cells. Which type of alveolar cells produce surfactant? A. type I cells B. type IV cells C. type III cells D. type II cells

D

Which homeostatic mechanism would the body of a critically ill client use to maintain normal pH? A. the lungs increase respiratory volume B. the lungs retain more CO2 to lower the pH C. the kidneys retain more HCO3 to raise the pH D. the lungs eliminate carbonic acid by blowing off more CO2

D

Hyperbaric oxygen therapy increases the blood's capacity to carry and deliver oxygen to compromised tissues. This therapy may be used for a client with: a) hyperthermia. b) a malignant tumor. c) pneumonia. d) a compromised skin graft.

D. A client with a compromised skin graft could benefit from hyperbaric oxygen therapy because increasing oxygenation at the wound site promotes wound healing. Hyperbaric oxygen therapy isn't indicated for malignant tumors, pneumonia, or hyperthermia.

A client with myasthenia gravis is receiving continuous mechanical ventilation. When the high-pressure alarm on the ventilator sounds, what should the nurse do? a) Check for an apical pulse. b) Increase the oxygen percentage. c) Ventilate the client with a handheld mechanical ventilator. d) Suction the client's artificial airway

D. A high-pressure alarm on a continuous mechanical ventilator indicates an obstruction in the flow of oxygen from the machine to the client. The nurse should suction the client's artificial airway to remove respiratory secretions that could be causing the obstruction. The sounding of a ventilator alarm has no relationship to the apical pulse. Increasing the oxygen percentage and ventilating with a handheld mechanical ventilator wouldn't correct the airflow blockage

The nurse is assessing the lungs of a patient diagnosed with pulmonary edema. Which of the following would be expected upon auscultation? a) Egophony b) Bronchial breath sounds c) Absent breath sounds d) Crackles at lung bases

D. A patient with pulmonary edema would be expected to have crackles in the lung bases, and possible wheezes. Egophony may occur in patients diagnosed with pleural effusion. Absent breath sounds occurs in pneumothorax. Bronchial breath sounds occur in consolidation, such as pneumonia.

A client is being discharged from an outpatient surgery center following a tonsillectomy. The nurse gives the following instructions: a) "You are allowed to have hot tea or coffee." b) "Gargle vigorously to clean your throat." c) "You may have a sore throat for 1 week." d) "Gargle with a warm salt solution."

D. A warm saline solution will help with removal of thick mucus and halitosis. It will be a gentle gargle, because a vigorous gargle may cause bleeding. A sore throat may be present for 3 to 5 days. Hot foods should be avoided

A client reports nasal congestion, sneezing, sore throat, and coughing up of yellow mucus. The nurse assesses the client's temperature as 100.2°F. The client states this is the third episode this season. The highest priority nursing diagnosis is a) Deficient knowledge related to prevention of upper respiratory infections b) Deficient fluid volume related to increased fluid needs c) Acute pain related to upper airway irritation d) Ineffective airway clearance related to excess mucus production

D. All the listed nursing diagnoses are appropriate for this client. Following Maslow's hierarchy of needs, physiological needs are addressed first and, within physiological needs, airway, breathing, and circulation are the most immediate. Thus, ineffective airway clearance is the priority nursing diagnosis

A client has a nursing diagnosis of "ineffective airway clearance" as a result of excessive secretions. An appropriate outcome for this client would be which of the following? a) Client reports no chest pain. b) Respiratory rate is 12 to 18 breaths per minute. c) Client can perform incentive spirometry. d) Lungs are clear on auscultation.

D. Assessment of lung sounds includes auscultation for airflow through the bronchial tree. The nurse evaluates for fluid or solid obstruction in the lung. When airflow is decreased, as with fluid or secretions, adventitious sounds may be auscultated. Often crackles are heard with fluid in the airways.

Which of the following is a true statement regarding severe acute respiratory syndrome (SARS)? a) Hypothermia will occur b) It is spread by fecal contamination c) Constipation usually develops d) It is the most contagious during the second week of illness

D. Based on available information, SARS is most likely to be contagious only when symptoms are present, and patients are most contagious during the second week of illness. Diarrhea and hyperthermia may occur with SARS. Respiratory droplets spread the SARS virus when an infected person coughs or sneezes

Before weaning a client from a ventilator, which assessment parameter is the most important for the nurse to obtain? a) Electrocardiogram (ECG) results b) Prior outcomes of weaning c) Fluid intake for the past 24 hours d) Baseline arterial blood gas (ABG) levels

D. Before weaning the client from mechanical ventilation, it's most important to have baseline ABG levels. During the weaning process, ABG levels will be checked to assess how the client is tolerating the procedure. Other assessment parameters are less critical. Measuring fluid volume intake and output is always important when a client is being mechanically ventilated. Prior attempts at weaning and ECG results are documented on the client's record, and the nurse can refer to them before the weaning process begins

What finding by the nurse may indicate that the patient has chronic hypoxia? a) Cyanosis b) Crackles c) Peripheral edema d) Clubbing of the fingers

D. Clubbing of the fingers is a change in the normal nail bed. It appears as sponginess of the nail bed and loss of the nail bed angle (Fig. 20-6). It is a sign of lung disease that is found in patients with chronic hypoxic conditions, chronic lung infections, or malignancies of the lung (Bickley, 2009). The other signs listed may represent only a temporary hypoxia.

A client undergoes a total laryngectomy and tracheostomy formation. On discharge, the nurse should give which instruction to the client and family? a) "Clean the tracheostomy tube with alcohol and water." b) "Limit the amount of protein in the diet." c) "Oral intake of fluids should be limited for 1 week only." d) "Family members should continue to talk to the client."

D. Commonly, family members are reluctant to talk to the client who has had a total laryngectomy and can no longer speak. To promote a supportive environment, the nurse should encourage family members to continue normal communication. The nurse should teach the client to clean the tracheostomy tube with hydrogen peroxide and rinse it with sterile saline solution, to consume oral fluids as desired, and to eat protein-rich foods to promote healing

A nurse observes constant bubbling in the water-seal chamber of a closed chest drainage system. What should the nurse conclude? a) The client has a pneumothorax. b) The chest tube is obstructed. c) The system is functioning normally. d) The system has an air leak.

D. Constant bubbling in the water-seal chamber indicates an air leak and requires immediate intervention. The client with a pneumothorax will have intermittent bubbling in the water-seal chamber. Clients without a pneumothorax should have no evidence of bubbling in the chamber. If the tube is obstructed, the fluid would stop fluctuating in the water-seal chamber.

Wound drains, inserted during the laryngectomy, stay in place until what criteria are met? a) The stoma is healed, about 6 weeks after surgery. b) The surgical site is dry with encrustations. c) The patient is able to assist with his own suctioning. d) Drainage is <30 mL/day for 2 consecutive days.

D. Drains are removed when secretions are minimal, which usually is less than 30 mL for 48 straight hours.

The nursing instructor is talking with the junior class of nursing students about lung cancer. What would be the best rationale the instructor could give for the difficulty of early diagnosis of lung cancer? a) Symptoms are often minimized by clients. b) There are no early symptoms of lung cancer. c) Symptoms often mimic other infectious diseases. d) Symptoms often do not appear until the disease is well established

D. Early diagnosis of cancer of the lung is difficult because symptoms often do not appear until the disease is well established. Option A is correct, but it is not the best answer. Option B is incorrect because it is not a true statement. Option C is incorrect because lung cancer is not an infectious disease

After lobectomy for lung cancer, a client receives a chest tube connected to a disposable chest drainage system. The nurse observes that the drainage system is functioning correctly when she notes tidal movements or fluctuations in which compartment of the system as the client breathes? a) Air-leak chamber b) Collection chamber c) Suction control chamber d) Water-seal chamber

D. Fluctuations in the water-seal compartment are called tidal movements and indicate normal function of the system as the pressure in the tubing changes with the client's respirations. The air-leak meter — not chamber — detects air leaking from the pleural space. The collection chamber connects the chest tube from the client to the system. Drainage from the tube drains into and collects in a series of calibrated columns in this chamber. The suction control chamber provides the suction, which can be controlled to provide negative pressure to the chest.

A physician stated to the nurse that the patient has fluid noted in the pleural space and will need a thoracentesis. The nurse would expect that the physician will document this fluid as which of the following? A. Hemothorax B. Consolidation C. Pneumothorax D. Pleural effusion

D. Fluid accumulating within the pleural space is called a pleural effusion. A pneumothorax is air in the pleural space. A hemothorax is blood within the pleural space. Consolidation is lung tissue that has become more solid in nature due to collapse of alveoli or infectious process.

The nurse assesses a patient with a heart rate of 42 and a blood pressure of 70/46. What type of hypoxia does the nurse determine this patient is displaying? a) Histotoxic hypoxia b) Anemic hypoxia c) Hypoxic hypoxia d) Circulatory hypoxia

D. Given this patient's vital signs, he appears to be in shock. Circulatory hypoxia is hypoxia resulting from inadequate capillary circulation. It may be caused by decreased cardiac output, local vascular obstruction, low-flow states such as shock, or cardiac arrest. Although tissue partial pressure of oxygen (PO2) is reduced, arterial oxygen (PaO2) remains normal. Circulatory hypoxia is corrected by identifying and treating the underlying cause.

The herpes simplex virus type 1 (HSV-1), which produces a cold sore (fever blister), has an incubation period of a) 3 to 6 months. b) 20 to 30 days. c) 0 to 3 months. d) 2 to12 days.

D. HSV-1 is transmitted primarily by direct contact with infected secretions. The time period 0 to 3 months exceeds the incubation period. The time period 20 to 30 days exceeds the incubation period. The time period 3 to 6 months exceeds the incubation period

A nurse is caring for a client who was intubated because of respiratory failure. The client is now receiving mechanical ventilation with a preset tidal volume and number of breaths each minute. The client has the ability to breathe spontaneously between the ventilator breaths with no ventilator assistance. The nurse should document the ventilator setting as: a) assist-control (AC) ventilation. b) pressure support ventilation (PSV). c) continuous positive airway pressure (CPAP). d) synchronized intermittent mandatory ventilation (SIMV).

D. In SIMV mode, the ventilator delivers a preset number of breaths at a preset tidal volume. The client can breathe on his own in between the breaths delivered by the ventilator. In PSV, a pressure plateau is added to the ventilator to prevent the airway pressure from falling beneath a preset level. In AC ventilation, the ventilator delivers a preset number of breaths at a preset tidal volume and any breaths that the client takes on his own are assisted by the ventilator so they reach the preset tidal volume. In CPAP, the ventilator provides only positive airway pressure; it doesn't provide any breaths to the client.

A client with a respiratory condition is receiving oxygen therapy. While assessing the client's PaO2, the nurse knows that the therapy has been effective based on which of the following readings? a) 45 mm Hg b) 58 mm Hg c) 120 mm Hg d) 84 mm Hg

D. In general, clients with respiratory conditions are given oxygen therapy only to increase the arterial oxygen pressure (PaO2) back to the client's normal baseline, which may vary from 60 to 95 mm Hg.

The client you are caring for has just been told they have advanced laryngeal cancer. What is the treatment of choice? a) Laser surgery b) Radiation therapy c) Partial laryngectomy d) Total laryngectomy

D. In more advanced cases, total laryngectomy may be the treatment of choice. Partial laryngectomy, laser surgery, and radiation therapy are not the treatment of choice for advanced cases of laryngeal cancer.

Histamine, a mediator that supports the inflammatory process in asthma, is secreted by a) Eosiniphils b) Neutrophils c) Lymphocytes d) Mast cells

D. Mast cells, neutrophils, eosinophils, and lymphocytes play key roles in the inflammation associated with asthma. When activated, mast cells release several chemicals called mediators. One of these chemicals is called histamine

Which of the following clinical manifestations should a nurse monitor for during a pulmonary angiography, which indicates an allergic reaction to the contrast medium? a) Hematoma b) Urge to cough c) Absent distal pulses d) Difficulty in breathing

D. Nurses must determine if the client has any allergies, particularly to iodine, shellfish, or contrast dye. During the procedure, the nurse should check for signs and symptoms of allergic reactions to the contrast medium, such as itching, hives, or difficulty in breathing. The nurses inspects for hematoma, absent distal pulses, after the procedure. When the contrast medium is infused, an urge to cough is often a sensation experienced by the client.

A patient is to receive an oxygen concentration of 70%. What is the best way for the nurse to deliver this concentration? a) A nasal cannula b) A Venturi mask c) An oropharyngeal catheter d) A partial rebreathing mask

D. Partial rebreathing masks have a reservoir bag that must remain inflated during both inspiration and expiration. The nurse adjusts the oxygen flow to ensure that the bag does not collapse during inhalation. A high concentration of oxygen (50% to 75%) can be delivered because both the mask and the bag serve as reservoirs for oxygen. The other devices listed cannot deliver oxygen at such a high concentration.

The nurse is caring for a client with chronic obstructive pulmonary disease. The client calls the doctor and states having difficulty breathing and overall feeling fatigued. The nurse realizes that this client is at high risk for which condition? a) Metabolic acidosis b) Respiratory alkalosis c) Metabolic alkalosis d) Respiratory acidosis

D. Respiratory acidosis occurs when the body is unable to blow off CO2 due to the hypoventilation of disease processes such as COPD. An increase in blood carbon dioxide concentration occurs and a decreased pH causing acidosis. Respiratory alkalosis is a decrease in acidity of the blood and often caused by hyperventilation. Metabolic acidosis/alkalosis are disorders that affect the bicarbonate

The nurse is performing chest auscultation for a patient with asthma. How does the nurse describe the high-pitched, sibilant, musical sounds that are heard? a) Rhonchi b) Rales c) Crackles d) Wheezes

D. Sibilant wheezes are continuous, musical, high-pitched, whistlelike sounds heard during inspiration and expiration caused by air passing through narrowed or partially obstructed airways; they may clear with coughing. Crackles, formerly called rales, are soft, high-pitched, discontinuous popping sounds that occur during inspiration (while usually heard on inspiration, they may also be heard on expiration); they may or may not be cleared by coughing. Rhonchi, or sonorous wheezes, are deep, low-pitched rumbling sounds heard primarily during expiration; they are caused by air moving through narrowed tracheobronchial passages.

A patient visited a health care clinic for treatment of upper respiratory tract congestion, fatigue, and sputum production that was rust-colored. Which of the following diagnoses is likely based on this history and inspection of the sputum? a) Bronchitis b) A lung abscess c) Bronchiectasis d) An infection with pneumococcal pneumonia

D. Sputum that is rust colored suggests infection with pneumococcal pneumonia. Bronchiectasis and a lung abscess usually are associated with purulent thick and yellow-green sputum. Bronchitis usually yields a small amount of purulent sputum.

A patient has a Mantoux skin test prior to being placed on an immunosuppressant for the treatment of Crohn's disease. What results would the nurse determine is not significant for holding the medication? a) 7 to 8 mm b) 9 mm c) 5 to 6 mm d) 0 to 4 mm

D. The Mantoux method is used to determine whether a person has been infected with the TB bacillus and is used widely in screening for latent M. tuberculosis infection. The size of the induration determines the significance of the reaction. A reaction of 0 to 4 mm is considered not significant. A reaction of 5 mm or greater may be significant in people who are considered to be at risk

During discharge teaching, a nurse is instructing a client about pneumonia. The client demonstrates his understanding of relapse when he states that he must: a) maintain fluid intake of 40 oz (1,200 ml) per day. b) follow up with the physician in 2 weeks. c) turn and reposition himself every 2 hours. d) continue to take antibiotics for the entire 10 days.

D. The client demonstrates understanding of how to prevent relapse when he states that he must continue taking the antibiotics for the prescribed 10-day course. Although the client should keep the follow-up appointment with the physician and turn and reposition himself frequently, these interventions don't prevent relapse. The client should drink 51 to 101 oz (1,500 to 3,000 ml) per day of clear liquids

A son brings his father into the clinic, stating that his father's color has changed to bluish around the mouth. The father is confused, with a respiratory rate of 28 breaths per minute and scattered crackles throughout. The son states this condition just occurred within the last hour. Which of the following factors indicates that the client's condition has lasted for more than 1 hour? a) Crackles b) Respiratory rate c) Son's statement d) Cyanosis

D. The client's appearance may give clues to respiratory status. Cyanosis, a bluish coloring of the skin, is a very late indicator of hypoxia. The presence of cyanosis is from decreased unoxygenated hemoglobin. In the presence of a pulmonary condition, cyanosis is assessed by observing the color of the tongue and lips

A young adult client has had a tonsillectomy and is in the immediate postoperative period. To make the client comfortable, the nurse intervenes by a) Removing the oral airway before the gag reflex has returned for client comfort b) Sitting the client in the semi-Fowler's position c) Maintaining a warm compress around the client's neck area d) Placing the client prone with the head turned to the side

D. The most comfortable position for the client in the immediate postoperative period is prone, not semi-Fowler's. The client's head is turned to the side to allow drainage from the mouth. The oral airway is removed after the gag reflex has returned. An ice collar, not warm compress, is applied to the neck area.

A client is receiving supplemental oxygen. When determining the effectiveness of oxygen therapy, which arterial blood gas value is most important? a) Bicarbonate (HCO3-) b) pH c) Partial pressure of arterial carbon dioxide (PaCO2) d) Partial pressure of arterial oxygen (PaO2)

D. The most significant and direct indicator of the effectiveness of oxygen therapy is the PaO2 value. Based on the PaO2 value, the nurse may adjust the type of oxygen delivery (cannula, Venturi mask, or mechanical ventilator), flow rate, and oxygen percentage. The other options reflect the client's ventilation status, not oxygenation. The pH, HCO3-, and PaCO2

A client is prescribed two sprays of a nasal medication twice a day. The nurse is teaching the client how to self-administer the medication and instructs the client to a) Tilt the head back when activating the spray of the medication. b) Wait 10 seconds before administering the second spray. c) Clean the medication container once each day. d) Blow the nose before applying medication into the nares.

D. The nurse instructs the client to blow the nose before administering the nasal medication. The client should keep the head upright, not tilted back. The client should wait at least 1 minute before administering the second spray and clean the container after each use

The nurse is caring for a client with allergic rhinitis. The patient asks the nurse about measures to help decrease allergic symptoms. The best response by the nurse is which of the following? A. Get your influenza vaccination yearly B. Take OTC nasal decongestants C. Refer to an ENT physician D. avoid exposure to allergens and irritants

D. The nurse instructs the patient with allergic rhinitis to avoid or reduce exposure to allergens and irritants, such as dusts, molds, animals, fumes, odors, powders, sprays, and tobacco smoke. Receiving an influenza vaccination each year is recommended for patients with infectious rhinitis. To prevent possible drug interactions, the patient is cautioned to read drug labels before taking any OTC medication. Patients with nasal septal deformities or nasal polyps may be referred to an ear, nose, and throat specialist.

The nurse assessed a 28-year-old woman who was experiencing dyspnea severe enough to make her seek medical attention. The history revealed no prior cardiac problems and the presence of symptoms for 6 months' duration. On assessment, the nurse noted the presence of both inspiratory and expiratory wheezing. Based on this data, which of the following diagnoses is likely? a) Acute respiratory obstruction b) Pneumothorax c) Adult respiratory distress syndrome d) Asthma

D. The presence of both inspiratory and expiratory wheezing usually signifies asthma if the individual does not have heart failure. Sudden dyspnea is an indicator of the other choices

Which of the following is true about both lung transplant and bullectomy? a) Both are aimed at curing COPD. b) Both are aimed at treating end-stage emphysema. c) Both are used to treat patients with bullous emphysema. d) Both are aimed at improving the overall quality of life of a patient with COPD.

D. The treatments for COPD are aimed more at treating the symptoms and preventing complications, thereby improving the overall quality of life of a patient with COPD. In fact, there is no cure for COPD. Lung transplant is aimed at treating end-stage emphysema and bullectomy is used to treat patients with bullous emphysema

In general, chest drainage tubes are not used for the patient undergoing a) wedge resection. b) lobectomy. c) segmentectomy. d) pneumonectomy.

D. Usually, no drains are used for the pneumonectomy patient because the accumulation of fluid in the empty hemothorax prevents mediastinal shift. With lobectomy, two chest tubes are usually inserted for drainage, the upper tube for air and the lower tube for fluid. With wedge resection, the pleural cavity usually is drained because of the possibility of an air or blood leak. With segmentectomy, drains are usually used because of the possibility of an air or blood leak.

What is the difference between respiration and ventilation? a) Ventilation is the exchange of gases in the lung. b) Ventilation is the process of gas exchange. c) Ventilation is the process of getting oxygen to the cells. d) Ventilation is the movement of air in and out of the respiratory tract.

D. Ventilation is the actual movement of air in and out of the respiratory tract. Respiration is the exchange of oxygen and CO2 between atmospheric air and the blood and between the blood and the cells. Therefore, options A, B, and C are incorrect.

A client arrives at the physician's office stating dyspnea; a productive cough for thick, green sputum; respirations of 28 breaths/minute, and a temperature of 102.8° F. The nurse auscultates the lung fields, which reveal poor air exchange in the right middle lobe. The nurse suspects a right middle lobe pneumonia. To be consistent with this anticipated diagnosis, which sound, heard over the chest wall when percussing, is anticipated? a) Tympanic b) Resonant c) Hyperresonant d) Dull

D. a dull percussed sound, heard over the chest wall, is indicative of little or no air movement in that area of the lung. Lung consolidation such as in pneumonia or fluid accumulation produces the dull sound. A tympanic sound is a high-pitched sound commonly heard over the stomach or bowel. A resonant sound is noted over normal lung tissue. A hyper resonant sound is an abnormal lower pitched sound that occurs when free air exists in disease processes such as pneumothorax

Which ventilation-perfusion ratio is exhibited in a client diagnosed with a pulmonary embolus?

Dead space A dead space exists when ventilation exceeds perfusion (high ventilation-perfusion ratios). An example of a dead space is a pulmonary embolus, pulmonary infarction, and cardiogenic shock. A low ventilation-perfusion ratio exists in pneumonia or with a mucus plug. A silent unit occurs in pneumothorax or acute respiratory distress syndrome.

Which ventilation-perfusion ratio is exhibited by a pulmonary emboli?

Dead space Correct Explanation: When ventilation exceeds perfusion a dead space exists. An example of a dead space is pulmonary emboli. A low ventilation-perfusion ratio exists in pneumonia or with a mucus plug. A silent unit occurs in pneumothorax or ARDS

A gerontologic nurse is analyzing the data from a client's focused respiratory assessment. The nurse is aware that the amount of respiratory dead space increases with age. What is the effect of this physiologic change?

Decreased diffusion capacity for oxygen The amount of respiratory dead space increases with age. Combined with other changes, this results in a decreased diffusion capacity for oxygen with increasing age, producing lower oxygen levels in the arterial circulation. Decreased shunting and increased ventilation do not occur with age.

The nurse working on a gerontology unit admits a 77-year-old with recent shortness of breath. The nurse knows that the amount of respiratory dead space increases with age. What do these changes result in? Increased ventilation Decreased diffusion capacity for oxygen Increased diffusion of gases Decreased shunting of blood

Decreased diffusion capacity for oxygen The amount of respiratory dead space increases with age. These changes result in a decreased diffusion capacity for oxygen with increasing age, producing lower oxygen levels in the arterial circulation. Shunting does not typically decrease and ventilation does not increase.

When the nurse is assessing the older adult patient, what gerontological changes in the respiratory system should the nurse be aware of? (Select all that apply)

Decreased gauge reflex Increased presence of collagen in the alveolar walls Decreased presence of mucus

The instructor in the anatomy and physiology class is talking about alveolar respiration. What would the instructor tell the class is the main purpose of alveolar respiration?

Determines amount of CO2 in the body Explanation: Alveolar respiration determines the amount of CO2 in the body. It does not determine the amount of O2, bicarbonate, or PaO2 in the body.

The instructor in the anatomy and physiology class is talking about alveolar respiration. What would the instructor tell the class is the main purpose of alveolar respiration? a) Determines amount of CO2 in the body b) Determines amount of bicarbonate in the body c) Determines amount of oxygen in the body d) Determines amount of PaO2 in the body

Determines amount of CO2 in the body Explanation: Alveolar respiration determines the amount of CO2 in the body. It does not determine the amount of O2, bicarbonate, or PaO2 in the body.

The nurse enters the room of a client who is being monitored with pulse oximetry. Which of the following factors may alter the oximetry results? Increased temperature of the room Reduced lighting in the room Placement of the probe on an earlobe Diagnosis of peripheral vascular disease

Diagnosis of peripheral vascular disease Pulse oximetry is a noninvasive method of monitoring oxygen saturation of hemoglobin. A probe is placed on the fingertip, forehead, earlobe, or bridge of nose. Inaccuracy of results may be from anemia, bright lights, shivering, nail polish, or peripheral vascular disease.

The nurse enters the room of a client who is being monitored with pulse oximetry. Which of the following factors may alter the oximetry results?

Diagnosis of peripheral vascular disease Explanation: Pulse oximetry is a noninvasive method of monitoring oxygen saturation of hemoglobin. A probe is placed on the fingertip, forehead, earlobe, or bridge of nose. Inaccuracy of results may be from anemia, bright lights, shivering, nail polish, or peripheral vascular disease

For air to enter the lungs (process of ventilation), the intrapulmonary pressure must be less than atmospheric pressure so air can be pulled inward. Select the movement of respiratory muscles that makes this happen during inspiration. Intercostals muscles relax to allow for expansion. Anteroposterior rib diameter decreases. Lungs are pulled up and pushed back against the thoracic cage. Diaphragm contracts and elongates the chest cavity.

Diaphragm contracts and elongates the chest cavity. The diaphragm contracts during inspiration and pulls the lungs in a downward and forward direction. The abdomen appears to enlarge because the abdominal contents are being compressed by the diaphragm. With inspiration, the diaphragmatic pull elongates the chest cavity, and the external intercostal muscles (located between and along the lower borders of the ribs) contract to raise the ribs, which expands the anteroposterior diameter. The effect of these movements is to decrease the intrapulmonary pressure.

For air to enter the lungs (process of ventilation), the intrapulmonary pressure must be less than atmospheric pressure so air can be pulled inward. Select the movement of respiratory muscles that makes this happen during inspiration.

Diaphragm contracts and elongates the chest cavity. Explanation: The diaphragm contracts during inspiration and pulls the lungs in a downward and forward direction. The abdomen appears to enlarge because the abdominal contents are being compressed by the diaphragm. With inspiration, the diaphragmatic pull elongates the chest cavity, and the external intercostal muscles (located between and along the lower borders of the ribs) contract to raise the ribs, which expands the anteroposterior diameter. The effect of these movements is to decrease the intrapulmonary pressure.

Which of the following clinical manifestations should a nurse monitor for during a pulmonary angiography, which indicates an allergic reaction to the contrast medium? Hematoma Urge to cough Absent distal pulses Difficulty in breathing

Difficulty in breathing Nurses must determine if the client has any allergies, particularly to iodine, shellfish, or contrast dye. During the procedure, the nurse should check for signs and symptoms of allergic reactions to the contrast medium, such as itching, hives, or difficulty in breathing. The nurses inspects for hematoma, absent distal pulses, after the procedure. When the contrast medium is infused, an urge to cough is often a sensation experienced by the client.

Which of the following clinical manifestations should a nurse monitor for during a pulmonary angiography, which indicates an allergic reaction to the contrast medium?

Difficulty in breathing Explanation: Nurses must determine if the client has any allergies, particularly to iodine, shellfish, or contrast dye. During the procedure, the nurse should check for signs and symptoms of allergic reactions to the contrast medium, such as itching, hives, or difficulty in breathing. The nurses inspects for hematoma, absent distal pulses, after the procedure. When the contrast medium is infused, an urge to cough is often a sensation experienced by the client.

The exchange of oxygen and carbon dioxide from the alveoli in the blood occurs by

Diffusion

A nurse is preparing a client for bronchoscopy. Which instruction should the nurse give to the client? Do not walk after the procedure for 4 to 6 hours. Do not cough after the procedure until you are walking. Do not talk for 2 hours before the procedure. Do not eat or drink for 6 hours before the procedure.

Do not eat or drink for 6 hours before the procedure. Bronchoscopy involves visualization of the trachea and bronchial tree. To prevent aspiration of stomach contents into the lungs, the nurse should instruct the client not to eat or drink anything for approximately 6 hours before the procedure. It isn't necessary for the client to avoid walking, talking, or coughing.

A nurse is preparing a client for bronchoscopy. Which instruction should the nurse give to the client?

Do not eat or drink for 6 hours before the procedure. Explanation: Bronchoscopy involves visualization of the trachea and bronchial tree. To prevent aspiration of stomach contents into the lungs, the nurse should instruct the client not to eat or drink anything for approximately 6 hours before the procedure. It isn't necessary for the client to avoid walking, talking, or coughing.

A client arrives at the physician's office stating dyspnea; a productive cough for thick, green sputum; respirations of 28 breaths/minute, and a temperature of 102.8° F. The nurse auscultates the lung fields, which reveal poor air exchange in the right middle lobe. The nurse suspects a right middle lobe pneumonia. To be consistent with this anticipated diagnosis, which sound, heard over the chest wall when percussing, is anticipated?

Dull Explanation: A dull percussed sound, heard over the chest wall, is indicative of little or no air movement in that area of the lung. Lung consolidation such as in pneumonia or fluid accumulation produces the dull sound. A tympanic sound is a high-pitched sound commonly heard over the stomach or bowel. A resonant sound is noted over normal lung tissue. A hyper resonant sound is an abnormal lower pitched sound that occurs when free air exists in disease processes such as pneumothorax.

During a pre-admission assessment,for what diagnosis would the nurse expect to find decreased tactile fremitus and hyperresonant percussion sounds?

Emphysema

A 68-year-old male patient has been admitted to the surgical unit from the PACU after surgical repair of an inguinal hernia. When performing the patient's admission assessment, the nurse notes that the patient has a barrel chest. This assessment finding should suggest to the nurse that the patient may have a history of what health problem?

Emphysema Barrel chest occurs as a result of lung hyperinflation, as in emphysema. There is an increase in the anteroposterior diameter of the thorax so that it approximates a 1:1 ratio. This assessment finding is not associated with asthma, bronchitis, or tuberculosis.

High or increased compliance occurs in which disease process?

Emphysema Explanation: High or increased compliance occurs if the lungs have lost their elasticity and the thorax is overdistended, as in emphysema. Conditions associated with decreased compliance include pneumothorax, pleural effusion, and acute respiratory distress syndrome (ARDS).

A patient is being educated in the use of incentive spirometry prior to having a surgical procedure. What should the nurse be sure to include in the education?

Encourage the patient to take approximately 10 breaths per hour, while awake. Explanation: The patient should be instructed to perform the procedure approximately 10 times in succession, repeating the 10 breaths with the spirometer each hour during waking hours. The patient should assume a semi-Fowler's position or an upright position before initiating therapy, not be supine. Coughing during and after each session is encouraged, not discouraged. The patient should Splint the incision when coughing postoperatively. The patient should still use the spirometer when in pain.

A patient with sinus congestion points to a location on the inside of his eye as the area of pain. The nurse documents that the patient is complaining of pain in which sinus?

Ethmoid Explanation: The ethmoidal sinuses are located between the eyes and behind the nose. Inflammation and swelling block drainage into the nose; eventually an infection results.

The Family Nurse Practitioner is performing a physical examination of a client. The Nurse Practitioner examines the client's anterior, posterior, and lateral chest walls. What is the Nurse Practitioner assessing?

Evidence of muscle weakness Explanation: The nurse examines the anterior, posterior, and lateral chest walls to check for lesions, symmetry, deformities, skin color, and evidence of muscle weakness or weight loss. The nurse examines the posterior pharynx and tonsils with a tongue blade and light to detect any difficulty in swallowing or hoarseness. The nurse inspects and gently palpates the trachea to assess the placement and deviation from the midline.

A healthcare provider requests a study of diaphragmatic motion because of suspected pathology. What diagnostic test will the nurse prepare the patient for?

Fluoroscopy

A patient with sinus congestion reports discomfort when the nurses is palpating the supraorbital ridges. The nurse is aware that the PT is referring to which sinus?

Frontal

A client with sinus congestion complains of discomfort when the nurse is palpating the supraorbital ridges. What sinus is the client referring? Frontal Sphenoidal Maxillary Ethmoidal

Frontal The nurse may palpate the frontal and maxillary sinuses for tenderness. Using the thumbs, the nurse applies gentle pressure in an upward fashion at the supraorbital ridges (frontal sinuses) and in the cheek area adjacent to the nose (maxillary sinuses). The ethmoidal sinuses are located between the nose and eyes. The sphenoidal sinuses are behind the nose between the eyes.

A client with sinus congestion complains of discomfort when the nurse is palpating the supraorbital ridges. What sinus is the client referring?

Frontal Explanation: The nurse may palpate the frontal and maxillary sinuses for tenderness. Using the thumbs, the nurse applies gentle pressure in an upward fashion at the supraorbital ridges (frontal sinuses) and in the cheek area adjacent to the nose (maxillary sinuses). The ethmoidal sinuses are located between the nose and eyes. The sphenoidal sinuses are behind the nose between the eyes.

A 4 year-old client has had recurrent episodes of bronchitis during the winter months. The client spends a significant amount of time outdoors enjoying the snow and breathes primarily through the mouth. What suggestion would the nurse most likely make to the client's mother?

Help the client learn to breathe through her nose.

A 4 year-old client has had recurrent episodes of bronchitis during the winter months. The client spends a significant amount of time outdoors enjoying the snow and breathes primarily through the mouth. What suggestion would the nurse most likely make to the client's mother?

Help the client learn to breathe through her nose. The vascular and ciliated mucous lining of the nasal cavities warms and humidifies inspired air. The lining of the sinuses is continuous with the mucous-membrane lining of the nasal cavity. Mucus traps particles that cilia sweep toward the pharynx. Immunoglobulin A (IgA) antibodies in the mucus protect the lower respiratory tract from infection. The turbinates are bones that change the flow of inspired air to moisturize and warm it better.

You are assessing the respiratory system of a client just admitted to your unit. What do you know to assess in addition to the physical and functional issues related to breathing?

How these issues affect the client's quality of life Explanation: Assessment of the respiratory system includes obtaining information about physical and functional issues related to breathing. It also means clarifying how these issues may affect the client's quality of life.

You are assessing the respiratory system of a client just admitted to your unit. What do you know to assess in addition to the physical and functional issues related to breathing?

How these issues affect the client's quality of life Explanation: Assessment of the respiratory system includes obtaining information about physical and functional issues related to breathing. It also means clarifying how these issues may affect the client's quality of life. Therefore, options B, C, and D are incorrect

The nurse is caring for a client with a lower respiratory tract infection. When planning a focused respiratory assessment, the nurse should know that this type of infection most often causes what?

Impaired gas exchange The lower respiratory tract consists of the lungs, which contain the bronchial and alveolar structures needed for gas exchange. A lower respiratory tract infection does not collapse bronchial structures or close the bronchial tree. An infection does not cause necrosis of lung tissues.

A nurse is concerned that a client may develop postoperative atelectasis. Which nursing diagnosis would be most appropriate if this complication occurs?

Impaired gas exchange Explanation: Airflow is decreased with atelectasis, which is a bronchial obstruction from collapsed lung tissue. If there is an obstruction, there is limited or no gas exchange in this area. Impaired gas exchange is thus the most likely nursing diagnosis with atelectasis.

The nurse has instructed a client on how to perform pursed-lip breathing. The nurse recognizes the purpose of this type of breathing is to accomplish which result?

Improve oxygen transport; induce a slow, deep breathing pattern; and assist the client to control breathing Pursed-lip breathing, which improves oxygen transport, helps induce a slow, deep breathing pattern and assists the client to control breathing, even during periods of stress. This type of breathing helps prevent airway collapse secondary to loss of lung elasticity in emphysema.

You are performing pulmonary function studies on clients in the clinic. What position do you know a client should be in to have maximum lung capacities and volumes? In the standing position Resting the head on a pillow Lying on the unaffected side Lying flat on the back

In the standing position The maximum lung capacities and volumes are best achieved when the client is sitting or standing. Lying on the unaffected side and resting the head on the pillow are the positions recommended for thoracentesis. Lying flat on the back is not applicable for achieving maximum lung capacities and volumes.

The nurse is caring for a critically ill client in the ICU. The nurse documents the client's respiratory rate as bradypnea. The nurse recognizes that bradypnea is associated with which condition? Pneumonia Increased intracranial pressure Pulmonary edema Metabolic acidosis

Increased intracranial pressure Bradypnea is associated with increased intracranial pressure, brain injury, and drug overdose. Respirations are slower than the normal rate (<10 breaths/min), with normal depth and regular rhythm. Tachypnea is commonly seen in clients with pneumonia, pulmonary edema, and metabolic acidosis.

The nurse is caring for a critically ill client in the ICU. The nurse documents the client's respiratory rate as bradypnea. The nurse recognizes that bradypnea is associated with which condition?

Increased intracranial pressure Explanation: Bradypnea is associated with increased intracranial pressure, brain injury, and drug overdose. Respirations are slower than the normal rate (<10 breaths/min), with normal depth and regular rhythm. Tachypnea is commonly seen in clients with pneumonia, pulmonary edema, and metabolic acidosis.

What would the instructor tell the students purulent fluid indicates? Inflammation Cancer Infection Heart failure

Infection A small amount of fluid lies between the visceral and parietal pleurae. When excess fluid or air accumulates, the physician aspirates it from the pleural space by inserting a needle into the chest wall. This procedure, called thoracentesis, is performed with local anesthesia. Thoracentesis also may be used to obtain a sample of pleural fluid or a biopsy specimen from the pleural wall for diagnostic purposes such as a culture, sensitivity, or microscopic examination. Purulent fluid is the recommended diagnosis for infection. Serous fluid may be associated with cancer, inflammatory conditions, or heart failure.

What would the instructor tell the students purulent fluid indicates?

Infection Explanation: A small amount of fluid lies between the visceral and parietal pleurae. When excess fluid or air accumulates, the physician aspirates it from the pleural space by inserting a needle into the chest wall. This procedure, called thoracentesis, is performed with local anesthesia. Thoracentesis also may be used to obtain a sample of pleural fluid or a biopsy specimen from the pleural wall for diagnostic purposes such as a culture, sensitivity, or microscopic examination. Purulent fluid is the recommended diagnosis for infection. Serous fluid may be associated with cancer, inflammatory conditions, or heart failure.

A client is receiving platelets. In order to decreased the risk of circulatory overload in this client, the nurse should do which of the following? a) Infuse each unit over 30-60 minutes per client tolerance. b) Flush the intravenous line with a liter of saline between units. c) Monitor vital signs closely before transfusion and once per shift. d) Administer each unit slowly over 3-4 hours.

Infuse each unit over 30-60 minutes per client tolerance. Explanation: Infuse each unit of FFP over 30-60 minutes per client tolerance. Platelet clumping will occur if administered too slowly. Vital signs should be monitored before and throughout the ... (more) Infuse each unit of FFP over 30-60 minutes per client tolerance. Platelet clumping will occur if administered too slowly. Vital signs should be monitored before and throughout the transfusion, not just once per shift. A liter of saline is too large an amount to flush the intravenous line and would contribute to fluid overload

The nurse is instructing the PT on the collection of sputum specimen. What should be included in the instructions? Select all that apply

Initially, clear the nose and the throat Take a few deep breaths before coughing Use diaphragmatic contractions to aid in the expulsion of sputum

A pediatrician diagnosed a child with swollen and inflamed adenoids. The nurse practitioner confirmed the diagnosis by: Inspecting the posterior region of the epiglottis. Inspecting the roof of the nasopharynx. Examining the base of the oropharynx. Palpating the throat above the cricoid cartilage.

Inspecting the roof of the nasopharynx. The adenoids are clusters of lymph tissue located between the back of the nose and the back of the throat in the nasopharynx. The adenoids are usually inspected by using a special mirror. They cannot be seen by looking directly into the mouth.

The maximum volume of air that can be exhaled after a normal inhalation is known as

Inspiratory reserve volume

Which respiratory volume is the maximum volume of air that can be inhaled after a normal exhalation?

Inspiratory reserve volume Explanation: Inspiratory reserve volume is normally 3000 mL. Tidal volume is the volume of air inhaled and exhaled with each breath. Expiratory reserve volume is the maximum volume of air that can be exhaled forcibly after a normal exhalation. Residual volume is the volume of air remaining in the lungs after a maximum exhalation.

The nurse is assisting a client with postural drainage. Which of the following demonstrates correct implementation of this technique?

Instruct the client to remain in each position of the postural drainage sequence for 10 to 15 minutes. Explanation: Postural drainage is usually performed two to four times daily, before meals (to prevent nausea, vomiting, and aspiration) and at bedtime. Prescribed bronchodilators, water, or saline may be nebulized and inhaled before postural drainage to dilate the bronchioles, reduce bronchospasm, decrease the thickness of mucus and sputum, and combat edema of the bronchial walls. The nurse instructs the client to remain in each position for 10 to 15 minutes and to breathe in slowly through the nose and out slowly through pursed lips to help keep the airways open so that secretions can drain while in each position. If the sputum is foul-smelling, it is important to perform postural drainage in a room away from other patients or family members. (Deodorizers may be used to counteract the odor. Because aerosol sprays can cause bronchospasm and irritation, they should be used sparingly and with caution.)

What happens to the diaphragm during inspiration? It contracts and flattens. It relaxes and flattens. It relaxes and raises. It contracts and raises.

It contracts and flattens. During inspiration, the diaphragm contracts and flattens, which expands the thoracic cage and increases the thoracic cavity.

A client is scheduled to have excess pleural fluid aspirated with a needle in order to relieve her dyspnea. The client inquires about the normal function of pleural fluid. What should the nurse describe?

It lubricates the movement of the thorax and lungs. The visceral pleura cover the lungs; the parietal pleura line the thorax. The visceral and parietal pleura and the small amount of pleural fluid between these two membranes serve to lubricate the thorax and lungs and permit smooth motion of the lungs within the thoracic cavity with each breath. The pleura do not allow full expansion of the lungs, prevent the lungs from collapsing, or limit lung expansion within the thoracic cavity.

A patient diagnosed with diabetic ketoacidosis would be expected to have which type of respiratory pattern?

Kussmaul respirations Explanation: Kussmaul respirations are seen in patients with diabetic ketoacidosis. In Cheyne-Stokes respiration, rate and depth increase, then decrease until apnea occurs. Biot's respiration is characterized by periods of normal breathing (3 to 4 breaths) followed by a varying period of apnea (usually 10 to 60 seconds).

During a pulmonary assessment, the nurse observes the chest for configuration. She identifies the findings as normal. Which of the following would be consistent with normal assessment? a) Lateral diameter greater than anteroposterior diameter b) Lateral diameter less than anteroposterior diameter c) Anteroposterior diameter that equals the lateral diameter d) Elevation of the scapula with the lateral diameter unaffected

Lateral diameter greater than anteroposterior diameter Explanation: Inspecting the thorax is part of assessment of the respiratory system. Normally, the ratio of the anteroposterior diameter to the lateral diameter is 1:2. Chest deformities are associated with respiratory disease

A client presents to the ED reporting severe coughing episodes. The client states that "the episodes are more intense at night." The nurse should suspect which of the following conditions based on the client's primary report?

Left-sided heart failure

The pulmonary circulation is considered a

Low-pressure system

A nurse is performing a physical assessment on a client who has a history of a respiratory infection. Which documentation, completed by the nurse, indicates the resolution of the infection? Select all that apply. Lung fields documented as clear in the bases. Decreased fremitus when the client speaks "99." Bronchovesicular sounds heard over the upper lung fields. Dull sounds percussed over the lung tissue. Palpable vibrations over the chest wall when the client speaks.

Lung fields documented as clear in the bases. Palpable vibrations over the chest wall when the client speaks. Decreased fremitus when the client speaks "99." Bronchovesicular sounds heard over the upper lung fields. To determine if the client's respiratory infection has resolved, the nurse should assess the client's normal respiratory status. Lungs will return to clear breath sounds. Palpable vibrations will be felt, as there is no blockage in the transmission. A client with consolidation of a lobe of the lung from pneumonia has increased tactile fremitus over that lobe. A decreased fremitus would indicate resolution of infection Bronchovesicular sounds will be noted over the upper lung fields. An increased fremitus is noted as the client speaks "99." Dull percussed sounds indicate an area of consolidation.

A nurse is performing a physical assessment on a client who has a history of a respiratory infection. Which documentation, completed by the nurse, indicates the resolution of the infection? Select all that apply.

Lung fields documented as clear in the bases. Palpable vibrations over the chest wall when the client speaks. Decreased fremitus when the client speaks "99." Bronchovesicular sounds heard over the upper lung fields. Explanation: To determine if the client's respiratory infection has resolved, the nurse should assess the client's normal respiratory status. Lungs will return to clear breath sounds. Palpable vibrations will be felt, as there is no blockage in the transmission. A client with consolidation of a lobe of the lung from pneumonia has increased tactile fremitus over that lobe. A decreased fremitus would indicate resolution of infection Bronchovesicular sounds will be noted over the upper lung fields. An increased fremitus is noted as the client speaks "99." Dull percussed sounds indicate an area of consolidation.

The nurse is performing a respiratory assessment for a client suspected of having a pleural effusion. Which assessment finding is most consistent with the suspicion of a pleural effusion? Select all that apply.

Lung fields dull to percussion/ Absence of breath sounds/ Pleural friction rub noted. Rational: Assessment findings consistent with a pleural effusion include affected lung fields being dull to percussion and absence of breath sounds. A pleural friction rub may also be present. The other listed signs are not typically associated with a pleural effusion.

A client has a nursing diagnosis of "ineffective airway clearance" as a result of excessive secretions. An appropriate outcome for this client would be which of the following?

Lungs are clear on auscultation. Explanation: Assessment of lung sounds includes auscultation for airflow through the bronchial tree. The nurse evaluates for fluid or solid obstruction in the lung. When airflow is decreased, as with fluid or secretions, adventitious sounds may be auscultated. Often crackles are heard with fluid in the airways

A physician has ordered that a client with suspected lung cancer undergo magnetic resonance imaging (MRI). The nurse explains the benefits of this study to the client. What is the reason the client with suspected lung cancer would undergo magnetic resonance imaging (MRI)?

MRI can view soft tissues and can help stage cancers. Explanation: MRI uses magnetic fields and radiofrequency signals to produce a detailed diagnostic image. MRI can visualize soft tissues, characterize nodules, and help stage carcinomas. The other options describe different studies.

A client is undergoing a thoracentesis. Which nursing action will the nurse perform following the procedure?

Maintain the client on bed rest for several hours. The client remains on bed rest and usually lies on the unaffected side for at least 1 hour to promote expansion of the lung on the affected side. The client should not ambulate following the procedure, but remain on bed rest for at least an hour. The client usually lies on the unaffected side to promote expansion of the lung on the affected side. Clients should cough forcefully when expectorating sputum for a specimen, but not following a thoracentesis.

Why is it important for a nurse to provide required information and appropriate explanations of diagnostic procedures to patients with respiratory disorders? a) Manage decreased energy levels b) Manage respiratory distress c) Aid the caregivers of the client d) Ensure adequate rest periods

Manage decreased energy levels Explanation: In addition to the nursing management of individual tests, patients with respiratory disorders require informative and appropriate explanations of any diagnostic procedures they will experience. Nurses must remember that for many of these patients, breathing may in some way be compromised. Energy levels may be decreased. For that reason, explanations should be brief yet complete and may need to be repeated later after a rest period. The nurse must also ensure adequate rest periods before and after the procedures. After invasive procedures, the nurse must carefully assess for signs of respiratory distress

A nurse has performed tracheal suctioning on a client who experienced increasing dyspnea prior to a procedure. When applying the nursing process, how can the nurse best evaluate the outcomes of this intervention?

Measure the client's oxygen saturation. The client's response to suctioning is usually determined by performing chest auscultation and by measuring the client's oxygen saturation. FET, incentive spirometry, and percussion are not normally used as evaluative techniques.

For a client who has a chest tube connected to a closed water-seal drainage system, the nurse should include which action in the care plan?

Measuring and documenting the drainage in the collection chamber Explanation: The nurse should regularly measure and document the amount of chest tube drainage to detect abnormal drainage patterns, such as may occur with a hemorrhage (if excessive) or a blockage (if decreased). Continuous bubbling in the water-seal chamber indicates a leak in the closed chest drainage system, which must be corrected. The nurse should keep the collection chamber below chest level to allow fluids to drain into it. The nurse shouldn't strip chest tubes because doing so may traumatize the tissue or dislodge the tube.

The nurse is caring for a client who is experiencing mild shortness of breath during the immediate postoperative period, with oxygen saturation readings between 89% and 91%. What method of oxygen delivery is most appropriate for the client's needs?

Nasal cannula A nasal cannula is used when the client requires a low to medium concentration of oxygen for which precise accuracy is not essential. The Venturi mask is used primarily for clients with COPD because it can accurately provide an appropriate level of supplemental oxygen, thus avoiding the risk of suppressing the hypoxic drive. The client's respiratory status does not require a partial- or non-rebreathing mask.

Upon palpation of the sinus area, what would the nurse identify as a normal finding? Tenderness during palpation Pain sensation behind the eyes Light not going through the sinus cavity No sensation during palpation

No sensation during palpation Sinus assessment involves using the thumbs to apply gentle pressure in an upward fashion at the sinuses. Tenderness suggests inflammation. The sinuses can be inspected by transillumination, where a light is passed through the sinuses. If the light fails to penetrate, the cavity contains fluid.

Upon palpation of the sinus area, what would the nurse identify as a normal finding?

No tenderness during palpation Explanation: Sinus assessment involves using the thumbs to apply gentle pressure in an upward fashion at the sinuses. Tenderness suggests inflammation. The sinuses can be inspected by transillumination, where a light is passed through the sinuses. If the light fails to penetrate, the cavity contains fluid.

A nurse practitioner diagnosed a patient with an infection in the maxillary sinuses. Select the area that the nurse palpated to make that diagnosis.

On the cheeks below the eyes To palpate the maxillary sinuses, the nurse should apply gentle pressure in the cheek area below the eyes, adjacent to the nose.

The nurse is performing an assessment for a PT with congestive heart failure. The nurse asks if the PT has difficulty breathing in any position other than upright. What is the nurse referring to?

Orthopnea

When assessing a client, which adaptation indicates the presence of respiratory distress?

Orthopnea Explanation: Orthopnea is the inability to breathe easily except when upright. This positioning can mean while in bed and propped with a pillow or sitting in a chair. If a client cannot breathe easily while lying down, there is an element of respiratory distress.

You are caring for a client with multiple myeloma. Why would it be important to assess this client for fractures? a) Osteopathic tumors destroy bone causing fractures. b) Osteolytic activating factor weakens bones producing fractures. c) Osteoclasts break down bone cells so pathologic fractures occur. d) Osteosarcomas form producing pathologic fractures.

Osteoclasts break down bone cells so pathologic fractures occur. Explanation: The abnormal plasma cells proliferate in the bone marrow, where they release osteoclast-activating factor. This in turn causes osteoclasts to break down bone cells, resulting in increased blood calcium and pathologic fractures. The plasma cells also form single or multiple osteolytic (bone-destroying) tumors that produce a 'punched-out' or 'honeycombed' appearance in bones such as the spine, ribs, skull, pelvis, femurs, clavicles, and scapulae. Weakened vertebrae lead to compression of the spine accompanied by significant pain. Options A, C, and D are distractors for this question

The nurse is explaining the safe and effective administration of nasal spray to a client with seasonal allergies. What information is most important to include in this teaching?

Overuse of nasal spray may cause rebound congestion. Explanation: The use of topical decongestants is controversial because of the potential for a rebound effect. The client should hold his or her head back for maximal distribution of the spray. Only the client should use the bottle.

A client has been receiving 100% oxygen therapy by way of a nonrebreather mask for several days. Now the client complains of tingling in the fingers and shortness of breath, is extremely restless, and describes a pain beneath the breastbone. What should the nurse suspect?

Oxygen toxicity Oxygen toxicity may occur when too high a concentration of oxygen (greater than 50%) is administered for an extended period (longer than 48 hours) (Urden, Stacy, & Lough, 2014). Signs and symptoms of oxygen toxicity include substernal discomfort, paresthesias, dyspnea, restlessness, fatigue, malaise, progressive respiratory difficulty, refractory hypoxemia, alveolar atelectasis, and alveolar infiltrates evident on chest x-rays.

Which diagnostic is more accurate in detecting malignancies than a CT scan?

PET scan Explanation: A PET scan is more accurate in detecting malignancies than a CT scan, and it has equivalent accuracy in detecting malignant nodules when compared with invasive procedures such as thorascopy. The gallium scan is used to stage bronchogenic cancer and document tumor regression after chemotherapy or radiation. An MRI is used to characterize pulmonary nodules, to help stage bronchogenic carcinoma, and to evaluate inflammatory activity in interstitial lung disease. Pulmonary angiography is used to investigate thromboembolic disease of the lungs

The nurse is reviewing the blood gas results for a PT with pneumonia. What arterial blood gas measurement best reflects the adequacy of alveolar ventilation?

PaCo2

The nurse is admitting a patient with COPD. The decrease of what substance in the blood gas analysis would indicate to the nurse that the patient is experiencing hypoxemia?

PaO2

The symbol used to identify the partial pressure of oxygen is?

PaO2

A client admitted to the hospital following a motor vehicle crash has suffered a flail chest. The nurse assesses the client for what most common clinical manifestation of flail chest?

Paradoxical chest movement Explanation: During inspiration, as the chest expands, the detached part of the rib segment (flail segment) moves in a paradoxical manner (pendelluft movement) in that it is pulled inward during inspiration, reducing the amount of air that can be drawn into the lungs. Upon expiration, because the intrathoracic pressure exceeds atmospheric pressure, the flail segment bulges outward, impairing the client's ability to exhale. The mediastinum then shifts back to the affected side. This paradoxical action results in increased dead space, a reduction in alveolar ventilation, and decreased compliance.

A client is receiving supplemental oxygen. When determining the effectiveness of oxygen therapy, which arterial blood gas value is most important?

Partial pressure of arterial oxygen (PaO2) Explanation: The most significant and direct indicator of the effectiveness of oxygen therapy is the PaO2 value. Based on the PaO2 value, the nurse may adjust the type of oxygen delivery (cannula, Venturi mask, or mechanical ventilator), flow rate, and oxygen percentage. The other options reflect the client's ventilation status, not oxygenation. The pH, HCO3-, and PaCO2

The nurse is discussing activity management with a client who is postoperative following thoracotomy. What instructions should the nurse give to the client regarding activity immediately following discharge?

Perform shoulder exercises five times daily. The nurse emphasizes the importance of progressively increased activity. The nurse also instructs the client on the importance of performing shoulder exercises five times daily. The client should ambulate with limits and realize that the return of strength will likely be gradual and likely will not include weight lifting or lengthy walks.

A client has been diagnosed with pulmonary hypertension, in which the capillaries in the alveoli are squeezed excessively. The nurse should recognize a disturbance in what aspect of normal respiratory function? A. Diffusion B. Perfusion

Perfusion Explanation: Perfusion is influenced by alveolar pressure. The pulmonary capillaries are sandwiched between adjacent alveoli and, if the alveolar pressure is sufficiently high, the capillaries are squeezed. This does not constitute a disturbance in ventilation (air movement), diffusion (gas exchange), or acid-base balance.

A patient comes to the emergency department reporting a knifelike pain when taking. Deep breath. What does this type of pain likely indicate to the nurse?

Pleurisy

A patient comes to the emergency department complaining of a knifelike pain when taking a deep breath. What does this type of pain likely indicate to the nurse? Pleurisy Bronchogenic carcinoma Lung infarction Bacterial pneumonia

Pleurisy Pleuritic pain from irritation of the parietal pleura is sharp and seems to "catch" on inspiration; patients often describe it as being "like the stabbing of a knife." In carcinoma, the pain may be dull and persistent because the cancer has invaded the chest wall, mediastinum, or spine.

A patient describes his chest pain as knife-like on inspiration. Which of the following is the most likely diagnosis?

Pleurisy Explanation: Pleuritic pain from irritation of the parietal pleura is sharp and seems to "catch" on inspiration. Some patients describe the pain as being "stabbed by a knife." Chest pain associated with the other conditions may be dull, aching, and persistent.

A patient comes to the emergency department complaining of a knifelike pain when taking a deep breath. What does this type of pain likely indicate to the nurse?

Pleurisy Explanation: Pleuritic pain from irritation of the parietal pleura is sharp and seems to "catch" on inspiration; patients often describe it as being "like the stabbing of a knife." In carcinoma, the pain may be dull and persistent because the cancer has invaded the chest wall, mediastinum, or spine.

The client is planned to have a splenectomy. The nurse should prepare which medication to administer to this client? a) Immunoglobulin G (IgG) b) Factor VIII c) Pneumococcal vaccine d) Aspirin

Pneumococcal vaccine Explanation: Without a spleen, the client's risk of infection is greatly increased. The pneumococcal vaccine should be administered, preferable before splenectomy. Aspirin should not be admin ... (more) Without a spleen, the client's risk of infection is greatly increased. The pneumococcal vaccine should be administered, preferable before splenectomy. Aspirin should not be administered due to the increased risk of bleeding. IgG is administered to client with increased chance of bacterial infections but is not routinely given to client undergoing splenectomy, as is the pneumococcal vaccine. Factor VII is given to treat bleeding disorders.

Which nursing intervention is most appropriate for a client with multiple myeloma? a) Balancing rest and activity b) Monitoring respiratory status c) Preventing bone injury d) Restricting fluid intake

Preventing bone injury Explanation: When caring for a client with multiple myeloma, the nurse should focus on relieving pain, preventing bone injury and infection, and maintaining hydration. Monitoring respiratory status and balancing rest and activity are appropriate interventions for any client. To prevent such complications as pyelonephritis and renal calculi, the nurse should keep the client well hydrated — not restrict his fluid intake.

The nurse is taking a respiratory history for a patient who has come into the clinic with a chronic cough. What information should the nurse obtain from this patient? Select all that apply

Previous history of lung disease in the patient or family Occupational and environmental influences Previous history of smoking

The nurse is taking a respiratory history for a patient who has come into the clinic with a chronic cough. What information should the nurse obtain from this patient? (Select all that apply.)

Previous history of lung disease in the patient or family Occupational and environmental influences Previous history of smoking Explanation: Risk factors associated with respiratory disease include smoking, exposure to allergens and environmental pollutants, and exposure to certain recreational and occupational hazards. Financial ability and social support are not pertinent to a chronic cough.

What is the primary function of the larynx? Preventing infection Facilitating coughing Protecting the lower airway from foreign objects Producing sound

Producing sound The larynx, or voice box, is a cartilaginous framework between the pharynx and trachea. Its primary function is to produce sound. While the larynx assists in protecting the lower airway, this is mainly the function of the epiglottis. Facilitating coughing is a secondary function of the larynx. Preventing infection is the main function of the tonsils and adenoids.

What is the primary function of the larynx?

Producing sound Explanation: The larynx, or voice box, is a cartilaginous framework between the pharynx and trachea. Its primary function is to produce sound. While the larynx assists in protecting the lower airway, this is mainly the function of the epiglottis. Facilitating coughing is a secondary function of the larynx. Preventing infection is the main function of the tonsils and adenoids.

A client has been discharged home after thoracic surgery. The home care nurse performs the initial visit and finds the client discouraged and saddened. The client states, "I am recovering so slowly. I really thought I would be better by now." What nursing action should the nurse prioritize?

Provide emotional support to the client and family. The recovery process may take longer than the client had expected, and providing support to the client is an important task for the home care nurse. It is not necessary, based on this scenario, to schedule a visit with the physician within 24 hours, or to get a referral to a psychiatrist or a social worker.

The clinical finding of pink, frothy sputum may be an indication of which condition? Pulmonary edema Bronchiectasis Lung abscess Infection

Pulmonary edema Profuse frothy, pink material, often welling up into the throat, may indicate pulmonary edema. Foul-smelling sputum and bad breath may indicate a lung abscess, bronchiectasis, or an infection caused by fusospirochetal or other anaerobic organisms.

A patient exhibited signs of an altered ventilation-perfusion ratio. The nurse is aware that adequate ventilation but impaired perfusion exists when the patient has which of the following conditions?

Pulmonary embolism Explanation: When a blood clot exists in a pulmonary vessel (embolus), impaired perfusion results. However, ventilation is adequate. With the other choices, ventilation is impaired but perfusion is adequate.

Which of the following is a noninvasive method of continuously monitoring the oxygen saturation of hemoglobin (SaO2)?

Pulse oximetry Pulse oximetry is a noninvasive method of continuously monitoring SaO2. Measurements of blood pH of arterial oxygen and carbon dioxide tensions are obtained when managing patients with respiratory problems and adjusting oxygen therapy as needed. This is an invasive procedure. Pulmonary function testing assesses respiratory function and determines the extent of dysfunction. Sputum studies are done to identify if any pathogenic organisms or malignant cells are in the sputum.

The nurse is caring for a client whose respiratory status has declined since shift report. The client has tachypnea, is restless, and displays cyanosis. Which diagnostic test should be assessed first? Pulse oximetry Arterial blood gases Pulmonary function test Chest x-ray

Pulse oximetry Pulse oximetry is a noninvasive method to determine arterial oxygen saturation. Normal values are 95% and above. Using this diagnostic test first provides rapid information of the client's respiratory system. All other options vary in amount of time and patient participation in determining further information regarding the respiratory system.

Which of the following is a noninvasive method of continuously monitoring the oxygen saturation of hemoglobin (SaO2)?

Pulse oximetry Explanation: Pulse oximetry is a noninvasive method of continuously monitoring SaO2. Measurements of blood pH of arterial oxygen and carbon dioxide tensions are obtained when managing patients with respiratory problems and adjusting oxygen therapy as needed. This is an invasive procedure. Pulmonary function testing assesses respiratory function and determines the extent of dysfunction. Sputum studies are done to identify if any pathogenic organisms or malignant cells are in the sputum.

The nurse is teaching the client in respiratory distress ways to prolong exhalation to improve respiratory status. The nurse tells the client to

Purse the lips when exhaling air from the lungs. To prolong exhalation, the client may perform breathing while sitting in a chair or walking. The client is to inhale through the nose and then exhale against pursed lips. There is no holding the breath.

A student nurse is working with a client who is diagnosed with head trauma. The nurse has documented Cheyne-Stokes respirations. The student would expect to see which of the following?

Regular breathing where the rate and depth increase, then decrease Explanation: Observing the rate and depth of respiration is an important aspect of the nursing assessment. Certain patterns of breathing are characteristic of specific disease states or conditions. Head trauma can cause damage to the respiratory center in the brain, thereby altering the rate and depth of respirations. Cheyne-Stokes breathing is characterized by a regular cycle in which the rate and depth of breathing increase, then decrease until apnea occurs

The nurse is caring for a client who is ready to be weaned from the ventilator. In preparing to assist in the collaborative process of weaning, the nurse should anticipate that the weaning of the client will progress in what order?

Removal from the ventilator, tube, and then oxygen

A nurse is educating a client in anticipation of a procedure that will require a water-sealed chest drainage system. What should the nurse tell the client and the family that this drainage system is used for?

Removing excess air and fluid Explanation: Chest tubes and closed drainage systems are used to re-expand the lung involved and to remove excess air, fluid, and blood. They are not used to maintain positive chest-wall pressure, monitor pleural fluid, or provide positive intrathoracic pressure.

gas exchange between lungs and blood and between the blood and tissues is called

Respiration

The nurse is caring for a client with chronic obstructive pulmonary disease. The client reports that he is having difficulty breathing and is feeling fatigued. The nurse realizes that this client is at high risk for which condition? Respiratory alkalosis Respiratory acidosis Metabolic acidosis Metabolic alkalosis

Respiratory acidosis Respiratory acidosis occurs when the body is unable to blow off CO2 due to the hypoventilation of disease processes such as COPD. An increase in blood carbon dioxide concentration occurs and a decreased pH causing acidosis. Respiratory alkalosis is a decrease in acidity of the blood and often caused by hyperventilation. Metabolic acidosis and alkalosis are not directly caused by respiratory disorders.

The client has just had an invasive procedure to assess the respiratory system. What does the nurse know should be assessed on this client? Loss of consciousness Masses in pleural space Respiratory distress Watery sputum

Respiratory distress After invasive procedures, the nurse must carefully check for signs of respiratory distress and blood-streaked sputum. Masses in the pleural space are a condition that affects fremitus. General examination of overall health and condition includes assessing the consciousness of a client.

A client with chronic bronchitis is admitted to the health facility. Auscultation of the lungs reveals low-pitched, rumbling sounds. What breath sound should the nurse document? Venous hum Rales Bronchovesicular Rhonchi

Rhonchi Rhonchi or sonorous wheezes are deep, low-pitched, rumbling sounds heard usually on expiration. The etiology of rhonchi is associated with chronic bronchitis. Rales or crackles are soft, high-pitched sounds. A venous hum is a blood flow humming sound. Bronchovesicular sound is an intermediate pitch with expiration and inspiration.

A client with chronic bronchitis is admitted to the health facility. Auscultation of the lungs reveals low-pitched, rumbling sounds. What breath sound should the nurse document?

Rhonchi Explanation: Rhonchi or sonorous wheezes are deep, low-pitched, rumbling sounds heard usually on expiration. The etiology of rhonchi is associated with chronic bronchitis. Rales or crackles are soft, high-pitched sounds. A venous hum is a blood flow humming sound. Bronchovesicular sound is an intermediate pitch with expiration and inspiration.

A client with chronic bronchitis is admitted to the health facility. Auscultation of the lungs reveals low-pitched, rumbling sounds. Which of the following describes these sounds?

Rhonchi Explanation: Rhonchi are deep, low-pitched, rumbling sounds heard usually on expiration. The etiology of rhonchi is associated with chronic bronchitis.

The nurse is providing discharge instructions to a client with pulmonary sarcoidosis. The nurse concludes that the client understands the information if the client correctly mentions which early sign of exacerbation?

Shortness of breath Early signs and symptoms of pulmonary sarcoidosis may include dyspnea, cough, hemoptysis, and congestion. Generalized symptoms include anorexia, fatigue, and weight loss.

The nurse is caring for a client who has a pleural effusion and who underwent a thoracoscopic procedure earlier in the morning. The nurse should prioritize assessment for which of the following?

Shortness of breath Follow-up care in the health care facility and at home involves monitoring the client for shortness of breath (which might indicate a pneumothorax). All of the listed options are relevant assessment findings, but shortness of breath is the most serious complication.

The nurse is caring for a client who has just returned to the unit after a colon resection. The client is showing signs of hypoxia. The nurse knows that this is probably caused by what?

Shunting Shunting appears to be the main cause of hypoxia after thoracic or abdominal surgery and most types of respiratory failure. Impairment of normal diffusion is a less common cause. Infection would not likely be present at this early stage of recovery and nitrogen narcosis only occurs from breathing compressed air.

Which ventilation-perfusion ratio is exhibited by acute respiratory distress syndrome (ARDS)?

Silent unit Explanation: When ventilation exceeds perfusion a dead space exists. An example of a dead space is a pulmonary emboli. A low ventilation-perfusion ratio exists in pneumonia or with a mucus plug. A silent unit occurs in pneumothorax or ARDS

The nurse is caring for a client who is to undergo a thoracentesis. In preparation for the procedure, the nurse places the client in which position? Sitting on the edge of the bed Supine Prone Lateral recumbent

Sitting on the edge of the bed If possible, it is best to place the client upright or sitting on the edge of the bed with the feet supported and arms and head on a padded over-the-bed table. Other positions in which the client could be placed include straddling a chair with arms and head resting on the back of the chair, or lying on the unaffected side with the head of the bed elevated 30 to 45 degrees (if the client is unable to assume a sitting position).

In which position should the client be placed for a thoracentesis?

Sitting on the edge of the bed If possible, place the client upright or sitting on the edge of the bed with the feet supported and arms and head on a padded over-the-bed table. Other positions in which the client could be placed include straddling a chair with the arms and head resting on the back of the chair, or lying on the unaffected side with the head of the bed elevated 30 to 45 degrees (if the client is unable to assume a sitting position).

The nurse is caring for a client who is to undergo a thoracentesis. In preparation for the procedure, the nurse places the client in which position?

Sitting on the edge of the bed Explanation: If possible, it is best to place the client upright or sitting on the edge of the bed with the feet supported and arms and head on a padded over-the-bed table. Other positions in which the client could be placed include straddling a chair with arms and head resting on the back of the chair, or lying on the unaffected side with the head of the bed elevated 30 to 45 degrees (if the client is unable to assume a sitting position).

The critical care nurse and the other members of the care team are assessing the client to see if he is ready to be weaned from the ventilator. What are the most important predictors of successful weaning that the nurse should identify?

Stable vital signs and ABGs Among many other predictors, stable vital signs and ABGs are important predictors of successful weaning. Pulse oximetry must greatly exceed 80%. Nutritional status is important, but vital signs and ABGs are even more significant. Clients who are weaned may or may not have full level of consciousness.

A client with myasthenia gravis is receiving continuous mechanical ventilation. When the high-pressure alarm on the ventilator sounds, what should the nurse do?

Suction the client's artificial airway. Explanation: A high-pressure alarm on a continuous mechanical ventilator indicates an obstruction in the flow of gas from the machine to the client. The nurse should suction the client's artificial airway to remove respiratory secretions that could be causing the obstruction. The sounding of a ventilator alarm has no relationship to the apical pulse. Increasing the oxygen percentage and ventilating with a handheld mechanical ventilator wouldn't correct the airflow blockage.

The nurse is admitting a client who just had a bronchoscopy. Which assessment should be the nurse's priority?

Swallow reflex

The nurse is assisting a physician with an endotracheal intubation for a client in respiratory failure. It is most important for the nurse to assess for:

Symmetry of the client's chest expansion Explanation: Immediately after intubation, the nurse should check for symmetry of chest expansion. This is one finding that indicates successful endotracheal placement. The tracheal cuff pressure is set between 15 and 20 mm Hg. Routine deflation of the tracheal cuff is not recommended, because the client could then aspirate secretions during the period of deflation. Warm, high, humidified air is administered through the endotracheal tube.

The nurse receives an order to obtain a sputum sample from a client with hemoptysis. When advising the client of the physician's order, the client states not being able to produce sputum. Which suggestion, offered by the nurse, is helpful in producing the sputum sample? Take deep breaths and cough forcefully. Drink 8 oz of water to thin the secretions for expectoration. Use the secretions present in the oral cavity. Tickle the back of the throat to produce the gag reflex.

Take deep breaths and cough forcefully. Taking deep breaths moves air around the sputum and coughing forcefully moves the sputum up the respiratory tract. Once in the pharynx, the sputum can be expectorated into a specimen container. Producing a gag reflex elicits stomach contents and not respiratory sputum. Dilute and thinned secretions are not helpful in aiding expectoration. A sputum culture is not a component of oral secretions.

For which reason does gas exchange decrease in older adults?

The alveolar walls contain fewer capillaries. Although the number of alveoli remains stable with age, the alveolar walls become thinner and contain fewer capillaries, resulting in decreased gas exchange. The lungs also lose elasticity and become stiffer. Elasticity of lungs does not increase with age, and the number of alveoli does not decrease with age.

For which reason does gas exchange decrease in older adults?

The alveolar walls contain fewer capillaries. Explanation: Although the number of alveoli remains stable with age, the alveolar walls become thinner and contain fewer capillaries, resulting in decreased gas exchange. The lungs also lose elasticity and become stiffer. Elasticity of lungs does not increase with age, and the number of alveoli does not decrease with age.

A client arrives at the physician's office stating 2 days of febrile illness, dyspnea, and cough. Upon assisting the client into a gown, the nurse notes that the client's sternum is depressed, especially on inspiration. Crackles are noted in the bases of the lung fields. Based on inspection, which will the nurse document?

The client has a funnel chest. Explanation: The question asks for a documentation based on inspection. A funnel chest, known as pectus excavatum, has the sternum depressed from the second intercostal space, and it is more pronounced on inspiration. The nurse would not diagnose chronic respiratory disease or pneumonia. The client would also not prescribe a cough suppressant.

The nurse caring for a client with an endotracheal tube recognizes several disadvantages of an endotracheal tube. What would the nurse recognize as a disadvantage of endotracheal tubes?

The cough reflex is depressed.

The nurse caring for a client with an endotracheal tube recognizes several disadvantages of an endotracheal tube. What would the nurse recognize as a disadvantage of endotracheal tubes?

The cough reflex is depressed. There are several disadvantages of an endotracheal tube. Disadvantages include suppression of the client's cough reflex, thickening of secretions, and depressed swallowing reflexes. Ulceration and stricture of the larynx or trachea may develop, but bleeding is not an expected finding. The tube should not influence cognition and daily ABGs are not always required.

The nurse caring for a client with an endotracheal tube recognizes several disadvantages of an endotracheal tube. What would the nurse recognize as a disadvantage of endotracheal tubes?

The cough reflex is depressed. Explanation: There are several disadvantages of an endotracheal tube. Disadvantages include suppression of the client's cough reflex, thickening of secretions, and depressed swallowing reflexes. Ulceration and stricture of the larynx or trachea may develop, but bleeding is not an expected finding. The tube should not influence cognition and daily ABGs are not always required.

The client is returning from the operating room following a bronchoscopy. Which action, performed by the nursing assistant, would the nurse stop if began prior to nursing assessment? The nursing assistant is pouring a glass of water to wet the client's mouth. The nursing assistant is assisting the client to a semi-Fowler's position. The nursing assistant is asking a question requiring a verbal response. The nursing assistant is assisting the client to the side of the bed to use a urinal.

The nursing assistant is pouring a glass of water to wet the client's mouth. When completing a procedure which sends a scope down the throat, the gag reflex is anesthetized to reduce discomfort. Upon returning to the nursing unit, the gag reflex must be assessed before providing any food or fluids to the client. The client may need assistance following the procedure for activity and ambulation but this is not restricted in the post procedure period.

The nurse suctions a patient through the endotracheal tube for 20 seconds and observes dysrhythmias on the monitor. What does the nurse determine is occurring with the patient?

The patient is hypoxic from suctioning. Apply suction while withdrawing and gently rotating the catheter 360 degrees (no longer than 10-15 seconds). Prolonged suctioning may result in hypoxia and dysrhythmias, leading to cardiac arrest.

A client experiences a head injury in a motor vehicle accident. The client's level of consciousness is declining, and respirations have become slow and shallow. When monitoring a client's respiratory status, which area of the brain would the nurse realize is responsible for the rate and depth? Central sulcus Wernicke's area The pons The frontal lobe

The pons The inspiratory and expiratory centers in the medulla oblongata and pons control the rate and depth of ventilation. When injury occurs or increased intracranial pressure results, respirations are slowed. The frontal lobe completes executive functions and cognition. The central sulcus is a fold in the cerebral cortex called the central fissure. The Wernicke's area is the area linked to speech.

While assessing the client, the nurse observes constant bubbling in the water-seal chamber of the client's closed chest-drainage system. What should the nurse conclude?

The system has an air leak. Constant bubbling in the chamber often indicates an air leak and requires immediate assessment and intervention. The client with a pneumothorax will have intermittent bubbling in the water-seal chamber. If the tube is obstructed, the nurse should notice that the fluid has stopped fluctuating in the water-seal chamber.

While assessing the client, the nurse observes constant bubbling in the water-seal chamber of the client's closed chest-drainage system. What should the nurse conclude?

The system has an air leak. Explanation: Constant bubbling in the chamber often indicates an air leak and requires immediate assessment and intervention. The client with a pneumothorax will have intermittent bubbling in the water-seal chamber. If the tube is obstructed, the nurse should notice that the fluid has stopped fluctuating in the water-seal chamber.

Perfusion refers to blood supply to the lungs, through which the lungs receive nutrients and oxygen. What are the two methods of perfusion? The two methods of perfusion are the alveolar and pulmonary circulation. The two methods of perfusion are the bronchial and alveolar circulation. The two methods of perfusion are the bronchial and capillary circulation. The two methods of perfusion are the bronchial and pulmonary circulation.

The two methods of perfusion are the bronchial and pulmonary circulation. Perfusion refers to blood supply to the lungs, through which the lungs receive nutrients and oxygen. The two methods of perfusion are the bronchial and pulmonary circulation.

Perfusion refers to blood supply to the lungs, through which the lungs receive nutrients and oxygen. What are the two methods of perfusion?

The two methods of perfusion are the bronchial and pulmonary circulation. Explanation: The two methods of perfusion are the bronchial and pulmonary circulation. There is no alveolar circulation. Capillaries are the vessels that performs the perfusion regardless of which area of the lung they are in.

The nurse is caring for a client diagnosed with asthma. While performing the shift assessment, the nurse auscultates breath sounds including sibilant wheezes, which are continuous musical sounds. What characteristics describe sibilant wheezes? They can be heard during inspiration and expiration. They occur when the pleural surfaces are inflamed. They result from air passing through widened air passages. They are heard in clients with decreased secretions.

They can be heard during inspiration and expiration. Sibilant or hissing or whistling wheezes are continuous musical sounds that can be heard during inspiration and expiration. They result from air passing through narrowed or partially obstructed air passages and are heard in clients with increased secretions. The crackling or grating sounds heard during inspiration or expiration are friction rubs. They occur when the pleural surfaces are inflamed.

The nurse is caring for a client diagnosed with asthma. While performing the shift assessment, the nurse auscultates breath sounds including sibilant wheezes, which are continuous musical sounds. What characteristics describe sibilant wheezes?

They can be heard during inspiration and expiration. Explanation: Sibilant or hissing or whistling wheezes are continuous musical sounds that can be heard during inspiration and expiration. They result from air passing through narrowed or partially obstructed air passages and are heard in clients with increased secretions. The crackling or grating sounds heard during inspiration or expiration are friction rubs. They occur when the pleural surfaces are inflamed.

A client in acute respiratory distress is brought to the emergency department. After endotracheal (ET) intubation and initiation of mechanical ventilation, the client is transferred to the intensive care unit. Before suctioning the ET tube, the nurse hyperventilates and hyperoxygenates the client. What is the rationale for these interventions?

They help prevent cardiac arrhythmias. ET suctioning removes oxygen, lowering the partial pressure of arterial oxygen; this, in turn, may induce a cardiac arrhythmia. Hyperventilating and hyperoxygenating the client before and during (or after) suctioning helps prevent this complication. Subcutaneous emphysema occurs when air from the pleural cavity leaks into subcutaneous tissue; it isn't a complication associated with suctioning. Hyperventilation and hyperoxygenation can't prevent a pneumothorax because this condition itself indicates air in the pleural space. Pulmonary edema is associated with cardiac dysfunction, not ET suctioning.

Which term refers to the volume of air inhaled or exhaled during each respiratory cycle? Functional residual capacity Maximal voluntary ventilation Vital capacity Tidal volume

Tidal volume Tidal volume refers to the volume of air inhaled or exhaled during each respiratory cycle when breathing normally. Normal tidal volume ranges from 400 to 700 ml. Vital capacity refers to the total volume of air that can be exhaled during a slow, maximal expiration after maximal inspiration. Functional residual capacity refers to the volume of air remaining in the lungs after a normal expiration. Maximal voluntary ventilation is the greatest volume of air expired in 1 minute with maximal voluntary effort.

Which term refers to the volume of air inhaled or exhaled during each respiratory cycle?

Tidal volume Explanation: Tidal volume refers to the volume of air inhaled or exhaled during each respiratory cycle when breathing normally. Normal tidal volume ranges from 400 to 700 ml. Vital capacity refers to the total volume of air that can be exhaled during a slow, maximal expiration after maximal inspiration. Functional residual capacity refers to the volume of air remaining in the lungs after a normal expiration. Maximal voluntary ventilation is the greatest volume of air expired in 1 minute with maximal voluntary effort.

A client is exhibiting signs of a pneumothorax following tracheostomy. The surgeon inserts a chest tube into the anterior chest wall. What should the nurse tell the family is the primary purpose of this chest tube?

To remove air from the pleural space Chest tubes and closed drainage systems are used to re-expand the lung involved and to remove excess air, fluid, and blood. The primary purpose of a chest tube is not to drain sputum secretions, monitor bleeding, or assist with mechanical ventilation.

A medical client rings her call bell and expresses alarm to the nurse, stating, "I've just coughed up this blood. That can't be good, can it?" How can the nurse best determine whether the source of the blood was the client's lungs?

Try to see if the blood is frothy or mixed with mucus. Though not definitive, blood from the lung is usually bright red, frothy, and mixed with sputum. Testing the pH of nonarterial blood samples is not common practice and would not provide important data. Similarly, oral suctioning and swabbing the client's mouth would not reveal the source.

A nurse is assessing a client's respiratory system. Which alveolar cells secrete surfactant to reduce lung surface tension? Type I Type IV Type II Macrophages

Type II There are three types of alveolar cells. Type I and type II cells make up the alveolar epithelium. Type I cells account for 95% of the alveolar surface area and serve as a barrier between the air and the alveolar surface; type II cells account for only 5% of this area but are responsible for producing type I cells and surfactant. Surfactant reduces surface tension, thereby improving overall lung function. Alveolar macrophages, the third type of alveolar cells, are phagocytic cells that ingest foreign matter and, as a result, provide an important defense mechanism. Type IV is not a category of alveolar cells.

_____________________ are the alveolar cells that secrete surfactant

Type II cells

A nurse is discussing squamous epithelial cells lining each alveolus, which consist of different types of cells. Which type of alveolar cells produce surfactant? Type II cells Type III cells Type IV cells Type I cells

Type II cells There are three types of alveolar cells. Type I and type II cells make up the alveolar epithelium. Type I cells account for 95% of the alveolar surface area and serve as a barrier between the air and the alveolar surface; type II cells account for only 5% of this area, but are responsible for producing type I cells and surfactant. Surfactant reduces surface tension, thereby improving overall lung function. Alveolar macrophages, the third type of alveolar cells, are phagocytic cells that ingest foreign matter and, as a result, provide an important defense mechanism. The epithelium of the alveoli does not contain Type IV cells.

A nurse is discussing squamous epithelial cells lining each alveolus, which consist of different types of cells. Which type of alveolar cells produce surfactant?

Type II cells Explanation: There are three types of alveolar cells. Type I and type II cells make up the alveolar epithelium. Type I cells account for 95% of the alveolar surface area and serve as a barrier between the air and the alveolar surface; type II cells account for only 5% of this area, but are responsible for producing type I cells and surfactant. Surfactant reduces surface tension, thereby improving overall lung function. Alveolar macrophages, the third type of alveolar cells, are phagocytic cells that ingest foreign matter and, as a result, provide an important defense mechanism. The epithelium of the alveoli does not contain Type IV cells.

Mr. Sam Wallace, a 53-year-old male, is a regular client in the respiratory group where you practice nursing. As with all adults, millions of alveoli form most of the pulmonary mass. The squamous epithelial cells lining each alveolus consist of different types of cells. Which type of the alveoli cells produce surfactant?

Type II cells Explanation: Type II cells—produce surfactant, a phospholipid that alters the surface tension of alveoli, preventing their collapse during expiration and limiting their expansion during inspiration

A nurse assesses a client's respiratory status. Which observation indicates that the client is having difficulty breathing?

Use of accessory muscles Explanation: The use of accessory muscles for respiration indicates the client is having difficulty breathing. Diaphragmatic and pursed-lip breathing are two controlled breathing techniques that help the client conserve energy.

A nurse caring for a patient with a pulmonary embolism understands that high ventilation-perfusion ratio may exist. What does this mean for the patient?

Ventilation exceeds perfusion

A nurse caring for a patient with a pulmonary embolism understands that a high ventilation-perfusion ratio may exist. What does this mean for the patient?

Ventilation exceeds perfusion. A high ventilation-perfusion rate means that ventilation exceeds perfusion, causing dead space. The alveoli do not have an adequate blood supply for gas exchange to occur. This is characteristic of a variety of disorders, including pulmonary emboli, pulmonary infarction, and cardiogenic shock.

A nurse caring for a patient with a pulmonary embolism understands that a high ventilation-perfusion ratio may exist. What does this mean for the patient? Ventilation matches perfusion. Perfusion exceeds ventilation. There is an absence of perfusion and ventilation. Ventilation exceeds perfusion.

Ventilation exceeds perfusion. A high ventilation-perfusion rate means that ventilation exceeds perfusion, causing dead space. The alveoli do not have an adequate blood supply for gas exchange to occur. This is characteristic of a variety of disorders, including pulmonary emboli, pulmonary infarction, and cardiogenic shock.

A nurse caring for a patient with a pulmonary embolism understands that a high ventilation-perfusion ratio may exist. What does this mean for the patient?

Ventilation exceeds perfusion. Explanation: A high ventilation-perfusion rate means that ventilation exceeds perfusion, causing dead space. The alveoli do not have an adequate blood supply for gas exchange to occur. This is characteristic of a variety of disorders, including pulmonary emboli, pulmonary infarction, and cardiogenic shock.

A client with a severe exacerbation of COPD requires reliable and precise oxygen delivery. Which mask will the nurse expect the health care provider to prescribe?

Venturi mask

A client with a severe exacerbation of COPD requires reliable and precise oxygen delivery. Which mask will the nurse expect the health care provider to prescribe?

Venturi mask The Venturi mask provides the most accurate method of oxygen delivery. Other methods of oxygen delivery include the aerosol mask, tracheostomy collar, and face tents, but these do not match the precision of a Venturi mask.

Most cases of acute pharyngitis are caused by which of the following?

Viral infection Explanation: Most cases of acute pharyngitis are caused by viral infection. Responsible viruses include the adenovirus, influenza virus, Epstein-Barr virus, and herpes simplex virus.

Which type of ventilator has a preset volume of air to be delivered with each inspiration?

Volume cycled With volume-cycled ventilation, the volume of air to be delivered with each inspiration is preset. Negative-pressure ventilators exert a negative pressure on the external chest. Time-cycled ventilators terminate or control inspiration after a preset time. When the pressure-cycled ventilator cycles on, it delivers a flow of air (inspiration) until it reaches a preset pressure, and then cycles off, and expiration occurs passively.

What is the purpose of the vascular and ciliated mucous lining of the nasal cavities? Move mucus to the back of the throat Moisten and filter expired air Cool and dry expired air Warm and humidify inspired air

Warm and humidify inspired air The vascular and ciliated mucous lining of the nasal cavities warms and humidifies inspired air. It is the function of the cilia alone to move mucus in the nasal cavities and filter the inspired air.

What is the purpose of the vascular and ciliated mucous lining of the nasal cavities?

Warm and humidify inspired air Explanation: The vascular and ciliated mucous lining of the nasal cavities warms and humidifies inspired air. It is the function of the cilia alone to move mucus in the nasal cavities and filter the inspired air.

After lobectomy for lung cancer, a client receives a chest tube connected to a disposable chest drainage system. The nurse observes that the drainage system is functioning correctly when she notes tidal movements or fluctuations in which compartment of the system as the client breathes?

Water-seal chamber Fluctuations in the water-seal compartment are called tidal movements and indicate normal function of the system as the pressure in the tubing changes with the client's respirations. The air-leak meter — not chamber — detects air leaking from the pleural space. The collection chamber connects the chest tube from the client to the system. Drainage from the tube drains into and collects in a series of calibrated columns in this chamber. The suction control chamber provides the suction, which can be controlled to provide negative pressure to the chest.

The nurse is performing chest auscultation for a PT with asthma. How does the nurse describe the high-pitched, sibilant, musicals sounds that are heard?

Wheezes

The nurse is caring for a client with a decrease in airway diameter causing airway resistance. The client experiences coughing and mucus production. Upon lung assessment, which adventitious breath sounds are anticipated? Crackles Wheezes Rhonchi Rubs

Wheezes A decrease in airway diameter, such as in asthma, produces breath sounds of wheezes. Wheezes are a whistling type of sound relating to the narrowing on the airway. A wheeze can have a high-pitched or low-pitched quality. Crackles, also noted as rales, are crackling or rattling sounds signifying fluid or exudate in the lung fields. Rhonchi are a course rattling sound similar to snoring usually caused by secretion in the bronchial tree. Rubs are secretions that can be heard in the large airway.

While auscultating the lungs of a client with asthma, the nurse hears a continuous, high-pitched whistling sound on expiration. The nurse will document this sound as which of the following?

Wheezes Wheezes, usually heard on expiration, are continuous, musical, high pitched, and whistle-like sounds caused by air passing through narrowed airways. Often, wheezes are associated with asthma.

The nurse is caring for a client with a decrease in airway diameter causing airway resistance. The client experiences coughing and mucus production. Upon lung assessment, which adventitious breath sounds are anticipated?

Wheezes Explanation: A decrease in airway diameter, such as in asthma, produces breath sounds of wheezes. Wheezes are a whistling type of sound relating to the narrowing on the airway. A wheeze can have a high-pitched or low-pitched quality. Crackles, also noted as rales, are crackling or rattling sounds signifying fluid or exudate in the lung fields. Rhonchi are a course rattling sound similar to snoring usually caused by secretion in the bronchial tree. Rubs are secretions that can be heard in the large airway.

The nurse is performing chest auscultation for a patient with asthma. How does the nurse describe the high-pitched, sibilant, musical sounds that are heard?

Wheezes Explanation: Sibilant wheezes are continuous, musical, high-pitched, whistle-like sounds heard during inspiration and expiration caused by air passing through narrowed or partially obstructed airways; they may clear with coughing. Crackles, formerly called rales, are soft, high-pitched, discontinuous popping sounds that occur during inspiration (while usually heard on inspiration, they may also be heard on expiration); they may or may not be cleared by coughing. Rhonchi, or sonorous wheezes, are deep, low-pitched rumbling sounds heard primarily during expiration; they are caused by air moving through narrowed tracheobronchial passages.

The nurse is caring for a client who has been scheduled for a bronchoscopy. How should the nurse prepare the client for this procedure?

Withhold food and fluids for several hours before the test. Food and fluids are withheld for 4 to 8 hours before the test to reduce the risk of aspiration when the cough reflex is blocked by anesthesia. IV fluids, bronchodilators, and a central line are unnecessary.

A client is chronically short of breath and yet has normal lung ventilation, clear lungs, and an arterial oxygen saturation SaO2 of 96% or better. The client most likely has:

a possible hematologic problem. Explanation: SaO2 is the degree to which hemoglobin (Hb) is saturated with oxygen. It doesn't indicate the client's overall Hb adequacy. Thus, an individual with a subnormal Hb level could have normal SaO2 and still be short of breath, indicating a possible hematologic problem. Poor peripheral perfusion would cause subnormal SaO2. There isn't enough data to assume that the client's problem is psychosomatic. If the problem were left-sided heart failure, the client would exhibit pulmonary crackles

A client is being seen in the pediatric clinic for a middle ear infection. The client's mother reports that when the client develops an upper respiratory infection, an ear infection seems quick to follow. What contributes to this event?

eustachian tubes The nasopharynx contains the adenoids and openings of the eustachian tubes. The eustachian tubes connect the pharynx to the middle ear and are the means by which upper respiratory infections spread to the middle ear. The client's infection is not caused by genetics. The oropharynx contains the tongue. The epiglottis closes during swallowing and relaxes during respiration.

A client is being seen in the pediatric clinic for a middle ear infection. The client's mother reports that when the client develops an upper respiratory infection, an ear infection seems quick to follow. What contributes to this event? epiglottis eustachian tubes oropharynx genetics

eustachian tubes The nasopharynx contains the adenoids and openings of the eustachian tubes. The eustachian tubes connect the pharynx to the middle ear and are the means by which upper respiratory infections spread to the middle ear. The client's infection is not caused by genetics. The oropharynx contains the tongue. The epiglottis closes during swallowing and relaxes during respiration.

What are the primary functions of the lungs? Select all that apply.

gas exchange ventilation The primary functions of the lungs include ventilation and gas exchange.

A client has been newly diagnosed with emphysema. The nurse should explain to the client that by definition, ventilation:

is breathing air in and out of the lungs

A client has been newly diagnosed with emphysema. The nurse should explain to the client that by definition, ventilation:

is breathing air in and out of the lungs. Explanation: Ventilation is the actual movement of air in and out of the respiratory tract. Diffusion is the exchange of oxygen and CO2 through the alveolar-capillary membrane. Pulmonary perfusion refers to the provision of blood supply to the lungs. A mechanical ventilator assists patients who are unable to breathe on their own.

The nurse needs to make the appropriate room assignment based on the client's problems, safety, and risk for infection to others. The client with an upper respiratory infection may transmit infection to susceptible people. Clients A, C, and D have increased susceptibility for infection because of immunosuppression or surgery.

laryngeal cancer is one of the most preventable types of cancer. Laryngeal cancer is one of the most preventable types of cancer; it can be prevented by abstaining from excessive drinking and smoking. Inhaling noxious fumes, such as in polluted air, is a risk factor for laryngeal cancer. Roughly 80% of laryngeal cancer cases occur in men. Squamous cell carcinoma accounts for most cases of laryngeal cancer.

A Black client with asthma seeks emergency care for acute respiratory distress. Because of this client's dark skin, the nurse should assess for cyanosis by inspecting the:

mucous membranes. Explanation: Skin color doesn't affect the mucous membranes. Therefore, the nurse can assess for cyanosis by inspecting the client's mucous membranes. The lips, nail beds, and earlobes are less-reliable indicators of cyanosis because they're affected by skin color.

The left lung in contrast to the right lung, has?

one less lobe

The nurse is interpreting blood gases for a client with acute respiratory distress syndrome (ARDS). Which set of blood gas values indicates respiratory acidosis?

pH 7.25, PaCO2 48, HCO3 24 pH less than 7.35, PaCO2 48, HCO3 24 indicate respiratory acidosis; pH 7.87, PaCO2 38, HCO3 28 indicate metabolic alkalosis; pH 7.47, PaCO2 28, HCO3 30 indicate respiratory alkalosis; and pH 7.49, PaCO2 34, HCO3 25 indicate respiratory alkalosis.

The lungs are enclosed in a serous membrane called the

pleura

A physician stated to the nurse that the client has fluid in the pleural space and will need a thoracentesis. The nurse expects the physician to document this fluid as

pleural effusion. Fluid accumulating within the pleural space is called a pleural effusion. A pneumothorax is air in the pleural space. A hemothorax is blood within the pleural space. Consolidation is lung tissue that has become more solid in nature as a result of the collapse of alveoli or an infectious process.

The nurse auscultates the lung sounds of a client during a routine assessment. The sounds produced are harsh and cracking, sounding like two pieces of leather being rubbed together. The nurse would be correct in documenting this finding as

pleural friction rub. A pleural friction rub is heard secondary to inflammation and loss of lubricating pleural fluid. Crackles are soft, high-pitched, discontinuous popping sounds that occur during inspiration. Sonorous wheezes are deep, low-pitched rumbling sounds heard primarily during expiration. Sibilant wheezes are continuous, musical, high-pitched, whistle-like sounds heard during inspiration and expiration.

Pink, frothy sputum may be an indication of

pulmonary edema. Explanation: Profuse frothy, pink material, often welling up into the throat, may indicate pulmonary edema. Foul-smelling sputum and bad breath may indicate a lung abscess, bronchiectasis, or an infection caused by fusospirochetal or other anaerobic organisms.

Pink frothy sputum may be an indication of

pulmonary edema. Explanation: Profuse, frothy pink material, often welling up into the throat, may indicate pulmonary edema. Foul-smelling sputum and bad breath may indicate a lung abscess, bronchiectasis, or an infection caused by fusospirochetal or other anaerobic organisms

A nurse is caring for a client who was intubated because of respiratory failure. The client is now receiving mechanical ventilation with a preset tidal volume and number of breaths each minute. The client has the ability to breathe spontaneously between the ventilator breaths with no ventilator assistance. The nurse should document the ventilator setting as:

synchronized intermittent mandatory ventilation (SIMV) In SIMV mode, the ventilator delivers a preset number of breaths at a preset tidal volume. The client can breathe on his own in between the breaths delivered by the ventilator. In PSV, a pressure plateau is added to the ventilator to prevent the airway pressure from falling beneath a preset level. In AC ventilation, the ventilator delivers a preset number of breaths at a preset tidal volume and any breaths that the client takes on his own are assisted by the ventilator so they reach the preset tidal volume. In CPAP, the ventilator provides only positive airway pressure; it doesn't provide any breaths to the client.

The volume of air inhaled and exhaled with each breath is termed

tidal volume. Tidal volume is the volume of air inhaled and exhaled with each breath. Residual volume is the volume of air remaining in the lungs after a maximum expiration. Vital capacity is the maximum volume of air exhaled from the point of maximum inspiration. Expiratory reserve volume is the maximum volume of air that can be exhaled after a normal inhalation.

Which hollow tube transports air from the laryngeal pharynx to the bronchi? pharynx larynx trachea bronchioles

trachea The trachea is a hollow tube composed of smooth muscle and supported by C-shaped cartilage. The trachea transports air from the laryngeal pharynx to the bronchi and lungs. This is a cartilaginous framework between the pharynx and trachea that produces sound. The bronchioles are smaller subdivisions of bronchi within the lungs. The pharynx, or throat, carries air from the nose to the larynx and food from the mouth to the esophagus.

Choice Multiple question - Select all answer choices that apply. Which of the following are assessment findings associated with thrombocytopenia? Select all that apply. a) Hypertension b) Bradypnea c) Bleeding gums d) Hematemesis e) Epistaxis

• Bleeding gums • Epistaxis • Hematemesis Explanation: Pertinent findings of thrombocytopenia include: bleeding gums, epistaxis, hematemesis, hypotension, and tachypnea.

Choice Multiple question - Select all answer choices that apply. Which of the following are immunosuppressants that prevent the patient's lymphocytes from destroying the stem cells in a patient diagnosed with aplastic anemia? Select all that apply. a) Cyclosporine b) ATG c) Filgrastim d) Corticosteroids e) Cyclophosphamide

• Cyclosporine • Corticosteroids • ATG • Cyclophosphamide Explanation: Immunosuppressants, including cyclosporine, corticosteroids, ATG, and cyclophosphamide, prevent the patient's lymphocytes from destroying the stem cells. Filgrastim is a growth-colony stimulating factor that may be used to decease the duration and severity of neutropenia associated with aplastic anemia

A client has suspected fluid accumulation in the pleural space of the lungs and is scheduled for a thoracentesis. The nurse will implement which of the following for this procedure? Select all that apply.

• Educate the client about the need to cleanse the thoracic area. • Apply pressure to the puncture site after the procedure. • Complete a respiratory assessment after the procedure. Explanation: A thoracentesis is performed to aspirate fluid or air from the pleural space. The nurse assists the client to a sitting or side-lying position, which provides support and exposes the base of the thorax. Encouraging a position of comfort helps the client to relax for the procedure. The nurse prepares the client by explaining the steps of the procedure and begins by cleansing the thoracic area using aseptic technique. After the procedure, the nurse applies pressure to the site to help stop bleeding; then, he or she applies an air-tight, sterile dressing. A chest x-ray verifies that there is no pneumonthorax. The nurse will monitor at intervals the client's respiratory function

The thalassaemia's are a group of hereditary anaemias characterised by which of the following? Select all that apply.

• Hypochromia • Extreme microcytosis • Haemolysis • Anaemia Explanation: The thalassaemia's are a group of hereditary anaemias characterised by hypochromia, extreme microcytosis, destruction of blood elements (haemolysis), and variable degrees of anaemia. Thrombocytopenia is not associated with thalassaemia's.

Willy Trout, a 7-year-old boy, has been brought to the ED where you practice nursing. Willy and some friends were having a contest to see who could throw a piece of candy the highest and catch it in their open mouth. As a result, Willy aspirated candy, which brings him to you via ambulance. Where would be the most likely place to find the aspirated piece of candy?

• Right mainstem bronchus • Right upper lung Explanation: Aspiration of foreign objects is more likely in the right upper lung

A client with chronic bronchitis is admitted with an exacerbation of symptoms. During the nursing assessment, the nurse will expect which of the following findings? Select all that apply.

• Use of accessory muscles to breathe • Purulent sputum with frequent coughing Explanation: Chronic bronchitis increases airway resistance and can thicken bronchial mucosa during an exacerbation. The client will have dyspnea requiring the use of accessory muscles to breathe, along with tachypnea and sputum production. Bronchial irritation and the need to expectorate mucus will lead to coughing. Percussion in this client would lead to resonant or hyperresonant sounds.

Choice Multiple question - Select all answer choices that apply. A 13-year-old client at the pulmonary clinic where you practice nursing has an extensive history of asthma and is seeing the pulmonologist for her monthly appointment. What are the primary functions of the lungs? Choose all correct options. a) Oxygen production b) Gas exchange c) Destroying CO d) Ventilation

• Ventilation • Gas exchange Explanation: The primary functions of the lungs include ventilation and gas exchange.


Conjuntos de estudio relacionados

Managerial Accounting Exam 1 (Ch. 14, 1, + 2)

View Set

Medical Terminology Chapter 6 PART TWO

View Set

Personal Health: Final Exam, Nursing 100 Final, Personal health Midterm

View Set

Study Guide Chapters 1-9 (Organizational Behavior)

View Set

3.1.11 - Social Engineering (Practice Questions)

View Set

Wordly Wise 3000 book 9 chapter 10

View Set